You are on page 1of 153

Kacmarek: Egan's Fundamentals of Respiratory Care, 10th Edition

Chapter41: Respiratory Failure and the Need for Ventilatory Support

Test Bank

MULTIPLE CHOICE

1. A diagnosis of respiratory failure can be made if which of the following are present?
1. PaO2 55 mm Hg, FIO2 0.21, PB 760 mm Hg
2. PaCO2 57 mm Hg, FIO2 0.21, PB 760 mm Hg
3. P(Aa)O2 45 mm Hg, FIO2 1.0, PB 760 mm Hg
4. PaO2/FIO2 400, PB 750 mm Hg
a. 1 and 2
b. 1, 3, and 4
c. 3 and 4
d. 1, 2, 3, and 4

ANS: A
Criteria for respiratory failure based on arterial blood gases have been established by Campbell
and generally define failure as a PaO2 (arterial partial pressure of oxygen) less than 60 mm Hg
and/or a PaCO2 (alveolar partial pressure of carbon dioxide) greater than 50 mm Hg in otherwise
healthy individuals breathing room air at sea level.

DIF: Application REF: p. 990 OBJ: 1

2. What is respiratory failure due to inadequate ventilation?


a. hypoxemic
b. hypercapnic
c. compensated
d. chronic

ANS: B
Hypercapnic (type II) respiratory failure describes “bellows failure” of the lungs resulting in
elevated carbon dioxide levels.

DIF: Recall REF: p. 990 OBJ: 2

3. Hypercapnic (type II) respiratory failure is a synonym for which one of the following terms?
a. mismatching
b. shunt
c. diffusion impairment
d. ventilatory failure

ANS: D

Copyright © 2013, 2009, 2003, 1999, 1995, 1990, 1982, 1977, 1973, 1969 by Mosby, an imprint of Elsevier Inc.
Test bank 41-2

Hypercapnic respiratory failure is also known as ventilatory failure.

DIF: Recall REF: p. 990 OBJ: 2

4. Hypoxemia can be caused by which of the following?


1. diffusion impairment
2. alveolar hypoventilation
3. mismatch
4. intrapulmonary shunting
a. 1, 2, and 3
b. 1, 3, and 4
c. 1, 2, 3, and 4
d. 2, 3, and 4

ANS: C
Hypoxemia can be caused by mismatch, shunt, alveolar hypoventilation, diffusion
impairment, perfusion impairment, decreased inspired oxygen, and venous admixture.

DIF: Recall REF: p. 1003 OBJ: 2

5. Which of the following best describes the difference between mismatch and shunt when
supplemental oxygen is administered?
a. Both will respond equally well.
b. mismatch will respond well but shunt will not.
c. mismatch will not respond but shunt will respond well.
d. Neither will respond to the administration of supplemental oxygen.

ANS: B
mismatch will respond to supplemental oxygen.

DIF: Recall REF: p. 991 OBJ: 2

6. Which of the following clinical signs suggest more severe hypoxemia?


a. tachycardia
b. cyanosis with polycythemia
c. central nervous system dysfunction
d. use of accessory muscles

ANS: C
More severe hypoxemia can lead to significant central nervous system dysfunction, ranging from
irritability to confusion to coma.

DIF: Recall REF: p. 990 OBJ: 2

Copyright © 2013, 2009, 2003, 1999, 1995, 1990, 1982, 1977, 1973, 1969 by Mosby, an imprint of Elsevier Inc.
Test bank 41-3

7. Which of the following clinical signs is most often associated with hypoxemia due to shunt?
a. diffuse wheezing
b. “white” chest radiograph
c. stridor
d. loud P2

ANS: B
Shunt usually presents with a “white” chest radiograph.

DIF: Recall REF: p. 992 OBJ: 2

8. A patient with interstitial lung disease who presents with hypoxemia due to diffusion defect
would have which of the following clinical signs?
1. fine bibasilar crackles
2. clubbing of the finger nail beds
3. jugular venous distention
4. increased P2
a. 1 and 2
b. 1, 3, and 4
c. 3 and 4
d. 1, 2, 3, and 4

ANS: D
Patients may have clubbing of the nail beds. Rheumatologic manifestations may be present if the
underlying cause is a connective tissue disorder. Joint abnormalities, Reynaud disease, and
telangiectasia (a vascular lesion formed by dilatation of a group of small blood vessels) may be
observed. The pallor of anemia can be a clue to poor gas exchange, although chronic hypoxemia
may lead to polycythemia and possibly cyanosis. Pulmonary hypertension may present with
signs of right heart failure such as edema, jugular vein distension, and a louder pulmonary
component of the second heart sound.

DIF: Recall REF: p. 992 OBJ: 2

9. What type of disease is associated with perfusion/diffusion impairment?


a. liver disease
b. renal disease
c. neuromuscular disease
d. vascular disease

ANS: D
Perfusion/diffusion impairment is a rare cause of hypoxemia found in individuals with liver
disease complicated by the hepatopulmonary syndrome.

DIF: Recall REF: p. 992 OBJ: 3

10. What is the most common cause of low mixed venous oxygen?

Copyright © 2013, 2009, 2003, 1999, 1995, 1990, 1982, 1977, 1973, 1969 by Mosby, an imprint of Elsevier Inc.
Test bank 41-4

a. liver disease
b. cardiac disease
c. neuromuscular disease
d. vascular disease

ANS: B
Congestive heart failure with low cardiac output is the most common cause of low mixed venous
oxygen, due to increased peripheral extraction of oxygen.

DIF: Recall REF: p. 993 OBJ: 3

11. What is the normal P(Aa)O2 range while breathing room air?
a. 25 mm Hg to 50 mm Hg
b. 10 mm Hg to 25 mm Hg
c. greater than 25 mm Hg
d. less than 10 mm Hg

ANS: B
The P(A-a)O2 ranges from 10 mm Hg in young patients to approximately 25 mm Hg in the
elderly while breathing room air.

DIF: Recall REF: p. 993 OBJ: 3

12. What happens to the P(Aa)O2 with mismatch and shunt?


a. It increases with mismatch and decreases with shunt.
b. It decreases with both mismatch and shunt.
c. It increases with both mismatch and shunt.
d. It does not change.

ANS: C
A mismatch and shunt both result in elevated P(Aa)O2 levels.

DIF: Recall REF: p. 993 OBJ: 3

13. What is the optimal treatment of intrapulmonary shunt?


a. increase the FIO2
b. decrease the FIO2
c. surgery
d. alveolar recruitment

ANS: D
Treatment of intrapulmonary shunt must be directed toward opening collapsed alveoli or clearing
fluid or exudative material before oxygen can be beneficial at below toxic levels.

Copyright © 2013, 2009, 2003, 1999, 1995, 1990, 1982, 1977, 1973, 1969 by Mosby, an imprint of Elsevier Inc.
Test bank 41-5

DIF: Recall REF: p. 993 OBJ: 3

14. A patient with an opiate drug overdose is unconscious and exhibits the following blood gas
results breathing room air: pH = 7.19; PCO2 = 89; HCO3 = 27; PO2 = 48. Which of the following
best describes this patient's condition?
a. chronic hypoxemic respiratory failure
b. chronic hypercapnic respiratory failure
c. acute hypoxemic respiratory failure
d. acute hypercapnic respiratory failure

ANS: D
Hypercapnic respiratory failure (“pump failure,” “ventilatory failure”) is characterized by an
elevated PaCO2, creating an uncompensated respiratory acidosis (whether acute or acute-on-
chronic).

DIF: Application REF: p. 993 OBJ: 3

15. All of the following would tend to cause hypercapnic respiratory failure except:
a. smoke inhalation
b. opiate drug overdose
c. chronic obstructive pulmonary disease
d. hypothyroidism

ANS: A
This ventilatory drive can be diminished by various factors such as drugs (overdose/sedation),
brainstem lesions, diseases of the central nervous system such as multiple sclerosis or
Parkinson’s disease, hypothyroidism, morbid obesity (e.g., obesity-hypoventilation), and sleep
apnea.

DIF: Recall REF: p. 995 OBJ: 3

16. Which of the following are associated with hypercapnic respiratory failure due to decreased
ventilatory drive?
1. brainstem lesions
2. encephalitis
3. hypothyroidism
4. asthma
a. 1, 2, and 3
b. 2 and 4
c. 3 and 4
d. 1, 2, 3, and 4

ANS: A

Copyright © 2013, 2009, 2003, 1999, 1995, 1990, 1982, 1977, 1973, 1969 by Mosby, an imprint of Elsevier Inc.
Test bank 41-6

This ventilatory drive can be diminished by various factors such as drugs (overdose/sedation),
brainstem lesions, diseases of the central nervous system such as multiple sclerosis or
Parkinson’s disease, hypothyroidism, morbid obesity (e.g., obesity-hypoventilation), and sleep
apnea.

DIF: Recall REF: p. 995 OBJ: 3

17. All of the following are associated with hypercapnic respiratory failure due to respiratory muscle
weakness or fatigue except:
a. hyperthyroidism
b. myasthenia gravis
c. amyotrophic lateral sclerosis
d. Guillain-Barré syndrome

ANS: A
Examples include spinal trauma, motor neuron disease where lesions of the anterior horn cells
may gradually lead to progressive ventilatory failure (such as in amyotrophic lateral sclerosis, or
poliomyelitis), motor nerve disorders (including Guillain-Barré syndrome and Charcot-Marie-
Tooth disease), disorders of the neuromuscular junction (such myasthenia gravis and botulism),
and muscular diseases (including muscular dystrophy, myositis, critical care myopathy, and
metabolic disorders).

DIF: Recall REF: p. 995 OBJ: 3

18. Which of the following is a feature of Guillain-Barré?


a. ascending muscle weakness
b. descending muscle weakness
c. limited to lower extremities
d. limited to trunk

ANS: A
Guillain-Barré syndrome can commonly show up with lower extremity weakness progressing to
the respiratory muscles in one third of patients.

DIF: Recall REF: p. 995 OBJ: 3

19. All of the following are associated with hypercapnic respiratory failure due to increased work of
breathing except:
a. asthma
b. myasthenia gravis
c. obesity
d. kyphoscoliosis

ANS: B

Copyright © 2013, 2009, 2003, 1999, 1995, 1990, 1982, 1977, 1973, 1969 by Mosby, an imprint of Elsevier Inc.
Test bank 41-7

Most commonly, this situation occurs when increased dead space accompanies COPD or
elevated airway resistance accompanies asthma. Both of these obstructive airway diseases may
raise respiratory work requirements excessively due to the presence of intrinsic positive end-
expiratory pressure. Increased workload can also result from thoracic abnormalities such as
pneumothorax, rib fractures, pleural effusions, and other conditions creating a restrictive burden
on the lungs. Finally, requirements for increased minute ventilation can arise when increased
CO2 production accompanies hypermetabolic states, such as in extensive burns.

DIF: Recall REF: p. 996 OBJ: 3

20. Which of the following information best helps to distinguish chronic hypercapnic respiratory
failure from acute hypercapnic respiratory failure?
a. long-standing dyspnea that worsens on exertion
b. forced expiratory volume in 1 second-to-forced vital capacity ratio (FEV1/FVC) of
less than 75% predicted
c. kidneys retaining bicarbonate to elevate the blood pH
d. physical signs of hypoxemia, such as cyanosis and clubbing

ANS: C
Most commonly, chronic hypercapnic respiratory failure accompanying COPD or obesity
hypoventilation syndrome would elicit a renal response by which the kidneys retain bicarbonate
to elevate the blood pH.

DIF: Application REF: p. 997 OBJ: 4

21. Which of the following is false about the “acute-on-chronic” form of respiratory failure?
a. It usually involves patients with hypoxemic respiratory failure.
b. It is most common in patients with chronic airway obstruction.
c. Bacterial or viral infections are common precipitating factors.
d. Mortality is associated with severity of acidosis.

ANS: A
Patients with chronic hypercapnic respiratory failure (chronic ventilatory failure) are at
significant risk for this, as indicated by the fact that COPD is now the fourth leading cause of
death in the United States. Acute-on-chronic respiratory failure can also be the presenting
manifestation of neuromuscular disease in the setting of a concurrent pulmonary infection. Most
common precipitating factors include bacterial or viral infections, congestive heart failure,
pulmonary embolus, chest wall dysfunction, and medical noncompliance.

DIF: Recall REF: p. 997 OBJ: 4

22. Which of the following is the cardinal sign of increased work of breathing?
a. hyperventilation
b. retractions
c. bradycardia
d. tachypnea

Copyright © 2013, 2009, 2003, 1999, 1995, 1990, 1982, 1977, 1973, 1969 by Mosby, an imprint of Elsevier Inc.
Test bank 41-8

ANS: D
Tachypnea is the cardinal sign of increased work of breathing.

DIF: Recall REF: p. 997 OBJ: 5

23. In patients suffering from acute respiratory acidosis, below what pH level are intubation and
ventilatory support generally considered?
a. 7.2
b. 7.3
c. 7.1
d. 7.0

ANS: A
See Table 41-3.

DIF: Recall REF: p. 998 OBJ: 6

24. Which of the following patients has the most serious problem with the adequacy of oxygenation?
Patient FIO2 PaO2
a.
b.
c.
d.

ANS: C
See Table 41-3.

DIF: Analysis REF: p. 998 OBJ: 6

25. A need for some form of ventilatory support is usually indicated when an adult’s rate of
breathing rises above what level?
a. 35/min
b. 30/min
c. 25/min
d. 20/min

ANS: A
See Table 41-3.

DIF: Recall REF: p. 998 OBJ: 6

26. Which of the following measures is/are useful indicators in assessing the adequacy of a patient’s
oxygenation?
1. PaO2–PaO2
2. PaO2-to-FIO2 ratio

Copyright © 2013, 2009, 2003, 1999, 1995, 1990, 1982, 1977, 1973, 1969 by Mosby, an imprint of Elsevier Inc.
Test bank 41-9

3. VD/VT
4. pulmonary shunt ( s/ t)
a. 1 and 2
b. 1 and 3
c. 2 and 3
d. 1, 2, and 3

ANS: A
See Table 41-3.

DIF: Recall REF: p. 998 OBJ: 6

27. Which of the following measures taken on adult patients indicate unacceptably high ventilatory
demands or work of breathing?
a. VE of 17 L/min
b. breathing rate of 22/min
c. VD/VT of 0.45
d. MIP of –40 cm H2O

ANS: A
See Table 41-3.

DIF: Application REF: p. 998 OBJ: 6

28. Ventilatory support may be indicated when the VC falls below what level?
a. 45 ml/kg
b. 65 ml/kg
c. 10 ml/kg
d. 30 ml/kg

ANS: C
See Table 41-3.

DIF: Recall REF: p. 998 OBJ: 6

29. What is the normal range of maximum inspiratory pressure, or MIP (also called negative
inspiratory force, or NIF), generated by adults?
a. –80 to –100 cm H2O
b. –50 to –80 cm H2O
c. –30 to –50 cm H2O
d. –20 to –30 cm H2O

ANS: A
See Table 41-3.

DIF: Recall REF: p. 998 OBJ: 6

Copyright © 2013, 2009, 2003, 1999, 1995, 1990, 1982, 1977, 1973, 1969 by Mosby, an imprint of Elsevier Inc.
Test bank 41-10

30. Which of the following MIP measures taken on an adult patient indicates inadequate respiratory
muscle strength?
a. –90 cm H2O
b. –70 cm H2O
c. –40 cm H2O
d. –15 cm H2O

ANS: D
See Table 41-3.

DIF: Recall REF: p. 998 OBJ: 6

31. Common bedside measures used to assess the adequacy of lung expansion include all of the
following except:
a. VC
b. respiratory rate
c. VT
d. VD/VT

ANS: D
See Table 41-3.

DIF: Recall REF: p. 998 OBJ: 6

32. Inadequate respiratory muscle strength is likely when a patient’s MVV is which of the
following?
a. <2 times the resting VE
b. >3 times the resting VE
c. <200 L/min
d. >120 L/min

ANS: A
See Table 41-3.

DIF: Recall REF: p. 998 OBJ: 6

33. You determine that an acutely ill patient can generate an MIP of –18 cm H2O. Based on this
information, what might you conclude?
a. The patient has inadequate respiratory muscle strength.
b. The patient has inadequate alveolar ventilation.
c. The patient has an excessive work of breathing.
d. The patient has an unstable or irregular ventilatory drive.

ANS: A
See Table 41-3.

Copyright © 2013, 2009, 2003, 1999, 1995, 1990, 1982, 1977, 1973, 1969 by Mosby, an imprint of Elsevier Inc.
Test bank 41-11

DIF: Application REF: p. 998 OBJ: 6

34. Which of the following indicate severely impaired oxygenation requiring high FIO2s and positive
end-expiratory pressure?
1. PaO2–PaO2 greater than 350 mm Hg on 100% O2
2. VC less than 10 ml/kg
3. PaO2/FIO2 less than 200
a. 1 and 2
b. 1 and 3
c. 2 and 3
d. 1, 3, and 3

ANS: B
See Table 41-3.

DIF: Recall REF: p. 998 OBJ: 7

35. Breathing 100% O2, a patient has a PaO2–PaO2 of 60 mm Hg. Based on this information, what
might you conclude?
a. The patient has severe hypoxemia.
b. The patient has an excessive work of breathing.
c. The patient has acceptable oxygenation.
d. The patient has inadequate ventilation.

ANS: C
See Table 41-3.

DIF: Recall REF: p. 998 OBJ: 7

36. What is the normal range for PaO2/FIO2?


a. 350 to 450
b. 250 to 350
c. 150 to 250
d. 75 to 150

ANS: A
See Table 41-3.

DIF: Recall REF: p. 998 OBJ: 7

37. Which of the following measures should be used in assessing the adequacy of a patient’s alveolar
ventilation?

1. PaO2
2. arterial pH

Copyright © 2013, 2009, 2003, 1999, 1995, 1990, 1982, 1977, 1973, 1969 by Mosby, an imprint of Elsevier Inc.
Test bank 41-12

3. PaCO2
a. 1 and 2
b. 1 and 3
c. 2 and 3
d. 1, 2, and 3

ANS: C
See Table 41-3.

DIF: Recall REF: p. 998 OBJ: 7

38. A patient with a 10-year history of chronic bronchitis and an acute viral pneumonia exhibits the
following blood gas results breathing room air: pH = 7.22; PCO2 = 67; HCO3 = 26; PO2 = 60.
Which of the following best describes this patient’s condition?
a. chronic hypoxemic respiratory failure
b. acute hypercapnic respiratory failure
c. chronic hypercapnic respiratory failure
d. acute hypoxemic respiratory failure

ANS: B
Assessment of the pH allows a determination of whether the problem is acute or chronic.

DIF: Application REF: p. 998 OBJ: 7

39. Because an elevated PaCO2 increases ventilatory drive in normal subjects, the clinical presence
of hypercapnia indicates which of the following?
1. inability of the stimulus to get to the muscles
2. weak or missing central nervous system response to the elevated PCO2
3. pulmonary muscle fatigue
a. 1 and 2
b. 1 and 3
c. 2 and 3
d. 1, 2, and 3

ANS: D
Because an elevated PaCO2 increases ventilatory drive in healthy subjects, the very existence of
hypoventilation suggests other problems with the respiratory apparatus. Specifically, the
presence of acute respiratory acidosis indicates one of three major problems: (1) the respiratory
center is not responding normally to the elevated PaCO2, (2) the respiratory center is responding
normally, but the signal is not getting through to the respiratory muscles, or (3) despite normal
neurologic response mechanisms, the lungs and chest bellows are simply incapable of providing
adequate ventilation due to parenchymal lung disease or muscular weakness.

DIF: Application REF: p. 999 OBJ: 7

40. Which of the following indicators are useful in assessing respiratory muscle strength?

Copyright © 2013, 2009, 2003, 1999, 1995, 1990, 1982, 1977, 1973, 1969 by Mosby, an imprint of Elsevier Inc.
Test bank 41-13

1. maximum voluntary ventilation (MVV)


2. forced vital capacity (FVC)
3. dead spacetotidal volume ratio (VD/VT)
3. maximum inspiratory pressure (MIP)
a. 1 and3
b. 2 and 4
c. 3 and 4
d. 1, 2, and4

ANS: D
The most commonly used tests to assess respiratory muscle strength at the bedside are MIP and
maximum expiratory pressure (MEP), FVC, and MVV.

DIF: Recall REF: p. 999 OBJ: 6

41. A reversible impairment in the response of an overloaded muscle to neural stimulation best
describes which of the following?
a. central respiratory muscle fatigue
b. transmission respiratory muscle fatigue
c. contractile respiratory muscle fatigue
d. chronic respiratory muscle fatigue

ANS: C
Contractile respiratory muscle fatigue is a reversible impairment in the contractile response to a
neural impulse in an overloaded muscle.

DIF: Recall REF: p. 999 OBJ: 6

42. When is respiratory muscle fatigue likely to occur?


a. when VE exceeds 20% of the maximum voluntary ventilation (MVV)
b. when VE exceeds 40% of the MVV
c. when VE exceeds 60% of the MVV
d. when VE exceeds 80% of the MVV

ANS: C
Comparing the spontaneous minute ventilation with MVV is a helpful index as fatigue and
failure are both likely to occur if the minute ventilation exceeds 60% of MVV.

DIF: Recall REF: p. 1000 OBJ: 6

43. In intubated patients, what do sources of increased imposed work of breathing include?
1. endotracheal tube
2. ventilator circuit
3. auto-PEEP
a. 1 and 2
b. 1 and 3

Copyright © 2013, 2009, 2003, 1999, 1995, 1990, 1982, 1977, 1973, 1969 by Mosby, an imprint of Elsevier Inc.
Test bank 41-14

c. 2 and 3
d. 1, 2, and 3

ANS: D
In intubated patients, sources of imposed work of breathing include the endotracheal tube,
ventilator circuit, and auto-PEEP due to dynamic hyperinflation with airflow obstruction, as is
commonly seen in the patient with COPD.

DIF: Recall REF: p. 1000 OBJ: 7

44. A patient develops acute hypercapnic respiratory failure due to muscle fatigue. Which of the
following modes of ventilatory support would you consider for this patient?
1. assist-control ventilation with adequate backup
2. continuous positive airway pressure
3. synchronized intermittent mandatory ventilation with adequate backup rate
4. bilevel pressure support by mask
a. 2 and 4
b. 3 and 4
c. 1, 2, and 3
d. 1, 3, and 4

ANS: D
Noninvasive positive-pressure ventilation can improve hypoxemia and hypercarbia by several
mechanisms including but not limited to (1) compensating for the inspiratory threshold load
imposed by intrinsic positive end-expiration pressure, (2) supplementing a reduced tidal volume,
(3) partial or complete unloading of the respiratory muscles, (4) reducing venous return and left
ventricular afterload, and (5) alveolar recruitment.

DIF: Application REF: p. 1000 OBJ: 7

45. Which of the following modes of ventilatory support would you recommend for a hypoxemic
patient with congestive heart failure?
a. continuous positive airway pressure (CPAP)
b. intermittent mandatory ventilation (IMV)
c. inverse-ratio pressure control ventilation (PCV)
d. high-level pressure support ventilation (PSV)

ANS: A
In a systematic review of randomized trials, noninvasive positive-pressure ventilation was found
to reduce intubation rates and mortality in patients with acute cardiogenic pulmonary edema.
Overall, the level of evidence was noted to be similar for CPAP without significant advantages
of bilevel positive-pressure ventilation over CPAP.

DIF: Analysis REF: p. 1000 OBJ: 7

Copyright © 2013, 2009, 2003, 1999, 1995, 1990, 1982, 1977, 1973, 1969 by Mosby, an imprint of Elsevier Inc.
Test bank 41-15

46. Which of the following modes of ventilatory support would you recommend for a severely
hypoxemic patient with acute lung injury or acute respiratory distress syndrome (ARDS)?
a. continuous positive airway pressure
b. high VT volume-cycled ventilation
c. pressure-controlled ventilation
d. bilevel pressure support by mask

ANS: C
Volume-cycled ventilation in patients with ARDS frequently leads to high peak airway and
plateau pressures.

DIF: Analysis REF: p. 1002 OBJ: 7

47. A patient who just suffered severe closed-head injury and has a high intracranial pressure (ICP)
is about to be placed on ventilatory support. Which of the following strategies could help to
lower the ICP?
a. Maintain a PaCO2 from 25 to 30 mm Hg (deliberate hyperventilation).
b. Allow as much spontaneous breathing as possible (SIMV).
c. Maintain a high mean pressure using PEEP levels of 10 to 15 cm H2O.
d. Maintain a PaCO2 of 50 to 60 mm Hg (deliberate hypoventilation).

ANS: A
Hyperventilation applied acutely and for short periods of time may be used to reduce ICP. The
goal is to lower the PaCO2 to between 25 to 30 mm Hg, which causes alkalosis, which in
combination with hypocapnia helps reduce cerebral blood flow until the ICP can be controlled by
other measures.

DIF: Application REF: p. 1002 OBJ: 7

48. Which of the following patients are at greatest risk for developing auto-PEEP during mechanical
ventilation?
a. those with acute lung injury
b. those with COPD
c. those with congestive heart failure
d. those with bilateral pneumonia

ANS: B
These patients frequently have problems with elevated airway pressure or dynamic
hyperinflation (auto-PEEP), which can cause barotrauma and increased dyssynchrony between
the patient and the ventilator.

DIF: Application REF: p. 1003 OBJ: 7

49. What are some causes of dynamic hyperinflation?


1. increased expiratory time
2. increased airway resistance

Copyright © 2013, 2009, 2003, 1999, 1995, 1990, 1982, 1977, 1973, 1969 by Mosby, an imprint of Elsevier Inc.
Test bank 41-16

3. decreased expiratory flow rate


a. 1 and 2
b. 1 and 3
c. 2 and 3
d. 1, 2, and 3

ANS: C
In such patients, lower tidal volumes (6 to 8 ml/kg), moderate respiratory rates, and high
inspiratory flow rates (70 to 100 L/min) are recommended to avoid dynamic hyperinflation.

DIF: Recall REF: p. 1003 OBJ: 7

50. Strategies to reduce auto-PEEP in mechanically ventilated patients with obstructive lung disease
include all of the following except which one?
a. Use high inspiratory flows (60 to 100 L/min).
b. Apply extrinsic PEEP.
c. Use low VT values (8 to 10 ml/kg).
d. Use high respiratory rates (greater than 25/min).

ANS: D
In such patients, lower tidal volumes (6 to 8 ml/kg), moderate respiratory rates, and high
inspiratory flow rates (70 to 100 L/min) are recommended to avoid dynamic hyperinflation.

DIF: Recall REF: p. 1003 OBJ: 7

51. Which of the following is the normal alveolar-to-arterial difference for a 56 year old female in
the emergency department?
a. 12 mm Hg
b. 14 mm Hg
c. 16 mm Hg
d. 18 mm Hg

ANS: D
[P(A-a)O2] = (age/4) + 4
[P(A-a)O2] = (56/4) + 4
[P(A-a)O2] = 18 mm Hg

DIF: Application REF: p. 1002 OBJ: 7

52. Which of the following are causes of hypoxemia?


1. ventilation/perfusion (V/Q) mismatch
2. alveolar hypoventilation
3. diffusion impairment
4. increased inspired O2
a. 1 and 2
b. 2 and 3

Copyright © 2013, 2009, 2003, 1999, 1995, 1990, 1982, 1977, 1973, 1969 by Mosby, an imprint of Elsevier Inc.
Test bank 41-17

c. 1, 2, and 3
d. 1, 2, 3, and 4

ANS: C
The following are causes of hypoxemia: V/Q mismatch (most common cause), shunt, alveolar
hypoventilation, diffusion impairment, perfusion/diffusion impairment (rare), decreased inspired
oxygen, and venous admixture.

DIF: Recall REF: p. 990 OBJ: 2

53. The respiratory therapist in the ICU is called to assess a patient with ARDS. The patient is SOB.
The x-ray shows “white” chest radiograph and the PAO2 is 60 torr on an FIO2 of 100%. Which of
the following is indicated?
a. shunting
b. alveolar hyperventilation
c. decreased CO2
d. perfusion impairment

ANS: A
Shunt is indicated by the following: shunt usually presents with a white radiograph. ARDS is a
classic example of shunting. Shunt also does not respond to high level of supplemental oxygen.

DIF: Analysis REF: p. 993 OBJ: 3

54. Mr. Adam is in the ICU on an FIO2 of 100%. An arterial blood gas reveals the following
information: pH of 7.18, PaCO2 of 59, PaO2 of 65 , HCO3 of 24. What action would you
recommend?
a. Provide ventilatory support.
b. Put patient on steroids.
c. Give patient Chest PT.
d. Put patient on CPAP.

ANS: A
The patient is in hypoxic (Type I) and hypercapnic (Type2) acute respiratory failure. Providing
full mechanical ventilatory support will provide the ventilator support needed to normalize pH
and improve oxygenation.

DIF: Analysis REF: p. 997 OBJ: 7

Copyright © 2013, 2009, 2003, 1999, 1995, 1990, 1982, 1977, 1973, 1969 by Mosby, an imprint of Elsevier Inc.
Test bank 41-18

Kacmarek: Egan's Fundamentals of Respiratory Care, 10th Edition

Chapter 42: Mechanical Ventilators

Test Bank

MULTIPLE CHOICE

1. Which of the following major categories of ventilator function are useful in classifying
ventilators?
1. control scheme
2. power conversion
3. ventilator output
a. 1 and 2
b. 1 and 3
c. 2 and 3
d. 1, 2, and 3

ANS: D
To understand mechanical ventilators, we must first understand their four basic functions:
• input power
• power transmission and conversion
• control system
• output (pressure, volume, and flow waveforms)

DIF: Recall REF: p. 1007 OBJ: 1

2. A ventilator can derive its input power from which of the following sources?
1. alternating current (AC) electricity
2. battery
3. pneumatic
a. 1 and 2
b. 1 and 3
c. 2 and 3
d. 1, 2, and 3

ANS: D
The power source for a ventilator is either electrical energy (Energy = Volts  Amperes  Time)
or compressed gas (Energy = Pressure  Volume).

DIF: Recall REF: p. 1007 OBJ: 1

Copyright © 2013, 2009, 2003, 1999, 1995, 1990, 1982, 1977, 1973, 1969 by Mosby, an imprint of Elsevier Inc.
Test bank 41-19

3. For which of the following uses might you consider the use of a purely pneumatically powered
ventilator?
1. as a backup to electrically powered ventilators
2. when electrical device cannot be used (e.g., magnetic resonance imaging)
3. during certain types of patient transport
a. 1 and 2
b. 1 and 3
c. 2 and 3
d. 1, 2, and 3

ANS: D
These devices are ideal in situations where electrical power is unavailable (e.g., during certain
types of patient transport) or as a backup to electrically powered ventilators in case of power
failures. They are also particularly useful where electrical power is undesirable, as near magnetic
resonance imaging equipment.

DIF: Application REF: p. 1007 OBJ: 2

4. Primary drive mechanisms used by modern ventilators include which of the following?
1. compressed gas or reducing valve
2. hydraulic or fluidic compressor
3. electrical motor or compressor
a. 1 and 2
b. 1 and 3
c. 2 and 3
d. 1, 2, and 3

ANS: B
Drive mechanisms can be either (1) a direct application of compressed gas via a pressure
reducing valve or (2) an indirect application via an electric motor or compressor.

DIF: Recall REF: p. 1007 OBJ: 2

5. Types of output control valves used in modern ventilators include all of the following except:
a. pneumatic diaphragm valve
b. proportional valve
c. electromagnetic poppet or plunger
d. linear screw valve

ANS: D
Commonly used output control valves include the pneumatic diaphragm, electromagnetic
poppet/plunger valve, and the proportional valve. Descriptions of these devices can be found in
respiratory care equipment textbooks.

DIF: Recall REF: p. 1007 OBJ: 2

Copyright © 2013, 2009, 2003, 1999, 1995, 1990, 1982, 1977, 1973, 1969 by Mosby, an imprint of Elsevier Inc.
Test bank 41-20

6. Which of the following equations best describes the pressure (P) necessary to drive gas into the
airway and inflate the lungs?
a. P = (Elastance  Volume) + (Resistance  Flow)
b. P = (Elastance – Volume) + (Resistance ÷ Flow)
c. P = (Volume + Compliance) + (Resistance ÷ Flow)
d. P = (Volume ÷ Compliance) – (Resistance  Flow)

ANS: A
Pvent + Pmus = (E  V) + (R  V).

DIF: Application REF: p. 1008 OBJ: 2

7. How would a ventilatory support device that uses pressure regulators, needle valves, and balloon
valves to regulate most or all of the parameters of ventilation be classified?
a. mechanically controlled
b. electronically controlled
c. fluidically controlled
d. pneumatically controlled

ANS: D
Pneumatic control is provided using gas-powered pressure regulators, needle valves, jet
entrainment devices, and balloon valves.

DIF: Recall REF: p. 1011 OBJ: 2

8. Which of the following types of ventilators would you select for use during an MRI procedure?
1. electronically controlled
2. pneumatically controlled
3. fluidically controlled
4. electrically controlled
a. 1 and 4
b. 2 and 3
c. 1, 2, and 4
d. 1, 2, 3, and 4

ANS: B
Some transport ventilators use pneumatic control systems. The Ohmeda Logic-07 is an example.
Fluidic control mechanisms have no moving parts. In addition, fluidic circuits are immune to
failure from surrounding electromagnetic interference, as can occur around MRI equipment.

DIF: Application REF: p. 1011 OBJ: 2

9. Which of the following ventilators is controlled by fluidic logic systems?


a. Siemens 300
b. Sechrist IV-100B

Copyright © 2013, 2009, 2003, 1999, 1995, 1990, 1982, 1977, 1973, 1969 by Mosby, an imprint of Elsevier Inc.
Test bank 41-21

c. Bird 8400ST
d. Bear 1000

ANS: B
Fluidic logic-controlled ventilators, such as the Bio-Med MVP-10 and Sechrist IV-100B, also
use pressurized gas to regulate the parameters of ventilation.

DIF: Recall REF: p. 1011 OBJ: 2

10. According to the equation of motion of the respiratory system, a ventilator can control all of the
following variables except:
a. volume
b. resistance
c. pressure
d. flow

ANS: B
There are only three variables in the equation of motion that a ventilator can control: pressure,
volume, and flow.

DIF: Recall REF: p. 1017 OBJ: 2

11. If the pressure waveform of a ventilator remains the same when a patient's lung mechanics
change, then what is the ventilator?
a. volume controller
b. pressure controller
c. time controller
d. flow controller

ANS: B
If the ventilator controls pressure, the pressure waveform will remain consistent but volume and
flow will vary with changes in respiratory system mechanics.

DIF: Application REF: p. 1019 OBJ: 2

12. Which of the following are characteristic of a ventilator that functions as a true volume
controller?
1. Its pressure waveform changes with changes in lung mechanics.
2. It measures and uses volume to control the volume waveform.
3. Its volume waveform stays constant with changes in lung mechanics.
a. 1 and 2
b. 1 and 3
c. 2 and 3
d. 1, 2, and 3

ANS: D

Copyright © 2013, 2009, 2003, 1999, 1995, 1990, 1982, 1977, 1973, 1969 by Mosby, an imprint of Elsevier Inc.
Test bank 41-22

If the ventilator controls volume, the volume and flow waveforms will remain consistent, but
pressure will vary with changes in respiratory mechanics. To qualify as a true volume controller,
a ventilator must measure volume and use this signal to control the volume waveform.

DIF: Recall REF: p. 1019 OBJ: 2

13. During volume control ventilation, the clinician has control over which of the following?
1. pressure waveform
2. volume waveform
3. flow waveform
a. 1 or 2
b. 2 or 3
c. 2
d. 1, 2, and 3

ANS: B
Volume can be controlled directly by the displacement of a device such as a piston or bellows.
Volume can be controlled indirectly by controlling flow.

DIF: Recall REF: p. 1019 OBJ: 2

14. Which of the following is true of the relationship between flow and volume?
1. Volume is the integral of flow.
2. Volume is the derivative of flow.
3. Flow is the derivative of volume.
a. 1 and 2
b. 1 and 3
c. 2 and 3
d. 1, 2, and 3

ANS: B
This follows from the fact that volume and flow are inverse functions of time (i.e., volume is the
integral of flow and flow is the derivative of volume).

DIF: Application REF: p. 1019 OBJ: 2

15. A ventilator's pressure waveform changes when a patient's lung mechanics change, but its
volume waveform remains the same. The device does not directly control the delivered volume.
What is this ventilator?
a. volume controller
b. pressure controller
c. time controller
d. flow controller

ANS: D

Copyright © 2013, 2009, 2003, 1999, 1995, 1990, 1982, 1977, 1973, 1969 by Mosby, an imprint of Elsevier Inc.
Test bank 41-23

If the ventilator controls flow, the flow and volume waveforms will remain consistent, but
pressure will vary with changes in respiratory mechanics. Flow can be controlled directly using
something as simple as a flow meter or as complex as a proportional solenoid valve. Flow can be
controlled indirectly by controlling volume.

DIF: Recall REF: p. 1019 OBJ: 2

16. The volume waveform generated by a ventilator remains the same against changing lung
mechanics. Which of the following parameters might this device be controlling?
1. volume
2. flow
3. pressure
a. 1 and 2
b. 1 and 3
c. 2 and 3
d. 1, 2, and 3

ANS: A
If the ventilator controls flow, the flow and volume waveforms will remain consistent, but
pressure will vary with changes in respiratory mechanics. Flow can be controlled directly using
something as simple as a flow meter or as complex as a proportional solenoid valve. Flow can be
controlled indirectly by controlling volume.

DIF: Application REF: p. 1019 OBJ: 2

17. Which of the following ventilators is the simplest example of a true flow controller?
a. Newport Wave
b. Bear 1
c. Emerson 3-PV
d. Sechrist

ANS: D
Infant ventilators, such as the Sechrist, are the simplest examples of flow controllers

DIF: Recall REF: p. 1019 OBJ: 2

18. A complete ventilatory cycle or breath consists of which of the following phases?

1. expiration
2. initiation of inspiration
3. inspiration
4. end of inspiration
a. 1 and 4
b. 2 and 3
c. 1, 2, and 4
d. 1, 2, 3, and 4

Copyright © 2013, 2009, 2003, 1999, 1995, 1990, 1982, 1977, 1973, 1969 by Mosby, an imprint of Elsevier Inc.
Test bank 41-24

ANS: D
A complete ventilatory cycle or breath consists of four phases: the initiation of inspiration,
inspiration itself, the end of inspiration, and expiration.

DIF: Recall REF: p. 1019 OBJ: 2

19. During mechanical ventilation, what variable causes a breath to begin?


a. limit
b. cycle
c. trigger
d. baseline

ANS: C
The variable causing a breath to begin is the trigger variable.

DIF: Recall REF: p. 1015 OBJ: 2

20. During mechanical ventilation, what variable causes a breath to end?


a. limit
b. cycle
c. trigger
d. baseline

ANS: B
The variable causing a breath to end is the cycle variable.

DIF: Recall REF: p. 1015 OBJ: 2

21. To describe what happens during the expiratory phase of mechanical ventilation, you must know
the value of which variable?
a. limit
b. cycle
c. trigger
d. baseline

ANS: D
To describe what happens during expiration, we must know what baseline variable is in effect.

DIF: Recall REF: p. 1017 OBJ: 2

22. If a ventilator, not the patient, initiates a breath, what is the trigger variable?
a. time
b. pressure
c. flow
d. volume

Copyright © 2013, 2009, 2003, 1999, 1995, 1990, 1982, 1977, 1973, 1969 by Mosby, an imprint of Elsevier Inc.
Test bank 41-25

ANS: A
If the machine initiates the breath, the trigger variable is time.

DIF: Recall REF: p. 1013 OBJ: 2

23. If a patient initiates a ventilator breath, the trigger variable could be all of the following except:
a. pressure
b. flow
c. time
d. volume

ANS: C
If the patient initiates the breath, pressure, flow, or volume may serve as the trigger variable.

DIF: Recall REF: p. 1013 OBJ: 2

24. A patient receiving time-triggered continuous mechanical ventilation at a preset rate of 10/min
stops breathing. Which of the following will occur?
a. The high-pressure limit alarm will sound (if properly set).
b. The patient will continue to receive 10 breaths/min.
c. The low tidal volume (VT) alarm will sound (if properly set).
d. Ventilation will drop to zero and the apnea alarm will sound.

ANS: B
When triggering by time, a ventilator initiates a breath according to a predetermined time
interval, without regard to patient effort.

DIF: Application REF: p. 1013 OBJ: 2

25. Pure time-triggered ventilation is the same as what type of ventilation?


a. assist
b. intermittent mandatory ventilation
c. assist and control
d. proportional assist

ANS: B
Currently, time triggering is most commonly seen when using the IMV mode (intermittent
mandatory ventilation).

DIF: Recall REF: p. 1013 OBJ: 2

26. A volume-cycled ventilator has a rate knob for setting the controlled frequency of breathing. If
this control is set to 12/min, which of the following other settings will determine the inspiratory
and expiratory times?
1. FIO2

Copyright © 2013, 2009, 2003, 1999, 1995, 1990, 1982, 1977, 1973, 1969 by Mosby, an imprint of Elsevier Inc.
Test bank 41-26

2. flow
3. volume
a. 1 and 2
b. 1 and 3
c. 2 and 3
d. 1, 2, and 3

ANS: C
When a rate control is used, inspiratory and expiratory times will vary according to other control
settings, such as flow and volume.

DIF: Application REF: p. 1013 OBJ: 2

27. When you adjust the pressure drop necessary to trigger a breath on a ventilator, what are you
adjusting on the machine?
a. sensitivity
b. pressure limit
c. mode setting
d. positive end-expiratory pressure (PEEP) level

ANS: A
Pressure triggering occurs when a patient’s inspiratory effort causes a drop in pressure within the
breathing circuit. When this pressure drop reaches the pressure sensing mechanism, the ventilator
triggers on and begins gas delivery. On most ventilators, you can adjust the pressure drop needed
to trigger a breath. The trigger level is often called the sensitivity.

DIF: Recall REF: p. 1013 OBJ: 2

28. When using pressure as the trigger variable, where do you typically set the trigger level?
a. 0.5 to 1.5 cm H2O below the baseline expiratory pressure
b. 0.5 to 1.5 cm H2O above the baseline expiratory pressure
c. 2.0 to 3.5 cm H2O below the baseline expiratory pressure
d. 2.0 to 3.5 cm H2O above the baseline expiratory pressure

ANS: A
Typically, you set the trigger level 0.5 to 1.5 cm H2O below the baseline expiratory pressure.

DIF: Recall REF: p. 1014 OBJ: 2

29. Which of the following is false about the application of flow triggering on a mechanical
ventilator?
a. The ventilator measures both input and output flow.
b. Between patient breaths, input flow exceeds output flow.
c. A relative drop in output flow triggers the machine to turn on.
d. Gas flows continuously through the ventilator circuit.

Copyright © 2013, 2009, 2003, 1999, 1995, 1990, 1982, 1977, 1973, 1969 by Mosby, an imprint of Elsevier Inc.
Test bank 41-27

ANS: B
The ventilator measures the flow coming out of the main flow control valve and also the flow
through the exhalation valve. Between breaths, these two flows are equal (assuming there are no
leaks in the patient circuit). When the patient makes an inspiratory effort, the flow through the
exhalation valve falls below the flow from the output valve. The difference between these two
flows is the flow trigger variable.

DIF: Recall REF: p. 1014 OBJ: 2

30. A physician requests that you switch from pressure-triggering a patient to flow-triggering. Which
of the following new settings would be appropriate?
a. Base flow = 0 L/min; trigger at 2 L/min
b. Base flow = 10 L/min; trigger at –2 cm H2O
c. Base flow = 10 L/min; trigger at 2 L/min
d. Base flow = 0 L/min; trigger at 10 cm H2O

ANS: C
For example, if you set the base continuous flow at 10 L/min and the trigger at 2 L/min, the
ventilator will trigger when the output flow falls to 8 L/min or less.

DIF: Analysis REF: p. 1014 OBJ: 2

31. Compared to using pressure as the trigger variable, what is the major advantage of flow-
triggering?
a. decreased work of breathing
b. improved minute ventilation (VE)
c. decreased physiologic dead space
d. improved arterial oxygenation

ANS: A
When compared with pressure, using flow as the trigger variable decreases a patient’s work of
breathing.

DIF: Recall REF: p. 1014 OBJ: 2

32. What ventilatory variable reaches and maintains a preset level before inspiration ends?
a. limit
b. cycle
c. trigger
d. baseline

ANS: A
A limit variable is one that can reach and maintain a preset level before inspiration ends but does
not terminate inspiration.

DIF: Recall REF: p. 1015 OBJ: 2

Copyright © 2013, 2009, 2003, 1999, 1995, 1990, 1982, 1977, 1973, 1969 by Mosby, an imprint of Elsevier Inc.
Test bank 41-28

33. Which of the following parameters can serve as the cycle variable during ventilatory support?
1. volume
2. pressure
3. flow
4. time
a. 1 and 4
b. 2 and 3
c. 1, 2, and 4
d. 1, 2, 3, and 4

ANS: D
The cycle variable can be pressure, volume, flow, or time.

DIF: Application REF: p. 1015 OBJ: 2

34. A volume-cycled ventilator provides gas under positive pressure during inspiration until what
point?
a. A preselected volume of gas is received by the patient.
b. An adjustable, preselected airway pressure is reached.
c. The inspiratory time equals or exceeds the expiratory time.
d. A preselected volume of gas is expelled from the device.

ANS: D
When a ventilator is set to volume-cycle, it delivers flow until a preselected volume has been
expelled from the device.

DIF: Application REF: p. 1015 OBJ: 3

35. Flow serves as a limit variable whenever a ventilator controls what?


a. pressure
b. flow
c. time
d. volume

ANS: B
When a ventilator is set to flow cycle, it delivers flow until a preset level is met and then flow
stops and expiration begins.

DIF: Recall REF: p. 1016 OBJ: 3

36. You observe that a ventilator reaches a preset pressure early in inspiration but holds it for a
specific time, after which inspiration ends. What mode of ventilatory support is in force?
a. time cycled
b. pressure limited
c. pressure cycled

Copyright © 2013, 2009, 2003, 1999, 1995, 1990, 1982, 1977, 1973, 1969 by Mosby, an imprint of Elsevier Inc.
Test bank 41-29

d. volume limited

ANS: A
Time cycling occurs when the inspiratory time has elapsed.

DIF: Analysis REF: p. 1016 OBJ: 3

37. A time-cycled constant flow generator is set up with a flow of 35 L/min and an inspiratory time
of 1.7 seconds. What is the approximate VT?
a. 750 ml (0.75 L)
b. 1000 ml (1.00 L)
c. 1900 ml (1.90 L)
d. 1200 ml (1.20 L)

ANS: B
If it is used, it may be set directly, or it may occur indirectly if the set inspiratory time is longer
than the inspiratory flow time (determined by the set tidal volume and flow; time =
volume/flow).

DIF: Application REF: p. 1017 OBJ: 3

38. What is the name of a breath where a patient is able to change the inspiratory time?
a. patient cycled
b. patient triggered
c. machine triggered
d. machine cycled

ANS: A
For the breath to be patient cycled, the patient must be able to change the inspiratory time, such
as by making either inspiratory or expiratory efforts. If this is not possible, then the breath is, by
definition, machine cycled.

DIF: Recall REF: p. 1017 OBJ: 3

39. In which of the following modes inspiration ends when flow decays to some preset value?
a. intermittent mandatory ventilation
b. pressure support ventilation
c. continuous mandatory ventilation
d. airway pressure release ventilation

ANS: B
Another example of patient cycling is the pressure support mode. Here, inspiration ends when
flow decays to some preset value (i.e., flow cycling).

DIF: Recall REF: p. 1017 OBJ: 3

Copyright © 2013, 2009, 2003, 1999, 1995, 1990, 1982, 1977, 1973, 1969 by Mosby, an imprint of Elsevier Inc.
Test bank 41-30

40. What parameter serves as the baseline variable on all modern ventilators?
a. pressure
b. flow
c. time
d. volume

ANS: A
Although pressure, volume, or flow could serve as the baseline variable, pressure control is the
most practical and is implemented by all modern ventilators.

DIF: Recall REF: p. 1017 OBJ: 3

41. What is the default baseline value during mechanical ventilation?


a. positive end-expiratory pressure (PEEP)
b. zero end-expiratory pressure (ZEEP)
c. negative end-expiratory pressure (NEEP)
d. continuous positive airway pressure (CPAP)

ANS: B
ZEEP is the default baseline value during positive-pressure ventilation, meaning that it is
normally in effect unless purposely changed.

DIF: Recall REF: p. 1017 OBJ: 3

42. What is the application of pressure above atmospheric at the airway throughout expiration during
mechanical ventilation?
a. positive end-expiratory pressure (PEEP)
b. pressure support ventilation
c. continuous mandatory ventilation (CMV)
d. continuous positive airway pressure (CPAP)

ANS: A
PEEP is the application of pressure above atmospheric pressure at the airway throughout
expiration.

DIF: Recall REF: p. 1017 OBJ: 3

43. What is the primary physiological effect of positive end-expiratory pressure (PEEP)?
a. increase the functional residual capacity (FRC)
b. increase the inspiratory reserve volume (IRV)
c. decrease the compliance of the lung (CL)
d. increase the length of expiration

ANS: A
PEEP elevates a patient’s FRC and can help improve oxygenation by preventing collapse of
alveolar units that are made unstable by lack of surfactant or disease.

Copyright © 2013, 2009, 2003, 1999, 1995, 1990, 1982, 1977, 1973, 1969 by Mosby, an imprint of Elsevier Inc.
Test bank 41-31

DIF: Recall REF: p. 1017 OBJ: 3

44. During mechanical ventilation, a spontaneous breath is defined as one that:


a. initiated and terminated by the machine
b. begun by the patient and ended by the machine
c. initiated and terminated by the patient
d. begun by the machine and ended by the patient

ANS: C
A spontaneous breath is a breath for which the patient decides the start time and the tidal volume.
That is, the patient both triggers and cycles the breath.

DIF: Recall REF: p. 1019 OBJ: 3

45. During mechanical ventilation, a mandatory breath is defined as one that is:
a. initiated or terminated by the machine
b. initiated and terminated by the machine
c. initiated and terminated by the patient
d. begun according to a preset time interval

ANS: A
A mandatory breath is a breath for which the machine sets the start time and/or the tidal volume.
That is, the machine triggers and/or cycles the breath.

DIF: Recall REF: p. 1019 OBJ: 3

46. While observing a patient receiving ventilatory support, you notice that all delivered breaths are
initiated or terminated by the machine. Which of the following modes of ventilatory support is in
force?
a. intermittent mandatory ventilation
b. partial ventilatory support
c. continuous mandatory ventilation
d. continuous spontaneous ventilation

ANS: C
In continuous mandatory ventilation, all breaths are mandatory.

DIF: Application REF: p. 1021 OBJ: 3

47. While observing a patient receiving ventilatory support, you notice that some delivered breaths
are begun or ended by the machine, whereas others are begun and ended by the patient. Which of
the following modes of ventilatory support is in force?
a. intermittent mandatory ventilation (IMV)
b. pressure support ventilation
c. continuous mandatory ventilation (CMV)

Copyright © 2013, 2009, 2003, 1999, 1995, 1990, 1982, 1977, 1973, 1969 by Mosby, an imprint of Elsevier Inc.
Test bank 41-32

d. airway pressure release ventilation

ANS: A
In IMV, breaths can be either mandatory or spontaneous.

DIF: Application REF: p. 1021 OBJ: 3

48. A mode that allows spontaneously breathing patients to breathe at a positive-pressure level, but
drops briefly to a reduced pressure level for CO2 elimination during each breathing cycle is also
known as:
a. intermittent mandatory ventilation
b. airway pressure release ventilation
c. continuous mandatory ventilation (CMV)
d. continuous spontaneous ventilation

ANS: B
At this level of description, we can avoid the cumbersome verbal ad hoc definition for airway
pressure release ventilation such as “a mode that allows spontaneously breathing patients to
breathe at a positive-pressure level, but drops briefly to a reduced pressure level for CO 2
elimination during each breathing cycle.”

DIF: Recall REF: p. 1021 OBJ: 3

49. Which of the following ventilator control systems is NOT considered closed loop?
a. orientation based
b. servo
c. adaptive
d. optimal

ANS: A
The basic concept of closed-loop control has evolved into at least seven different ventilator
control systems.

DIF: Recall REF: p. 1021 OBJ: 3

50. Which of the closed-loop controllers is used by all ventilators?


a. setpoint
b. auto setpoint
c. adaptive
d. servo

ANS: A
Most ventilators use at least setpoint control.

DIF: Recall REF: p. 1023 OBJ: 3

Copyright © 2013, 2009, 2003, 1999, 1995, 1990, 1982, 1977, 1973, 1969 by Mosby, an imprint of Elsevier Inc.
Test bank 41-33

51. Which of the following modes is a good example of adaptive control?


a. intermittent mandatory ventilation
b. airway pressure release ventilation
c. continuous mandatory ventilation (CMV)
d. pressure-regulated volume control (PRVC)

ANS: D
One of the first examples of a mode using adaptive control was PRVC on the Siemens Servo 300
ventilator.

DIF: Recall REF: p. 1023 OBJ: 3

52. A ventilator that controls pressure and delivers a rectangular pressure waveform will also exhibit
what waveform?
a. exponential (rise) volume
b. rectangular flow
c. ascending ramp pressure
d. sinusoidal flow

ANS: A
See Figure 42-11.

DIF: Recall REF: p. 1022 OBJ: 3

53. A ventilator that controls flow and delivers a rectangular flow waveform will also exhibit what
waveform?
a. exponential (rise) volume
b. rectangular pressure
c. ascending ramp pressure
d. descending ramp volume

ANS: C
See Figure 42-11.

DIF: Recall REF: p. 1022 OBJ: 3

54. Peak airway pressure is highest with what waveform?


a. sinusoidal flow
b. rectangular flow
c. ascending ramp flow
d. descending ramp flow

ANS: C
See Figure 42-11.

DIF: Recall REF: p. 1022 OBJ: 3

Copyright © 2013, 2009, 2003, 1999, 1995, 1990, 1982, 1977, 1973, 1969 by Mosby, an imprint of Elsevier Inc.
Test bank 41-34

55. Mean airway pressure is lowest with what waveform?


a. sinusoidal flow
b. rectangular flow
c. ascending ramp flow
d. descending ramp flow

ANS: C
See Figure 42-11.

DIF: Recall REF: p. 1022 OBJ: 3

56. Mean airway pressure is highest with what waveform?


a. rectangular flow
b. rectangular pressure
c. ascending ramp flow
d. sinusoidal flow

ANS: B
See Figure 42-11.

DIF: Recall REF: p. 1022 OBJ: 3

57. During volume-targeted ventilation, which of the following settings determine the machine-
delivered minute volume?
1. volume
2. flow
3. rate
a. 1 and 2
b. 1 and 3
c. 2 and 3
d. 1, 2, and 3

ANS: B
See Table 42-1.

DIF: Application REF: p. 1013 OBJ: 3

58. During volume-targeted ventilation, which of the following settings determine the inspiratory
time?
1. volume
2. flow
3. rate
a. 1 and 2
b. 1 and 3
c. 2 and 3

Copyright © 2013, 2009, 2003, 1999, 1995, 1990, 1982, 1977, 1973, 1969 by Mosby, an imprint of Elsevier Inc.
Test bank 41-35

d. 1, 2, and 3

ANS: A
See Table 42-1.

DIF: Recall REF: p. 1013 OBJ: 3

59. During volume-targeted ventilation, which of the following settings determine the expiratory
time?
1. volume
2. flow
3. rate
a. 1 and 2
b. 1 and 3
c. 2 and 3
d. 1, 2, and 3

ANS: D
See Table 42-1.

DIF: Recall REF: p. 1013 OBJ: 3

60. During volume-targeted ventilation, which of the following settings determine the total cycle
time?
1. volume
2. flow
3. rate
a. 1 and 2
b. 2 and 3
c. 3 only
d. 1, 2, and 3

ANS: C
See Table 42-1.

DIF: Recall REF: p. 1013 OBJ: 3

61. During volume-targeted ventilation, which of the following settings determine I:E ratio?
1. volume
2. flow
3. rate
a. 1 and 2
b. 1 and 3
c. 2 and 3
d. 1, 2, and 3

Copyright © 2013, 2009, 2003, 1999, 1995, 1990, 1982, 1977, 1973, 1969 by Mosby, an imprint of Elsevier Inc.
Test bank 41-36

ANS: D
See Table 42-1.

DIF: Application REF: p. 1013 OBJ: 3

62. A patient receiving continuous mandatory ventilation in the control mode has an inspiratory time
of 1.5 seconds and an expiratory time of 2.5 seconds. What is the frequency of breathing?
a. 10/min
b. 12/min
c. 15/min
d. 18/min

ANS: C
See Table 42-1.

DIF: Application REF: p. 1013 OBJ: 3

63. A patient is receiving continuous mandatory ventilation in the control mode at a rate of 15/min.
The expiratory time is 2.9 seconds. What is the inspiratory time?
a. 1.1 seconds
b. 1.3 seconds
c. 1.5 seconds
d. 1.7 seconds

ANS: A
See Table 42-1.

DIF: Application REF: p. 1013 OBJ: 3

64. A patient is receiving continuous mandatory ventilation in the control mode at a rate of 10/min.
The inspiratory time control is set at 40%. What is the inspiratory time?
a. 1.60 seconds
b. 1.85 seconds
c. 2.40 seconds
d. 3.50 seconds

ANS: C
See Table 42-1.

DIF: Application REF: p. 1013 OBJ: 3

65. A patient is receiving continuous mandatory ventilation in the control mode at a rate of 12/min.
The inspiratory time control is set at 33%. What is the expiratory time?
a. 1.65 seconds
b. 2.45 seconds
c. 3.35 seconds

Copyright © 2013, 2009, 2003, 1999, 1995, 1990, 1982, 1977, 1973, 1969 by Mosby, an imprint of Elsevier Inc.
Test bank 41-37

d. 3.85 seconds

ANS: C
See Table 42-1.

DIF: Application REF: p. 1013 OBJ: 3

66. A patient is receiving continuous mandatory ventilation in the control mode at a rate of 15/min.
The inspiratory time is 0.8 second. What is the expiratory time?
a. 3.2 seconds
b. 2.8 seconds
c. 2.4 seconds
d. 4.2 seconds

ANS: A
See Table 42-1.

DIF: Application REF: p. 1013 OBJ: 3

67. A patient is receiving continuous mandatory ventilation in the control mode at a rate of 20/min.
The inspiratory time is 0.75 second. What is the percentage inspiratory time?
a. 20%
b. 25%
c. 30%
d. 33%

ANS: B
See Table 42-1.

DIF: Application REF: p. 1013 OBJ: 3

68. A patient is receiving continuous mandatory ventilation in the control mode at a rate of 10/min.
The inspiratory time control is set at 25%. What is the I:E ratio?
a. 1:3
b. 1:2
c. 1:4
d. 1:1

ANS: A
See Table 42-1.

DIF: Application REF: p. 1013 OBJ: 3

69. A patient is receiving continuous mandatory ventilation through a constant flow generator in the
control mode at a rate of 20/min with a VT of 750 mL. The inspiratory time is 1 second. What is
the flow?

Copyright © 2013, 2009, 2003, 1999, 1995, 1990, 1982, 1977, 1973, 1969 by Mosby, an imprint of Elsevier Inc.
Test bank 41-38

a. 15 L/min
b. 30 L/min
c. 45 L/min
d. 60 L/min

ANS: C
See Table 42-1.

DIF: Application REF: p. 1013 OBJ: 3

70. During pressure-targeted ventilation, which of the following settings determine VT?
1. pressure difference
2. inspiratory time
3. time constant
a. 1 and 2
b. 2 and 3
c. 3 only
d. 1, 2, and 3

ANS: D
See Table 42-1.

DIF: Recall REF: p. 1013 OBJ: 3

71. In which of the following situations is volume-controlled ventilation sometimes used?


1. when a precise PaCO2 has to be maintained (some closed-head injuries)
2. when more even distribution of ventilation is required
3. when ventilating patients with severe, refractory hypoxemia
4. when ventilating patients with unstable or changing ventilatory drives
a. 1 and 2
b. 3 and 4
c. 1, 2, and 3
d. 1, 3, and 4

ANS: A
Volume-controlled continuous mandatory ventilation is indicated when a precise minute
ventilation or blood gas parameter, such as PaCO2, is therapeutically essential to the care of
patients with normal lung mechanics. Theoretically, volume control (with a constant inspiratory
flow) results in a more even distribution of ventilation (compared to pressure control) among
lung units with different time constants where the units have equal resistances but unequal
compliances (e.g., acute respiratory distress syndrome [ARDS]).

DIF: Recall REF: p. 1019 OBJ: 3

72. In which of the following situations is pressure-controlled ventilation sometimes used?


1. when a precise PaCO2 has to be maintained (some closed-head injuries)

Copyright © 2013, 2009, 2003, 1999, 1995, 1990, 1982, 1977, 1973, 1969 by Mosby, an imprint of Elsevier Inc.
Test bank 41-39

2. when more even distribution of ventilation is required


3. when ventilating patients with severe, refractory hypoxemia
4. when tidal volume in unstable due to leaks
a. 1 and 2
b. 3 and 4
c. 1, 2, and 3
d. 1, 3, and 4

ANS: B
Pressure-controlled continuous mandatory ventilation is indicated when adequate oxygenation
has been difficult to achieve in other modes of ventilation. The instability of tidal volume caused
by airway leaks can be minimized by using pressure-controlled ventilation rather than volume
controlled ventilation.

DIF: Recall REF: p. 1019 OBJ: 3

73. Which of the following is the primary parameter used to alter the breath size in pressure
controlled?
a. positive inspiratory pressure (PIP)positive end-expiratory pressure (PEEP)
b. continuous positive airway pressure (CPAP)
c. tidal volume
d. flow

ANS: A
Because tidal volume is not directly controlled, the pressure gradient (PIP – PEEP) is the primary
parameter used to alter the breath size and hence carbon dioxide tensions.

DIF: Recall REF: p. 1013 OBJ: 3

74. Spontaneous breath modes include all of the following except:


a. pressure support ventilation (PSV)
b. continuous positive airway pressure (CPAP)
c. bilevel CPAP (BiPAP)
d. continuous mandatory ventilation assist-control

ANS: D
CPAP, PSV, automatic tube compensation, and proportional assist ventilation are continuous
spontaneous breath modes.

DIF: Recall REF: p. 1018 OBJ: 3

75. What is the mode of ventilatory support in which patient’s inspiratory efforts are augmented with
a set amount of positive airway pressure?
a. intermittent mandatory ventilation
b. continuous mandatory ventilation (CMV)
c. pressure support ventilation (PSV)

Copyright © 2013, 2009, 2003, 1999, 1995, 1990, 1982, 1977, 1973, 1969 by Mosby, an imprint of Elsevier Inc.
Test bank 41-40

d. positive end-expiratory pressure (PEEP)

ANS: C
PSV is a form of PC-CSV that assists the patient’s inspiratory efforts.

DIF: Recall REF: p. 1016 OBJ: 3

76. The respiratory therapist has been called to transport a patient from the emergency department to
obtain a CT scan, which of the following types of ventilator should he therapist chose to
transport the patient?
a. electric
b. apneuistic
c. pneumatic
d. electronic

ANS: C
For patient transport you must use either a pneumatically powered ventilator or one that can run
solely on batteries. Always take along a manually powered bag-valve mask and for long
transports be sure to have back-up power available (extra cylinders or batteries).

DIF: Application REF: p. 1007 OBJ: 2

Copyright © 2013, 2009, 2003, 1999, 1995, 1990, 1982, 1977, 1973, 1969 by Mosby, an imprint of Elsevier Inc.
Test bank 41-41

Physiology of Ventilatory Support

MULTIPLE CHOICE

1. In which of the following types of ventilation is alveolar expansion during inspiration due to a
decrease in pleural pressure?
1. positive-pressure ventilation (PPV)
2. negative-pressure ventilation (NPV)
3. spontaneous ventilation
a. 1 and 2
b. 1 and 3
c. 2 and 3
d. 1, 2, and 3

ANS: C
NPV decreases pleural pressure (Pp1) during inspiration by exposing the chest to subatmospheric
pressure.

2. Which of the following statements are TRUE about negative-pressure ventilation (NPV)?
1. NPV is similar to spontaneous breathing.
2. Airway (mouth) pressure during NPV is zero.
3. Expiration during NPV is by passive recoil.
4. NPV decreases pressure at the body surface.
a. 2 and 4
b. 1, 2, 3, and 4
c. 3 and 4
d. 1, 3, and 4

ANS: B
Mechanical NPV is similar to spontaneous breathing. NPV decreases pleural pressure (Ppl)
during inspiration by exposing the chest to subatmospheric pressure. Negative pressure at the
body surface (Pbs) is transmitted first to the pleural space and then to the alveoli (Palv). Because
the airway opening remains exposed to atmospheric pressure during NPV, a transairway pressure
gradient is created. Thus, gas flows from the relatively high pressure at the airway opening (zero)
to the relatively low pressure in the alveoli (negative).

3. In which of the following types of ventilation is alveolar expansion during inspiration due to an
increase in alveolar pressure?
1. negative-pressure ventilation
2. positive-pressure ventilation
3. spontaneous ventilation
a. 1 and 2
b. 2 and 3
c. 2
d. 1, 2, and 3

Copyright © 2013, 2009, 2003, 1999, 1995, 1990, 1982, 1977, 1973, 1969 by Mosby, an imprint of Elsevier Inc.
Test bank 41-42

ANS: C
Gas flows into the lungs because pressure at the airway opening (Pawo) is positive and alveolar
pressure (Palv) is initially zero or less positive.

4. All of the following occur with positive-pressure ventilation (PPV), except?


a. During inspiration, pleural pressure decreases.
b. During inspiration, pressure in the alveoli increases.
c. The pressure gradients of normal breathing are reversed.
d. During inspiration, alveolar pressure exceeds pleural pressure.

ANS: A
Because alveolar pressure is greater than pleural pressure (Ppl) during PPV, positive pressure is
transmitted from the alveoli to the pleural space, causing pleural pressure to increase during
inspiration.

5. In which of the following types of ventilation can pleural pressure become positive during
inspiration?
1. positive-pressure ventilation
2. spontaneous ventilation
3. negative-pressure ventilation
a. 2 and 3
b. 1 and 2
c. 1, 2, and 3
d. 1

ANS: D
Because alveolar pressure is greater than pleural pressure (Ppl) during PPV, positive pressure is
transmitted from the alveoli to the pleural space, causing pleural pressure to increase during
inspiration.

6. Which of the following conditions is associated with a lack of response to increased FIO2 in
patients receiving positive-pressure ventilation?
a. dead space
b. Shunt
c. Hypoxemia
d. Hypoventilation

ANS: B
If the patient is receiving mechanical ventilation and has adequate alveolar ventilation, failure of
the PaO2 to respond to increased FIO2 likely means that the hypoxemia is due to diffusion defect
or shunt.

7. Which of the following strategies are useful in the management of shunt?

Copyright © 2013, 2009, 2003, 1999, 1995, 1990, 1982, 1977, 1973, 1969 by Mosby, an imprint of Elsevier Inc.
Test bank 41-43

1. positive end-expiratory pressure (PEEP)


2. permissive hypercapnia
3. control of membrane permeability
a. 2 and 3
b. 1 and 3
c. 1, 2, and 3
d. 1 only

ANS: B
The use of PEEP and control of membrane permeability accompany the management of shunt.

8. Administration of positive end-expiratory pressure (PEEP) or continuous positive airway


pressure (CPAP) is associated with all of the following benefits except maintaining:
a. alveoli open
b. alveoli stable
c. fluid-filled alveoli open
d. surfactant-depleted alveoli closed

ANS: D
Administration of PEEP with mechanical ventilation or to a spontaneously breathing patient in
the form of CPAP helps to maintain open and stabilize small, collapsed, or fluid-filled alveoli.

9. Which of the following is considered a normal spontaneous tidal volume?


a. 3 to 5 mL/kg
b. 5 to 7 mL/kg
c. 7 to 9 mL/kg
d. 10 to 12 mL/kg

ANS: B
A normal spontaneous tidal volume is approximately 5 to 7 mL/kg.

10. Which of the following is the recommended tidal volume for mechanical ventilation of a patient
with ARDS who is in acute respiratory failure?
a. 4 to 8 mL/kg
b. 3 to 5 mL/kg
c. 6 to 10 mL/kg
d. 10 to 12 mL/kg

ANS: A
The currently accepted tidal volume for mechanical ventilation in acute respiratory failure is 4 to
8 mL/kg for patients with acute respiratory distress syndrome (ARDS).

11. Which of the following conditions does not require high mechanical respiratory rates?
a. metabolic alkalosis
b. ARDS
c. increased intracranial pressure

Copyright © 2013, 2009, 2003, 1999, 1995, 1990, 1982, 1977, 1973, 1969 by Mosby, an imprint of Elsevier Inc.
Test bank 41-44

d. metabolic acidosis

ANS: A
Conditions that may necessitate a higher initial rate include ARDS, acutely increased intracranial
pressure (with caution) and metabolic acidosis.

12. Which of the following is the explanation for the increased ratio when excessive positive
end-expiratory pressure (PEEP) is used?
a. diversion of blood from ventilated alveoli to hypoventilated alveoli
b. diversion of blood from hypoventilated alveoli to ventilated alveoli
c. shunt-like effect
d. Hyperexpansion

ANS: A
When excessive PEEP is used, blood flow is diverted from ventilated alveoli to hypoventilated
alveoli; the result is an increased ratio.

13. Which of the following is the consequence of decreased resistance and compliance?
a. It takes more time to fill the alveoli.
b. It takes more time to empty the alveoli.
c. It takes less time to fill and more time to empty the alveoli.
d. It takes less time to fill and empty the alveoli.

ANS: D
If compliance or resistance decreases, the time constant for a given lung unit decreases, and the
lung fills and empties faster.

14. Mean airway pressure may be increased by all of the following adjustments, except increasing
the:
a. inspiratory time
b. frequency
c. positive end-expiratory pressure level
d. FIO2

ANS: D

15. During pressure-targeted modes of ventilatory support, the volume delivered depends on which
of the following?
1. set pressure limit
2. patient lung mechanics
3. patient effort
a. 1 and 2
b. 1 and 3
c. 2 and 3
d. 1, 2, and 3

Copyright © 2013, 2009, 2003, 1999, 1995, 1990, 1982, 1977, 1973, 1969 by Mosby, an imprint of Elsevier Inc.
Test bank 41-45

ANS: D
The volume delivered during pressure-controlled modes varies with changes in set pressure,
patient effort, and lung mechanics.

16. All of the following occur in pressure-limited modes of ventilation, except:


a. The volume delivered at a given pressure must decrease as compliance falls.
b. The inspiratory flow varies with patient effort and lung mechanics.
c. Active effort by the patient against inspiration will decrease delivered volume.
d. The volume delivered at a given pressure must decrease as Raw rises.

ANS: D
An increase in resistance, active exhalation, or muscle tensing by the patient during inspiration
also decreases delivered volume in pressure ventilation.

17. The volume delivered by a pressure-limited ventilator will decrease under all of the following
conditions, except:
a. the patient’s lung or thoracic (chest wall) compliance falls.
b. airway resistances rises (inspiratory time less than 3 times the time constant).
c. the patient tenses the respiratory muscles during inspiration.
d. airway resistances rises (inspiratory time greater than 3 times the time constant).

ANS: B
However, if insufficient time is available for pressure equilibration, delivered volume decreases
as airway resistance increases.

18. Which of the following are considered safe settings for a recruitment maneuver?
1. pressures up to 50 cm H2O
2. pressures up to 35 cm H2O
3. pressures applied for 5 to 10 minutes
4. pressures applied for 1 to 3 minutes
a. 1 and 3
b. 1 and 4
c. 2 and 3
d. 2 and 4

ANS: B
The maximum pressure needed to recruit a given patient’s lung is unknown; however, most
agree that pressures up to 50 cm H2O are safe with most patients when applied for short (1 to 3
minutes) periods of time.

19. The magnitude of WOBpt depends on which of the following?


1. compliance
2. resistance
3. ventilatory drive

Copyright © 2013, 2009, 2003, 1999, 1995, 1990, 1982, 1977, 1973, 1969 by Mosby, an imprint of Elsevier Inc.
Test bank 41-46

4. trigger sensitivity
5. peak flow
a. 1 and 3
b. 1, 2, and 4
c. 1, 2, 3, 4, and 5
d. 1, 2, and 5

ANS: C
The magnitude of WOBpt depends on compliance, resistance, and ventilatory drive and on
ventilator variables, such as trigger sensitivity, peak flow, and tidal volume.

20. In which of the following modes of ventilatory support would the patient’s work of breathing be
greatest?
a. continuous positive airway pressure (CPAP)
b. pressure-supported ventilation (PSV)
c. intermittent mandatory ventilation (IMV)
d. continuous mandatory ventilation (CMV)

ANS: A
As the mode is changed from CPAP to PSV to synchronized intermittent mandatory ventilation
to time-triggered CMV, the ventilator assumes more of the work.

21. In which of the following modes of ventilatory support would the patient’s work of breathing be
least?
a. continuous positive airway pressure (CPAP)
b. pressure-supported ventilation (PSV)
c. intermittent mandatory ventilation (IMV)
d. continuous mandatory ventilation (CMV)

ANS: D
As the mode is changed from CPAP to PSV to synchronized intermittent mandatory ventilation
to time-triggered CMV, the ventilator assumes more of the work

22. When bedside work of breathing measures are unavailable, you should adjust the level of
pressure-supported ventilation (PSV) to which of the following breathing patterns?
Spontaneous Rate VT
a. S
b.
c.
d.

Copyright © 2013, 2009, 2003, 1999, 1995, 1990, 1982, 1977, 1973, 1969 by Mosby, an imprint of Elsevier Inc.
Test bank 41-47

ANS: A
Most clinicians increase PSV until the breathing pattern approaches normal, that is, until the
spontaneous ventilatory rate is 15 to 25 breaths/min and the spontaneous tidal volume (VT) is
normal (5 to 8 mL/kg).

23. To prevent muscle fatigue or atrophy, the level of PSV should be adjusted to achieve what work
load?
a. 0 J/L
b. 0.6 to 0.8 J/L
c. 0 to 0.5 J/L
d. >0.8 J/L

ANS: B
Normal work of breathing is 0.6 to 0.8 J/L.

24. Which level of plateau pressure increases the likelihood of causing lung injury?
a. > 15 cm H2O
b. > 25 cm H2O
c. > 30 cm H2O
d. It doesn’t matter as long as positive inspiratory pressure is less than 50 cm H2O.

ANS: C
Alveolar pressures of 30 cm H2O or greater have an increased likelihood of causing lung injury.

25. All of the following factors would tend to increase mean airway pressure except:
a. short inspiratory times
b. increased mandatory breaths
c. increased levels of positive inspiratory pressure (PIP)
d. increased levels of positive end-expiratory pressure (PEEP)

ANS: A
Mean airway pressure is decreased by decreasing inspiratory time, tidal volume, respiratory rate,
PEEP, or PIP.

26. Primary indications for using positive end-expiratory pressure (PEEP) in conjunction with
mechanical ventilation include which of the following?
1. when dynamic hyperinflation occurs in chronic obstructive pulmonary disease (COPD)
patients
2. when the imposed work of breathing is excessive

Copyright © 2013, 2009, 2003, 1999, 1995, 1990, 1982, 1977, 1973, 1969 by Mosby, an imprint of Elsevier Inc.
Test bank 41-48

3. when acute lung injury causes refractory hypoxemia


a. 1 and 2
b. 1 and 3
c. 2 and 3
d. 1, 2, and 3

ANS: B
PEEP is used primarily to improve oxygenation in patients with refractory hypoxemia. PEEP
may be indicated in the care of patients with COPD who have dynamic hyperinflation (auto-
PEEP) during mechanical ventilatory support after other efforts to decrease auto-PEEP fail.

27. In which of the following patients is positive end-expiratory pressure (PEEP) most indicated?
FIO2 PaO2
a.
b.
c.
d.

ANS: C
As a rule, refractory hypoxemia exists when a patient’s PaO2 cannot be maintained above 50 to
60 mm Hg with an FIO2 of 0.40 to 0.50 or more.

28. Beneficial physiological effects of positive end-expiratory pressure (PEEP) include which of the
following?
1. increased PaO2 for given FIO2
2. increased lung compliance (CL)
3. decreased shunt fraction
4. increased functional residual capacity
a. 1, 2, 3, and 4
b. 3 and 4
c. 2, 3, and 4
d. 2 and 4

ANS: A

29. Detrimental effects of positive end-expiratory pressure (PEEP) include all of the following
except:
a. increased incidence of barotrauma
b. decreased venous return or cardiac output
c. increased pulmonary vascular resistance
d. increased CL

ANS: D
See Table 43-2.

Copyright © 2013, 2009, 2003, 1999, 1995, 1990, 1982, 1977, 1973, 1969 by Mosby, an imprint of Elsevier Inc.
Test bank 41-49

30. Contraindications for using positive end-expiratory pressure (PEEP) in conjunction with
mechanical ventilation include which of the following?
1. untreated bronchopleural fistula
2. chronic airway obstruction
3. untreated pneumothorax
a. 1 and 2
b. 1 and 3
c. 2 and 3
d. 1, 2, and 3

ANS: B
PEEP is contraindicated in the presence of an unmanaged bronchopleural fistula or
pneumothorax.

31. Compared with a square wave flow pattern, a decelerating flow waveform has all of the
following potential benefits except:
a. reduced peak pressure
b. improved cardiac output
c. less inspiratory work
d. decreased volume of dead spacetotidal volume ratio (VD/VT)

ANS: B
Compared with a square flow waveform, decreasing flow has been shown to reduce peak
pressure, inspiratory work, VD/VT, and P(A–a)O2 without affecting hemodynamic values.

32. Which of the following is a benefit of high inspiratory flows during positive-pressure
ventilation?
a. improved gas exchange
b. higher peak pressures
c. reduced air trapping
d. higher work of breathing

ANS: C
High ventilator inspiratory flow allows more time for exhalation and reduces the incidence of air
trapping.

33. Physiological effects of adding a volume-limited inflation hold to mandatory breaths include
which of the following?
1. decreased PaCO2
2. increased inspiratory time
3. decreased VD/VT
4. longer expiratory times
a. 2 and 4
b. 1, 2, 3, and 4

Copyright © 2013, 2009, 2003, 1999, 1995, 1990, 1982, 1977, 1973, 1969 by Mosby, an imprint of Elsevier Inc.
Test bank 41-50

c. 3 and 4
d. 1, 2, and 3

ANS: D
By momentarily maintaining lung volume under conditions of no flow, an inflation hold allows
additional time for gas redistribution between lung units with different time constants. In both
animal and human studies, increasing the length of an inflation hold decreases the VD/VT, PaCO2,
and inert gas washout time. Adding an inflation hold effectively increases total inspiratory time,
thereby shortening the time available for exhalation.

34. Volume-controlled (VC) modes of mechanical ventilation include which of the following?
1. VC continuous mandatory ventilation
2. VC intermittent mandatory ventilation
3. volume-assured, pressure-controlled
4. bilevel positive airway pressure
a. 2 and 4
b. 1, 2, 3, and 4
c. 1 and 2
d. 1, 3, and 4

ANS: C
VC modes include VC continuous mandatory ventilation and VC synchronized intermittent
mandatory ventilation.

35. Which of the following modes of support provides all of the patient’s minute ventilation (V E) as
mandatory volume-controlled (VC) breaths?
a. VC continuous mandatory ventilation
b. VC intermittent mandatory ventilation
c. pressure-supported ventilation
d. continuous positive airway pressure

ANS: A
VC continuous mandatory ventilation provides all of the patient’s minute ventilation as
mandatory breaths.

DIF: Recall REF: p. 1059 OBJ: 3

36. Which of the following modes of ventilatory support would result in the highest mean airway
pressure?
a. volume-controlled intermittent mandatory ventilation
b. (volume-controlled intermittent mandatory ventilation) + pressure-supported
ventilation
c. pressure-controlled intermittent mandatory ventilation
d. volume-controlled continuous mandatory ventilation

ANS: D

Copyright © 2013, 2009, 2003, 1999, 1995, 1990, 1982, 1977, 1973, 1969 by Mosby, an imprint of Elsevier Inc.
Test bank 41-51

Because every breath is volume controlled, mean airway pressure tends to be greater compared
with the mean airway pressure with synchronized intermittent mandatory ventilation and
pressure-supported ventilation, and pulmonary arterial pressure and cardiac output may be lower.

37. What are some key causes of patient-ventilator asynchrony and increased work of breathing
during pressure-triggered volume-controlled continuous mandatory ventilation?
1. improper trigger setting
2. insufficient inspiratory flow
3. high peak airway pressures
a. 1 and 2
b. 1 and 3
c. 2 and 3
d. 1, 2, and 3

ANS: A
If sensitivity is set too low, such that considerable effort is necessary to trigger the ventilator,
patientventilator asynchrony occurs. A pressure sensitivity of –0.5 to –1.5 cm H2O or flow
sensitivity of 1 to 2 L/min is regarded as optimal. Inspiratory flow must be set to meet the
patient’s inspiratory demand. An insufficient inspiratory flow can cause patientventilator
asynchrony and increased work of breathing.

38. Inspection of the airway pressure waveform of a patient receiving volume-controlled continuous
mandatory ventilation assist–control with constant flow reveals a large dip or drop in pressure at
the beginning of inspiration. Which of the following problems is most likely?
a. The trigger setting is improper.
b. The inspiratory flow is inadequate.
c. The set volume is too large.
d. The pressure limit is too low.

ANS: A
At the beginning of inspiration, a small negative deflection on the manometer suggests an
appropriate sensitivity setting.

39. During volume-controlled continuous mandatory ventilation, should either compliance decrease
or airway resistance (Raw) increase, what will happen?
a. The peak airway pressure will decrease.
b. The inspiratory flow will increase.
c. The peak airway pressure will increase.
d. The inspiratory time will decrease.

ANS: C
Should compliance decrease or airway resistance increase, the pressure needed to deliver the
volume increases.

Copyright © 2013, 2009, 2003, 1999, 1995, 1990, 1982, 1977, 1973, 1969 by Mosby, an imprint of Elsevier Inc.
Test bank 41-52

40. The volume of gas actually delivered to a patient by most positive-pressure ventilation is always
less than that expelled from the machine. Which of the following factors help to explain this
finding?
1. gas compression under pressure
2. presence of built-in leaks
3. expansion of the ventilator circuitry
a. 2 and 3
b. 1 and 2
c. 1, 2, and 3
d. 1 and 3

ANS: D
First, gases are compressed when delivered under pressure. Thus, the generated volume (at
atmospheric pressure) occupies less space when delivered under pressure. Second, most
ventilator circuits are somewhat compliant.

41. After accounting for the compressed volume loss on a stable adult patient receiving volume-
controlled continuous mandatory ventilation at a preset volume of 700 mL, you still note a 150-
mL difference between the expected and the actual delivered volume. Which of the following is
most likely causing this problem?
a. gas absorption across the alveolar membrane
b. increase in the respiratory quotient
c. bronchopleural fistula or pneumothorax
d. leak in the patient–ventilator system

ANS: D
An additional factor that can cause a patient to receive less volume than the ventilator delivers is
a leak.

42. Pressure-controlled (PC) modes of ventilatory support include all of the following except:
a. pressure-controlled continuous mandatory ventilation (PC-CMV)
b. pressure-controlled intermittent mandatory ventilation
c. volume-controlled continuous mandatory ventilation
d. volume-assured pressure-supported ventilation (VAPSV)

ANS: C
Included in this category are PC-CMV, pressure-controlled SIMV (PC-SIMV), pressure-
supported ventilation (PSV), continuous positive airway pressure (CPAP), airway pressure
release ventilation (APRV), bilevel positive airway pressure (BiPAP), and VAPSV and within-
breath dual control modes with a volume target (volume support ventilation and pressure-
regulated volume control).

43. Which of the following modes of ventilatory support is used to help decrease airway and alveolar
pressures?
a. pressure-controlled continuous mandatory ventilation
b. pressure-controlled intermittent mandatory ventilation

Copyright © 2013, 2009, 2003, 1999, 1995, 1990, 1982, 1977, 1973, 1969 by Mosby, an imprint of Elsevier Inc.
Test bank 41-53

c. volume-controlled continuous mandatory ventilation


d. volume-assured pressure-supported ventilation

ANS: A
Pressure-controlled continuous mandatory ventilation may be used to reduce airway and alveolar
pressures in any ventilated patient.

44. What mode of pressure-controlled ventilation is designed to prevent alveoli with short time
constants from collapsing, thereby improving oxygenation?
a. pressure-controlled inverse ration ventilation
b. pressure-controlled intermittent mandatory ventilation
c. volume-assured pressure-supported ventilation
d. bilevel positive airway pressure

ANS: A
Because alveoli affected by ALI/ARDS have short time constants, more time is allotted for
inspiration and less time is allotted for expiration.

45. A patient switched from pressure-controlled continuous mandatory ventilation (CMV) with
positive end-expiratory pressure (PEEP) to pressure-controlled inverse ratio ventilation (PC-
IRV) shows a good improvement in PaO2 but a decrease in tissue oxygenation. Which of the
following best explains this observation?
a. High mean pressures caused by PC-IRV decreased pulmonary blood flow.
b. Intrinsic PEEP caused by PC-IRV resulted in increased alveolar recruitment.
c. High mean pressures caused by PC-IRV decreased cardiac output.
d. Intrinsic PEEP caused by PC-IRV compressed the pulmonary capillaries.

ANS: C
Although some studies have shown improvement in oxygenation with PC-IRV versus CMV with
PEEP, others have shown concurrent decreases in cardiac output.

46. In which mode of ventilatory support does the patient breathe spontaneously at an elevated
airway pressure, with short, intermittent decreases in pressure to a lower level?
a. volume-assured pressure-supported ventilation
b. pressure-controlled inverse ratio ventilation (PC-IRV)
c. bilevel positive airway pressure
d. airway pressure release ventilation (APRV)

ANS: D
A mode related to PC-IRV is APRV, in which the patient breathes spontaneously throughout
periods of high and low applied continuous positive airway pressure.

47. What does pressure-supported ventilation consist of?

Copyright © 2013, 2009, 2003, 1999, 1995, 1990, 1982, 1977, 1973, 1969 by Mosby, an imprint of Elsevier Inc.
Test bank 41-54

a. patient-triggered, pressure-limited, flow-cycled breaths


b. machine-triggered, pressure-limited, flow-cycled breaths
c. patient-triggered, pressure-limited, time-cycled breaths
d. machine-triggered, flow-limited, pressure-cycled breaths

ANS: A
Pressure-supported ventilation is a pressure-targeted mode of ventilation that is patient triggered,
pressure limited, and flow cycled (Figure 43-16).

48. What are some primary uses for pressure-supported ventilation (PSV)?
1. recruiting collapsed alveoli and improving oxygenation
2. augmenting patient’s spontaneous VT
3. overcoming the imposed work of breathing
a. 1 and 2
b. 1 and 3
c. 2 and 3
d. 1, 2, and 3

ANS: C
Since the first description of PSV in 1982, it has been used either to overcome the imposed
resistance associated with the artificial airway or to provide ventilatory support with minimal
control.

49. For patients with respiratory insufficiency, pressure-supported ventilation (PSV) has all of the
following advantages over spontaneous breathing except:
a. decreased respiratory rate
b. increased VT
c. decreased O2 consumption
d. increased muscle activity

ANS: D
Clinical studies have shown that compared with spontaneous breathing (including that occurring
during synchronized intermittent mandatory ventilation), PSV can result in a decreased
respiratory rate, increased tidal volume, reduced respiratory muscle activity, and decreased
oxygen consumption.

50. What spontaneous pressure-controlled breath mode allows separate regulation of the inspiratory
and expiratory pressures?
a. bilevel positive airway pressure (BiPAP)
b. continuous positive airway pressure (CPAP)
c. pressure-supported ventilation
d. pressure-controlled intermittent mandatory ventilation

ANS: A

Copyright © 2013, 2009, 2003, 1999, 1995, 1990, 1982, 1977, 1973, 1969 by Mosby, an imprint of Elsevier Inc.
Test bank 41-55

With BiPAP, inspiratory positive airway pressure (IPAP or pressure-supported ventilation) and
expiratory positive airway pressure (EPAP or positive end-expiratory pressure) are set.

51. Bilevel positive airway pressure (BiPAP) is used for all of the following purposes except:
a. nocturnal ventilatory support of chronic disease patients
b. preventing intubation of patients with acute exacerbations of chronic obstructive
pulmonary disease (COPD)
c. treatment of obstructive sleep apnea (OSA) in the home
d. providing ventilatory support for patients with status asthmaticus

ANS: D
Although it was originally developed to enhance the capabilities of home CPAP systems used for
management of OSA, BiPAP has since been successfully used in the home and the hospital for
noninvasive ventilatory support of patients with acute and chronic respiratory failure. BiPAP has
been found to be useful in the prevention of intubation in acute exacerbation of COPD.

52. Which of the following modes of ventilatory support combines the advantages of pressure-
controlled and volume-controlled ventilation?
a. volume-assured pressure-supported ventilation
b. pressure-supported ventilation
c. bilevel positive airway pressure
d. airway pressure release ventilation

ANS: A
Pressure-supported ventilation with a volume guarantee is the goal of volume-assured pressure-
supported ventilation.

53. During volume-assured pressure-supported ventilation, the breath will be pressure-limited under
what conditions?
a. The delivered tidal volume (VT) is greater than the preset minimum VT.
b. The patient’s lung or thoracic compliance decreases from the baseline.
c. The delivered VT is less than the preset minimum VT.
d. The patient’s Raw increases from baseline.

ANS: A
If delivered tidal volume is greater than the preset minimum tidal volume, the breath becomes a
pressure-supported breath.

54. During volume-assured pressure-supported ventilation, if the desired VT is not reached or


exceeded at the preset pressure support level, what happens?
a. Flow continues at a constant rate until the desired volume is achieved.
b. The breath terminates when a predetermined low flow is achieved.
c. Flow decreases exponentially until the desired volume is achieved.
d. Flow increases linearly until the desired volume is achieved.

Copyright © 2013, 2009, 2003, 1999, 1995, 1990, 1982, 1977, 1973, 1969 by Mosby, an imprint of Elsevier Inc.
Test bank 41-56

ANS: A
Although a minimum tidal volume is guaranteed, volume-assured pressure-supported ventilation
allows tidal volume to exceed the set level according to patient demand.

55. What are some physiological advantages of volume-assured pressure-supported ventilation?


1. improved patient–ventilator synchrony
2. increased pressure-time product
3. decreased work of breathing
a. 1 and 2
b. 1 and 3
c. 2 and 3
d. 1, 2, and 3

ANS: B
Physiological effects of volume-assured pressure-supported ventilation include improved
patientventilator synchrony and reduced pressure-time product, which is an indicator of
decreased work of breathing.

56. All of the following are TRUE about continuous positive airway pressure (CPAP), except:
a. It maintains alveoli at greater inflation volumes.
b. It holds airway pressure essentially constant.
c. It provides the pressure gradient needed for ventilation.
d. It has side effects similar to those of positive pressure ventilation.

ANS: C
Because airway pressure does not change, CPAP does not provide ventilation.

57. Which of the following variables determine the level of support achieved with adaptive support
ventilation?
1. patient effort
2. flow
3. time constant
a. 1 and 3
b. 2
c. 1
d. 1, 2, and 3

ANS: A
Adaptive support ventilation is a dual-controlled mode of ventilation in which an automated
increase or decrease in ventilatory support is based on patient effort and time constants.

58. Which of the following variables determine the level of support achieved with proportional assist
ventilation?
1. patient effort
2. elastance

Copyright © 2013, 2009, 2003, 1999, 1995, 1990, 1982, 1977, 1973, 1969 by Mosby, an imprint of Elsevier Inc.
Test bank 41-57

3. resistance
a. 1 and 3
b. 2
c. 1
d. 1, 2, and 3

ANS: D
Proportional assist ventilation is a mode of ventilation designed to vary inspiratory pressure in
proportion to patient effort, elastance, and resistance.

59. Which of the following variables determine the level of support achieved with adaptive support
ventilation?
1. patient effort
2. elastance
3. resistance of the endotracheal tube
a. 1 and 3
b. 2
c. 1
d. 1, 2, and 3

ANS: A
Automatic tube compensation is similar to the flow assist of proportional assist ventilation but
only considers the resistance of the endotracheal tube.

60. Which of the following parameters are set when volume-supported ventilation (VSV) is used?
1. tidal volume
2. maximum peak pressure
3. positive end-expiratory pressure (PEEP)
4. flow
a. 1 and 3
b. 2
c. 1, 2, 3, and 4
d. 1, 2, and 3

ANS: D
In VSV, a desired tidal volume, maximum peak pressure, FIO2, and PEEP are set.

61. What factor primarily determines the effect of positive-pressure ventilation (PPV) on the cardiac
output?
a. peak airway pressure
b. mean pleural pressure
c. CO2
d. expiratory time

ANS: B

Copyright © 2013, 2009, 2003, 1999, 1995, 1990, 1982, 1977, 1973, 1969 by Mosby, an imprint of Elsevier Inc.
Test bank 41-58

More specifically, the decrease in left ventricular output corresponded to the increase in pleural
pressure that occurred with PPV.

62. Which of the following are potential effects of positive-pressure ventilation on the cardiovascular
system?
1. decreased venous return
2. decreased cranial perfusion pressures
3. increased pulmonary blood flow
4. decreased ventricular stroke volume
a. 2 and 4
b. 1 and 4
c. 3 and 4
d. 1, 2, 3, and 4

ANS: B
Positive pleural pressure compresses the intrathoracic veins and increases central venous and
right atrial filling pressures. As these pressures increase, venous return to the heart is impeded
and right ventricular preload and stroke volume decrease, as does pulmonary blood flow.

63. Moderate rises in pleural pressure during positive-pressure ventilation have a minimal effect on
cardiac output in normal subjects. What are some reasons for this lack of effect?
1. compensatory dilation of the large arteries
2. compensatory increase in venomotor tone
3. compensatory increase in the cardiac rate
a. 2 and 3
b. 1 and 2
c. 1, 2, and 3
d. 1 and 3

ANS: A
Compensatory mechanisms used to counter the decrease in stroke volume include an increased
heart rate, an increase in systemic vascular and peripheral venous resistance, and shunting of
blood away from the kidneys and lower extremities, which results in a consistent blood pressure.

64. Ventricular dysfunction occurs in patients receiving positive-pressure ventilation for all of the
following reasons except:
a. hypovolemia
b. excessive tidal volume
c. receiving more than optimal positive end-expiratory pressure (PEEP)
d. hypervolemia

ANS: D
It appears that right or left ventricular dysfunction occurs if the patient is hypovolemic, receiving
an excessive tidal volume, or receiving more than optimum PEEP.

Copyright © 2013, 2009, 2003, 1999, 1995, 1990, 1982, 1977, 1973, 1969 by Mosby, an imprint of Elsevier Inc.
Test bank 41-59

65. Assuming a constant rate of breathing, which of the following inspiratory/expiratory ratios (1:E)
would tend to most greatly impair a patient’s systemic diastolic pressure?
a. 1:4
b. 1:3
c. 1:2
d. 1:1

ANS: D
The factors of positive-pressure ventilation that may decrease the systemic diastolic pressure are
high mean airway pressure, due to a high positive end-expiratory pressure, high tidal volume, or
long inspiratory time.

66. Potential effects of hyperventilation on the central nervous system include which of the
following?
1. increased O2 consumption
2. increased cerebral vascular resistance (CVR)
3. increased intracranial pressure (ICP)
a. 1 and 2
b. 2 and 3
c. 1 and 3
d. 1, 2, and 3

ANS: B
When mechanical hyperventilation is used, CVR increases, and the result is decreased ICP.

67. Hyperventilation should generally be avoided during mechanical ventilatory support. Exceptions
to this rule include:
1. trying to calm an agitated patient.
2. failure of other methods to reduce intracranial pressure
3. hypokalemia causing cardiac arrhythmias
a. 2 and 3
b. 1 and 3
c. 2
d. 1 and 2

ANS: C
Hyperventilation should be used temporarily after traumatic brain injury until other methods can
be used to decrease elevated intracranial pressure.

68. A patient receiving long-term positive-pressure ventilation support exhibits a progressive weight
gain and a reduction in the hematocrit. Which of the following is the most likely cause of this
problem?
a. pulmonary hemorrhage
b. water retention

Copyright © 2013, 2009, 2003, 1999, 1995, 1990, 1982, 1977, 1973, 1969 by Mosby, an imprint of Elsevier Inc.
Test bank 41-60

c. hypovolemia
d. hyponatremia

ANS: B
Among critically ill patients, water retention usually is evident when rapid weight gain occurs. In
addition, such patients may have a reduced hematocrit, which is also consistent with
hypervolemia due to water retention.

69. Positive-pressure ventilation (PPV) can reduce urinary output by how much?
a. 10% to 20%
b. 30% to 50%
c. 60% to 70%
d. 80% to 90%

ANS: B
In terms of direct effect, PPV can reduce urinary output as much as 30% to 50%.

70. Which of the following mechanisms explains the impaired renal function seen in patients
receiving ventilatory support with positive pressure?
1. decreased secretion of aldosterone
2. decreased intravascular volume
3. increased secretion of vasopressin
a. 1
b. 2
c. 1 and 3
d. 1, 2, and 3

ANS: B
Results of more recent analysis tend to refute this explanation, instead showing that impaired
renal function during positive-pressure ventilation is better associated with a decrease in
intravascular volume.

71. Which of the following is the best explanation for the decreased levels of atrial natriuretic
hormone commonly observed among patients receiving positive-pressure ventilation?
a. stimulation of the pulmonary stretch receptors
b. inhibition of posterior pituitary function
c. inhibition of the cortex of the adrenal gland
d. decreased right atrial transmural pressure

ANS: D
Decreased right atrial transmural pressure is primarily responsible for the decrease in atrial
natriuretic hormone, which leads to sodium retention.

72. Which of the following mechanisms explains the hepatic dysfunction in patients receiving
positive-pressure ventilation (PPV)?
a. decreased hepatic blood flow

Copyright © 2013, 2009, 2003, 1999, 1995, 1990, 1982, 1977, 1973, 1969 by Mosby, an imprint of Elsevier Inc.
Test bank 41-61

b. increased portal venous pressure


c. hepatic congestion
d. increased bilirubin conjugation

ANS: A
These effects appear to be directly related to the reduction in hepatic blood flow that occurs with
PPV.

73. Which of the following gastrointestinal conditions are commonly associated with long-term
positive-pressure ventilation (PPV)?
1. bleeding
2. ulceration
3. diarrhea
a. 1 and 2
b. 2 and 3
c. 1 and 3
d. 1, 2, and 3

ANS: A
An increase in splanchnic resistance can contribute to gastric mucosal ischemia and helps
explain the high incidence of gastrointestinal bleeding and stress ulceration in patients receiving
long-term PPV.

74. What is traumatic injury to lung tissue caused by excessive pressure called?
a. pulmonary barotrauma
b. pulmonary hemorrhage
c. pulmonary infarction
d. pulmonary embolism

ANS: A
High ventilation pressure has long been associated with barotrauma.

75. Types of damage associated with pulmonary barotrauma include all of the following except:
a. pneumoconiosis
b. pneumomediastinum
c. pneumothorax
d. subcutaneous emphysema

ANS: A
Barotrauma is categorized as pneumothorax, pneumomediastinum, pneumopericardium, and
subcutaneous emphysema (Figure 43-21).

76. Physical assessment indicating the presence of a tension pneumothorax includes all of the
following except:
a. unequal chest excursion

Copyright © 2013, 2009, 2003, 1999, 1995, 1990, 1982, 1977, 1973, 1969 by Mosby, an imprint of Elsevier Inc.
Test bank 41-62

b. hyperresonance upon chest percussion


c. absent breath sounds
d. loud breath sounds

ANS: D
Pneumothorax is identified by observation of a decrease in chest movement, hyperresonance on
percussion, and decreased or absent breath sounds over the affected side.

77. Which of the following terms describe the lung injury associated with the use of low tidal
volumes?
a. biotrauma
b. barotrauma
c. volutrauma
d. atelectrauma

ANS: D
Lung damage may also occur when ventilating at low tidal volumes, if alveoli are allowed to
deflate and reinflate repeatedly with each breath. This injury is called atelectrauma.

78. Which of the following terms describes the lung injury associated with the release of
prostanoids?
a. biotrauma
b. barotrauma
c. volutrauma
d. atelectrauma

ANS: A
These biochemical signals cause the release of cytokines, complement, prostanoids, leukotrienes,
reactive oxygen species, and proteases. The release of these substances has been called
“biotrauma.”

79. Which of the following lung units would be most prone to air-trapping?
a. one with high resistance and low compliance
b. one with low resistance and low compliance
c. one with normal resistance and low compliance
d. one with high resistance and high compliance

ANS: D
Air-trapping occurs with incomplete emptying of lung units. Lung units prone to air-trapping are
those with long-time constants (1.e., with high resistance or high compliance).

80. What factors contribute to the development of autopositive end-expiratory pressure (PEEP)?
1. high expiratory Raw
2. high inspiratory flows
3. inadequate expiratory time
a. 1 and 2

Copyright © 2013, 2009, 2003, 1999, 1995, 1990, 1982, 1977, 1973, 1969 by Mosby, an imprint of Elsevier Inc.
Test bank 41-63

b. 1 and 3
c. 2 and 3
d. 1, 2, and 3

ANS: B
By effectively increasing the time constant of the lung, high expiratory resistance prolongs
exhalation to the point at which air trapping begins (Figure 43-23, B). Any shortening of the
expiratory time (Figure 43-23, C) aggravates the problem and increases both distal airway
pressure and lung volume (auto-PEEP).

81. Detrimental effects of autopositive end-expiratory pressure (PEEP) include all of the following
except:
a. increased work of breathing
b. increased pulmonary barotrauma
c. decreased pulmonary vascular resistance
d. decreased venous return

ANS: C
By increasing FRC and alveolar pressure, auto-PEEP increases the risk and severity of
barotrauma and volutrauma. Auto-PEEP also increases the work of breathing and impedes
venous return, the result being a decrease in cardiac output. Auto-PEEP also can increase
pulmonary vascular resistance.

82. What patients are at greatest risk for autopositive end-expiratory pressure (PEEP)?
1. those supported by spontaneous breath modes
2. those with high airway resistance
3. those with high expiratory flow resistance
a. 1 and 2
b. 2 and 3
c. 1 and 3
d. 1, 2, and 3

ANS: B
Patients at greatest risk of development of auto-PEEP are those with high airway resistance who
are being supported by modes that limit expiratory time.

83. Ventilatory support strategies likely to result in autopositive end-expiratory pressure (PEEP)
include all of the following except:
a. continuous mandatory ventilation (CMV) assist-control
b. inverse ratio ventilation (IRV)
c. low-rate intermittent mandatory ventilation
d. low inspiratory flows

ANS: C

Copyright © 2013, 2009, 2003, 1999, 1995, 1990, 1982, 1977, 1973, 1969 by Mosby, an imprint of Elsevier Inc.
Test bank 41-64

High-risk ventilatory support techniques include any method that increases the 1:E ratio,
especially CMV at a high rate or in the assist-control mode, and approaches that purposefully
shorten expiratory time, such as IRV or the use of low inspiratory flow.

84. The increased work of breathing associated with autopositive end-expiratory pressure (PEEP)
during mechanical ventilation is due to:
1. hyperinflation or impaired contractility of the diaphragm
2. large alveolar pressure drops required to trigger breaths
3. increased volume of the intrathoracic airways
a. 1 and 2
b. 1 and 3
c. 2 and 3
d. 1, 2, and 3

ANS: A
First, hyperinflation caused by auto-PEEP stretches the lung, and the stretching impairs the
contractile action of the diaphragm. Second, in pressure- or flow-triggered breaths, the high
alveolar pressure caused by auto-PEEP must be overcome before any airway pressure change
can occur.

85. Which level of FIO2 and what time of exposure have been associated with oxygen toxicity?
1. FIO2 0.5
2. FIO2 0.7
3. FIO2 0.6
4. 24 to 48 hours
5. 48 to 96 hours
a. 1 and 5
b. 3 and 4
c. 3 and 5
d. 1 and 4

ANS: B
An FIO2 of 0.5 or more for longer than 24 to 48 hours is associated with the development of
oxygen toxicity.

86. Which of the following groups of patients are primarily affected by ventilator-associated
pneumonia?
1. infants
2. adults older than 65 years
3. immunosuppressed
4. thoracoabdominal surgery
5. depressed sensorium
a. 1, 2, 3 and 5

Copyright © 2013, 2009, 2003, 1999, 1995, 1990, 1982, 1977, 1973, 1969 by Mosby, an imprint of Elsevier Inc.
Test bank 41-65

b. 1, 2, 3, 4, and 5
c. 3 and 5
d. 1 and 4

ANS: B
Pneumonia is the second most common nosocomial infection, primarily affecting infants and
young children, adults older than 65 years, persons with severe underlying disease, and those
who are immunosuppressed, have depressed sensorium or cardiopulmonary disease, or have had
thoracoabdominal surgery.

87. During ventilatory support, peak inspiratory pressure (PIP) is the pressure needed to overcome
which of the following?

1. chest wall compliance


2. lung compliance
3. airway resistance
4. systemic arterial pressure
a. 1 and 2 only
b. 2 and 3 only
c. 1, 2, and 3 only
d. 2, 3, and 4 only

ANS: C
During positive pressure mechanical ventilation, peak inspiratory pressure (PIP) is the pressure
necessary to overcome airway resistance and lung and chest wall compliance.

88. The respiratory therapist has been called to place a 70 kg male patient with ARDS on ventilatory
support. The physician has requested a respiratory rate of 20/min. Which of the following would
be an appropriate VT for this patient?
a. 140 mL
b. 200 mL
c. 350 mL
d. 700 mL

ANS: C
The currently acceptable tidal volume for a mechanically ventilated patient with ARDS in acute
respiratory failure is 4 to 8 mL/kg. Therefore the VT must be set between 280 mL (70 kg x 4
mL/kg) to 560 mL (70 kg x 8 mL/kg).

89. Patients receiving mechanical ventilation are usually turned every two hours to prevent which of
the following?
1. atelectasis
2. secretion retention
3. pressure sores
4. hyperoxemia

Copyright © 2013, 2009, 2003, 1999, 1995, 1990, 1982, 1977, 1973, 1969 by Mosby, an imprint of Elsevier Inc.
Test bank 41-66

a. 1 and 2 only
b. 2 and 3 only
c. 1, 2 and 3 only
d. 1, 2, 3, and 4

ANS: C
Patients receiving mechanical ventilation are turned frequently, usually a minimum of every 2
hours, unless turning is contraindicated. Turning mechanically ventilated patients help prevent
atelectasis, hypoxemia, secretion retention, and pressure sores.

90. Which of the following is the recommended tidal volume for mechanical ventilation in patients
with COPD?
a. 4 to 8 mL/kg
b. 3 to 5 mL/kg
c. 6 to 8 mL/kg
d. 10 to 12 mL/kg

ANS: C
The currently acceptable tidal volume for mechanically ventilated patients in acute respiratory
failure with normal lungs or with COPD is 6 to 8 mL/kg.

91. During pressure-controlled continuous mandatory ventilation, when the patient’s lung
compliance increases, which of the following will occur?
a. The tidal volume will increase.
b. The FRC will increase.
c. The peak airway pressure will increase.
d. The inspiratory time will decrease.

ANS: A

Copyright © 2013, 2009, 2003, 1999, 1995, 1990, 1982, 1977, 1973, 1969 by Mosby, an imprint of Elsevier Inc.
Test bank 41-67

Chapter 44: Initiating and Adjusting Invasive Ventilatory Support

Test Bank

1. Which of the following is the LEAST frequent cause of acute respiratory failure needing
mechanical ventilation?
a. sepsis
b. postoperative respiratory failure
c. heart failure
d. aspiration

ANS: D
The most common causes of acute respiratory failure that necessitate mechanical ventilation are
listed in Table 44-2.

DIF: Recall REF: p. 1090 OBJ: 1

2. Which of the following clinical findings is LEAST likely to be seen in a patient with acute
hypoxic respiratory failure?
a. confusion
b. tachycardia
c. hypotension
d. dyspnea

ANS: C
Clinical manifestations of acute hypoxemia and acute ventilatory failure are listed in Table 44-6.

DIF: Recall REF: p. 1109 OBJ: 1

3. Which of the following findings would you expect to see in a patient who has acute ventilatory
failure with severe hypercapnia?
a. jugular vein distension
b. pale, dry skin
c. bradycardia
d. hyperresponsiveness and dilated pupils

ANS: C
Clinical manifestations of acute hypoxemia and acute ventilatory failure are listed in Table 44-6.

DIF: Recall REF: p. 1109 OBJ: 2

4. Physiological goals of artificial ventilatory support include which of the following?

Copyright © 2013, 2009, 2003, 1999, 1995, 1990, 1982, 1977, 1973, 1969 by Mosby, an imprint of Elsevier Inc.
Test bank 41-68

1. support or manipulate gas exchange


2. reduce or manipulate the work of breathing
3. increase lung volume
a. 1 and 2
b. 2 and 3
c. 1 and 3
d. 1, 2, and 3

ANS: D
The goals of mechanical ventilatory support are to maintain adequate alveolar ventilation and
oxygen delivery, restore acid-base balance, and reduce the work of breathing with minimum
harmful side effects and complications.

DIF: Application REF: p. 1089 OBJ: 2

5. After starting volume-cycled mechanical ventilation on a patient in respiratory failure with a V T


of 10 ml/kg, you measure and obtain a plateau pressure of 45 cm H2O. Which of the following
actions would you recommend to the patient’s physician?
a. Decrease the inspiratory flow.
b. Lower the delivered VT.
c. Administer a bronchodilator.
d. Add PEEP.

ANS: B
Plateau pressure (Pplat) during mechanical ventilation reflects alveolar pressure, the best bedside
clinical reflection of transalveolar pressure. Limiting Pplat reduces the likelihood of ventilator-
induced lung injury, although patients with decreased thoracic compliance may require plateau
pressures greater than 30 cm H2O without resulting overdistention.

DIF: Application REF: p. 1089 OBJ: 2

6. All the following are hazards associated with mechanical ventilation except:
a. reduced cardiac output
b. liver failure
c. increased work of breathing
d. acute lung injury

ANS: B
Hazards of mechanical ventilation include decreased venous return and cardiac output, increased
work of breathing and ventilatory muscle dysfunction due to inappropriate ventilator settings,
and ventilator-induced lung injury. Nosocomial pneumonia poses a significant risk for intubated
patients.

DIF: Recall REF: p. 1090 OBJ: 2

Copyright © 2013, 2009, 2003, 1999, 1995, 1990, 1982, 1977, 1973, 1969 by Mosby, an imprint of Elsevier Inc.
Test bank 41-69

7. What goal(s) does the practitioner hope to achieve when selecting initial ventilatory support
settings?
1. Optimize oxygenation.
2. Optimize ventilation.
3. Maintain acid-base balance.
4. Avoid harmful side effects.
a. 1 only
b. 2 and 3 only
c. 1, 3, and 4
d. 1, 2, 3, and 4

ANS: D
In the selection of initial ventilator settings, the goal is to optimize the patient’s oxygenation,
ventilation, and acid-base balance while avoiding harmful side effects.

DIF: Recall REF: p. 1090 OBJ: 3

8. To stabilize a patient during the initial application of ventilatory support, which of the following
parameters must be set?
1. airway temperature
2. ventilatory support mode
3. oxygen concentration (FIO2)
4. minute ventilation (f, VT)
a. 1 and 3
b. 1, 2, and 3
c. 2, 3, and 4
d. 1, 2, 3, and 4

ANS: B
Box 44-3 summarizes key decisions that must be made as a part of initial ventilator setup.

DIF: Recall REF: p. 1090 OBJ: 3

9. All of the following are advantages of -Assist Control Volume ventilation, except:
a. Minimal safe level of ventilation achieved
b. Patient can set breathing rate
c. May reduce work of breathing
d. Pressure is limited

ANS: D
Advantages of assist-control volume ventilation include the assurance that a minimum safe level
of ventilation is achieved, yet the patient can still set his or her own breathing rate. In the event
of sedation or apnea, a minimum safe level of ventilation is guaranteed by the selection of an
appropriate backup rate, usually approximately 4 to 6 breaths/min less than the patient’s assist
rate but not less than the rate

Copyright © 2013, 2009, 2003, 1999, 1995, 1990, 1982, 1977, 1973, 1969 by Mosby, an imprint of Elsevier Inc.
Test bank 41-70

necessary to provide a minimum safe level of ventilation (e.g., a backup rate of at least 12 to 14
breaths/min). Because assist-control ventilation usually provides full ventilatory support, it may
result in less WOB. In volume control ventilation, pressure is variable and not limited.

DIF: Recall REF: p. 1092 OBJ: 3

10. Which of the following is an advantage of Pressure Control Ventilation?


a. Higher mean airway pressure can decrease venous return
b. VT varies depending on lung compliance, resistance, and patient effort
c. Improved gas distribution allows for lower VT
d. If VT or minute ventilation alarms are not set properly,
alveolar hypoventilation and acidosis may not be detected

ANS: C
Box 44-5 summarizes the Advantages and Disadvantages of Pressure Control Ventialtion.

DIF: Recall REF: p. 1094 OBJ: 3

11. In which of the following clinical conditions is noninvasive ventilation (NIV) NOT
recommended?
a. management of acute exacerbation of chronic obstructive pulmonary disease
(COPD)
b. management of premature extubation
c. management of cardiogenic pulmonary edema
d. management of acute respiratory distress syndrome (ARDS)

ANS: D
Most clinicians consider HFOV and conventional ventilation the best ways to manage ARDS
who require mechanical ventilation.

DIF: Recall REF: p. 1096 OBJ: 3

12. All of the following are specific clinical objectives of ventilatory support, except:
a. to reverse hypoxemia
b. to prevent or reverse atelectasis
c. to prevent sedation and neuromuscular blockade
d. to reverse acute respiratory acidosis

ANS: C
The specific clinical objectives of ventilatory support are listed in Box 44-2.

DIF: Application REF: p. 1091 OBJ: 3

13. What percentage of mechanical ventilated patients has a tracheostomy tube place at some point?
a. 5 - 10%
b. 10 - 15%

Copyright © 2013, 2009, 2003, 1999, 1995, 1990, 1982, 1977, 1973, 1969 by Mosby, an imprint of Elsevier Inc.
Test bank 41-71

c. 20 - 25%
d. 30- 35%

ANS: A
Approximately 5 -10% of patients receiving mechanical ventilation have a tracheotomy
performed at some point.

DIF: Recall REF: p. 1091 OBJ: 4

14. Which of the following modes of ventilatory support would you recommend for a patient who
can breathe spontaneously and only requires assistance to overcome the work of breathing
created by the ET tube ?
a. pressure-targeted continuous mandatory ventilation (CMV)
b. pressure-supported ventilation (PSV)
c. volume-targeted CMV
d. pressure-targeted intermittent mandatory ventilation

ANS: B
PSV can reduce work of breathing and may improve patient ventilator synchrony by placing
more control with the patient. Many clinicians use PSV simply to overcome WOB imposed by
the artificial airway

DIF: Application REF: p. 1095 OBJ: 4

15. Compared to a pressure-controlled strategy, what is the primary advantage of volume-controlled


ventilatory support?
a. provides a decelerating flow pattern
b. limits and controls peak airway pressures
c. improves patient–ventilator synchrony
d. guarantees a minimum minute ventialtion

ANS: D
The primary advantage of volume-controlled ventilation is maintenance of a stable minute
ventilation in the face of changing lung mechanics

DIF: Recall REF: p. 1092 OBJ: 4

16. Compared with a volume-cycled strategy, what are some potential advantages of pressure-
targeted ventilatory support?
1. limit and control of peak airway pressures
2. direct control over inspiratory time
3. provision of a decelerating flow pattern
a. 1 and 2
b. 2 and 3
c. 1 and 3
d. 1, 2, and 3

Copyright © 2013, 2009, 2003, 1999, 1995, 1990, 1982, 1977, 1973, 1969 by Mosby, an imprint of Elsevier Inc.
Test bank 41-72

ANS: C
Pressure-control ventilation is useful in limiting airway pressure and providing a decreasing
(decelerating) flow, which may improve gas distribution, patient comfort, and synchrony.

DIF: Recall REF: p. 1094 OBJ: 4

17. What is the recommended range for the tidal volume for the initial ventilator settings in the
volume control mode?
a. 4 to 6 ml/kg
b. 6 to 8 ml/kg
c. 10 to 12 ml/kg
d. 12 to 15 ml/kg

ANS: B
Tidal volume of 6 to 8 ml/kg of ideal body weight.

DIF: Recall REF: p. 1092 OBJ: 4

18. If the patient is being ventilated via a mechanical ventilator via synchronized intermittent
mandatory ventilation with partial ventilatory support, what would probably happen to PaCO 2 if
the patient suddenly had no spontaneous breathing?
a. Increase
b. Decrease
c. stay the same
d. change according to FIO2

ANS: A
With partial ventilatory support, if spontaneous breathing ceases or becomes inadequate, as may
be the case with the development of rapid shallow breathing or apnea, alveolar ventilation may
decrease, and PaCO2 may increase above an acceptable level.

DIF: Analysis REF: p. 1123 OBJ: 5

19. Which of the following represents a clinical situation where partial ventilatory support is
commonly used?
a. patient with head trauma
b. during weaning from continuous mandatory ventilation
c. while ventilating an asthmatic
d. in a drug overdose case

ANS: B
Partial ventilatory support techniques may be especially useful for weaning patients from
mechanical ventilatory support, and pressure-supported ventilation (PSV) and synchronized
intermittent mandatory ventilation have been used as partial support strategies for weaning.

Copyright © 2013, 2009, 2003, 1999, 1995, 1990, 1982, 1977, 1973, 1969 by Mosby, an imprint of Elsevier Inc.
Test bank 41-73

DIF: Application REF: p. 1092 OBJ: 5

20. Which of the following situations is most likely to call for ventilator settings of low volume and
high rate while allowing for permissive hypercapnia?
a. patient with ARDS
b. patient with neuromuscular disease
c. patient with chronic obstructive pulmonary disease
d. child with croup

ANS: A
Ventilation strategies for lung protection in ARDS include a low VT, rapid respiratory rates, and
permissive hypercapnia if necessary to avoid overdistention or Pplat >28 cm H2O.

DIF: Application REF: p. 1125 OBJ: 5

21. When a patient is initially started on mechanical ventilation common orders from the physician
in the patient’s chart include all the following except:
a. FIO2
b. Mode
c. Sensitivity
d. tidal volume

ANS: C
Initial ventilator settings include choice of mode, tidal volume, rate, FIO 2, and PEEP. The
respiratory therapist must set the trigger level, inspiratory flow or time, alarms and limits, backup
ventilation, and humidification.

DIF: Recall REF: p. 1091 OBJ: 5

22. Which of the following is one of the modes of ventilation that may be considered when partial
ventilatory support is indicated?
a. assist-control pressure ventilation
b. proportional assist ventilation (PAV)
c. volume-control continuous mandatory ventilation
d. assist-control volume ventilation

ANS: B
Normally, when partial ventilatory support is indicated, SIMV, PSV, volume support, PAV, and
NAVA are the modes of choice.

DIF: Recall REF: p. 1092 OBJ: 5

23. Which of the following statements is NOT true regarding ventilation is the assist-control mode?
a. Every breath is supported by the ventilator.
b. Usually ensures a minimum safe level of ventilation is given.
c. Assist-control mode is typically applied using the volume control mode.

Copyright © 2013, 2009, 2003, 1999, 1995, 1990, 1982, 1977, 1973, 1969 by Mosby, an imprint of Elsevier Inc.
Test bank 41-74

d. It is usually applied with a backup rate of 5 to 8 breaths/min.

ANS: D
With assist-control mode, every breath is supported by the ventilator. Breaths are patient- or
time-triggered to inspiration and may be volume or pressure limited. Inspiration may be volume,
pressure, or time cycled to the expiratory phase. Assist-control ventilation typically is delivered
as volume-controlled (VC) continuous mandatory ventilation. Suggested initial settings for
assist-control volume ventilation in the care of adults are listed in Box 44-5.
Advantages of assist-control volume ventilation include the assurance that a minimum, safe level
of ventilation is achieved. Every breath is a volume breath, yet the patient can set his or her own
breathing rate. In the event of sedation or apnea, a minimum, safe level of ventilation is
guaranteed by the selection of an appropriate backup rate, usually approximately 2 to 4
breaths/min below the patient’s assist rate, but not less than the rate necessary to provide a
minimum safe level of ventilation (e.g., a backup rate of at least 8 to 10 breaths/min, depending
on tidal volume set). Because assist-control ventilation usually provides full ventilatory support,
it may result in a lower work of breathing than partial support modes.

DIF: Application REF: p. 1092 OBJ: 5

24. What will most likely result if the patient ventilated in the assist-control mode develops a high
ventilatory drive?
a. respiratory alkalosis
b. respiratory acidosis
c. ventilatory failure
d. large drop in oxygenation

ANS: A
Patients with a high ventilatory drive may trigger the ventilator at a rapid rate that results in
hyperventilation and respiratory alkalosis.

DIF: Application REF: p. 1124 OBJ: 6

25. Which of the following remedies is NOT commonly used in patients ventilated in the assist-
control mode with a high ventilatory drive to avoid hyperventilation?
a. synchronized intermittent mandatory ventilation
b. Tranquilizers
c. Analgesics
d. reduced inspiratory flow

ANS: D
If methods of correcting the problem fail, one may try the synchronized intermittent mandatory
ventilation mode, which should be helpful in preventing respiratory alkalosis. Other options
include administration of analgesics, sedatives, and tranquilizers.

DIF: Recall REF: p. 1124 OBJ: 6

Copyright © 2013, 2009, 2003, 1999, 1995, 1990, 1982, 1977, 1973, 1969 by Mosby, an imprint of Elsevier Inc.
Test bank 41-75

26. Air-trapping is a major concern in patients with what diagnosis when using the assist-control
mode?
a. Pneumonia
b. chronic obstructive pulmonary disease (COPD)
c. chest trauma
d. neuromuscular disease

ANS: B
Patients with COPD are at special risk of air trapping in the assist-control mode, especially if
they attempt to breathe at an increased rate.

DIF: Recall REF: p. 1104 OBJ: 6

27. Which of the following statements is NOT true regarding the use of controlled ventilation?
a. may allow the muscles of breathing to rest
b. can use larger 1:E ratio and may improve oxygenation
c. requires use of paralytic agents in spontaneously breathing patients
d. therapist has little control of needed inspiratory flow and pressure

ANS: D
Advantages of controlled ventilation include eliminating the work of breathing and complete
control over the patient’s inspiratory and expiratory time, flow, and pressure. In cases in which
the work of breathing is high, controlled ventilation may allow for ventilatory muscle rest,
reduce the oxygen consumption of the ventilatory muscles, and “free up” oxygen for delivery to
the tissues. Controlled ventilation may allow prolonged inspiratory times and the use of 1:E
ratios greater than 1:1 in cases in which other methods have failed to improve oxygenation.
Disadvantages of controlled ventilation include the need for sedatives and perhaps paralytic
drugs in the care of patients with spontaneous breathing efforts. The administration of paralytic
agents has been associated with the development of prolonged neuropathy in some patients.
Paralytic agents have no effect on the patient’s level of consciousness and should not be given
without concurrent and appropriate sedation. In addition, in the care of apneic patients, ventilator
malfunction or disconnection can lead to death of the patient.

DIF: Recall REF: p. 1094 OBJ: 6

28. Which of the following statements is NOT true regarding the use of intermittent mandatory
ventilation (IMV) or synchronized intermittent mandatory ventilation (SIMV)?
a. The machine breath is typically time cycled to inspiration.
b. The patient can breathe spontaneously between machine breaths.
c. SIMV is easy to apply.
d. SIMV may help maintain ventilatory muscle strength.

ANS: A

Copyright © 2013, 2009, 2003, 1999, 1995, 1990, 1982, 1977, 1973, 1969 by Mosby, an imprint of Elsevier Inc.
Test bank 41-76

IMV and SIMV may be used as partial or full ventilatory support techniques. Patients breathe
spontaneously between mandatory machine breaths. The machine breath may be time cycled
(IMV) or patient triggered (SIMV) to inspiration. The inspiratory phase usually is delivered with
a specific inspiratory flow waveform, which may be adjustable.
With SIMV, the machine breath is typically volume cycled to expiration; however, the breath
may also be pressure limited and time cycled on most ventilators. SIMV often is combined with
pressure support to overcome the WOBI during spontaneous breathing due to demand flow
systems, ventilator circuits, and artificial airways. SIMV allows the clinician to vary the amount
of support provided from minimal to full ventilatory support. Further, SIMV is easy to apply.
SIMV also may be slightly better than assist-control ventilation in preventing hyperventilation
and respiratory alkalosis. SIMV may provide lower mean airway pressure than assist-control
ventilation and may help maintain ventilatory muscle strength and coordination. Disadvantages
of SIMV include the possible development of respiratory muscle dysfunction, especially in
patients with rapid shallow spontaneous breathing patterns; acute hypoventilation with use of
low rates if patients do not continue to do their share of breathing; and an increase in work of
breathing due to the demand flow system, ventilator circuit, and artificial airway or due to
inappropriate trigger sensitivity or ventilator peak flow settings. There is also evidence that
SIMV may delay weaning compared to spontaneous breathing trials or pressure support. The
advantages and disadvantages of assist-control and SIMV modes are found in Table 44-5.

DIF: Recall REF: p. 1095 OBJ: 6

29. Which of the following statements is NOT true regarding pressure-supported ventilation?
a. It is patient triggered, pressure limited, and flow cycled.
b. It can reduce the work of breathing during intermittent mandatory ventilation
mode.
c. The usual range is 10 to 15 cm H2O.
d. It is recommended for use in most patients in the synchronized intermittent
mandatory ventilation (SIMV) mode.

ANS: C
Pressure support ventilation assists the patient’s spontaneous inspiration with a clinician-selected
level of positive pressure. Pressure support is patient triggered, pressure limited, and flow cycled.
Pressure-support ventilation can reduce work of breathing and may improve patientventilator
synchrony and comfort while limiting the pressure applied to the airway. Pressure support to
overcome WOBI should be considered in the care of all spontaneously breathing patients in the
SIMV mode. SIMV, as originally developed, allowed spontaneous breathing interspersed with
volume-limited, patient- or time-triggered machine breaths. The WOBI during spontaneous
breathing due to slow demand flow systems, ventilator circuitry, and the artificial airway
prompted the addition of pressure support. Today it is suggested that all spontaneous breaths
during SIMV be supported with an appropriate level of pressure support, usually in the range of
5 to 10 cm H2O, to overcome the WOBI.

DIF: Recall REF: p. 1095 OBJ: 6

30. In what scenario is pressure-controlled ventilation (PCV) most often used?

Copyright © 2013, 2009, 2003, 1999, 1995, 1990, 1982, 1977, 1973, 1969 by Mosby, an imprint of Elsevier Inc.
Test bank 41-77

a. when limiting plateau pressure is needed


b. when a pneumothorax is present
c. when the patient has chronic obstructive pulmonary disease
d. when bilateral pneumonia is present

ANS: A
PCV may be used immediately upon ventilator initiation when limiting the plateau pressure is a
concern and in the care of patients expected to need prolonged inspiration or an increased 1:E
ratio (1:1, 1.5:1, 2:1). These patients typically have acute lung injury or ARDS.

DIF: Recall REF: p. 1094 OBJ: 6

31. All of the following are used as alternative lung protective strategies in patients with ARDS
except:
a. Prone positioning
b. ECMO
c. high-frequency ventilation
d. pressure-support ventilation

ANS: D
Alternative lung protective strategies in patients with ALI/ARDS include prone positioning,
ECMO, and high-frequency ventilation.

DIF: Application REF: p. 1127 OBJ: 6

32. What mode of ventilation is most often applied to the intensive care unit patient needing
mechanical ventilation?
a. pressure-supported ventilation
b. pressure-controlled ventilation
c. assist-control or synchronized intermittent mandatory ventilation (SIMV)
d. control mode

ANS: C
Most patients who need mechanical ventilation in the acute care setting initially are given
volume ventilation in the assist-control or SIMV with pressure support mode.

DIF: Recall REF: p. 1096 OBJ: 6

33. A physician orders intubation and mechanical ventilation in the synchronized intermittent
mandatory ventilation mode for a 170-lb adult man with neuromuscular disease. Which of the
following initial settings would you recommend?
Rate VT
a.
b.
c.
d.

Copyright © 2013, 2009, 2003, 1999, 1995, 1990, 1982, 1977, 1973, 1969 by Mosby, an imprint of Elsevier Inc.
Test bank 41-78

ANS: A
A larger tidal volume and lower rate (10 to 12 ml/kg and 8 to 12 breaths/min) may be considered
for patients with neuromuscular disease.

DIF: Analysis REF: p. 1099 OBJ: 6

34. A ventilator has separate rate and VT controls. If you set a VT of 850 ml and a respiratory rate of
12/min in the continuous mandatory ventilation mode, what will the minute ventilation be?
a. 7800 ml/min (7.8 L/min)
b. 8500 ml/min (8.5 L/min)
c. 9600 ml/min (9.6 L/min)
d. 10,200 ml/min (10.2 L/min)

ANS: D
Tidal volume (VT) and rate (f) determine minute ventilation (E).

DIF: Application REF: p. 1096 OBJ: 6

35. A ventilator has separate rate and minute ventilation controls. A physician orders continuous
mandatory ventilation with a VT of 950 ml at a respiratory rate of 12/min. What minute
ventilation would you set on this ventilator?
a. 7900 ml/min (7.9 L/min)
b. 8600 ml/min (8.6 L/min)
c. 9400 ml/min (9.4 L/min)
d. 11,400 ml/min (11.4 L/min)

ANS: D
Tidal volume (VT) and rate (f) determine minute ventilation (E).

DIF: Application REF: p. 1096 OBJ: 6

36. On a ventilator that has separate rate and minute ventilation (VE) controls, the rate is set at
13/min and the VE at 11 L/min. About what VT is the patient receiving?
a. 700 ml
b. 850 ml
c. 1000 ml
d. 1200 ml

ANS: B
E = f  VT.

DIF: Application REF: p. 1096 OBJ: 6

37. For adolescents in the 8- to 16-year-old age range, which of the following ranges of ventilator
setting would you initially recommend?

Copyright © 2013, 2009, 2003, 1999, 1995, 1990, 1982, 1977, 1973, 1969 by Mosby, an imprint of Elsevier Inc.
Test bank 41-79

Rate VT
a.
b.
c.
d.

ANS: D
Recommended initial tidal volume and frequency for various patient types are described in Table
44-7.

DIF: Application REF: p. 1118 OBJ: 6

38. A physician orders intubation and mechanical ventilation in the continuous mandatory
ventilation assist-control mode for a 125-lb adult woman with normal lungs. Which of the
following initial settings would you recommend?
Rate VT
a.
b.
c.
d.

ANS: B
See Table 44-7.

DIF: Application REF: p. 1118 OBJ: 6

39. A physician orders intubation and mechanical ventilation in the synchronized intermittent
mandatory ventilation mode for a 200-lb ideal body weight (IBW) adult man with normal lungs.
Which of the following initial ventilator settings would you recommend?
Rate VT
a.
b.
c.
d.

ANS: B
See Table 44-7.

DIF: Application REF: p. 1118 OBJ: 6

40. A physician orders intubation and mechanical ventilation in the synchronized intermittent
mandatory ventilation mode for a 160-lb adult man with a history of chronic obstructive
pulmonary disease. Which of the following settings would you recommend?
Rate VT
a.
b.

Copyright © 2013, 2009, 2003, 1999, 1995, 1990, 1982, 1977, 1973, 1969 by Mosby, an imprint of Elsevier Inc.
Test bank 41-80

c.
d.

ANS: A
See Table 44-7.

DIF: Analysis REF: p. 1118 OBJ: 6

41. On some ventilators, which of the following can occur if a trigger setting is set too sensitive on a
mechanical ventilator?
a. autotriggering
b. flow dyssynchrony
c. Barotrauma
d. increased workload

ANS: A
Trigger sensitivity for patient-triggered ventilation should be set at the lowest possible level to
minimize trigger work while avoiding ventilator autotriggering.

DIF: Recall REF: p. 1097 OBJ: 6

42. Which of the following trigger levels is appropriate when setting a ventilator for pressure-
triggering?
a. 0.5 to 1.5 cm H2O above the baseline pressure
b. 1.5 to 2.5 cm H2O below the baseline pressure
c. 0.5 to 1.5 cm H2O below the baseline pressure
d. 2.5 to 3.5 cm H2O below the baseline pressure

ANS: C
With pressure triggering, the range is generally –0.5 to –1.5 cm H2O.

DIF: Recall REF: p. 1098 OBJ: 6

43. Which of the following is FALSE about flow-triggered ventilatory support?


a. The work of breathing with flow-triggering is less than with pressure triggering.
b. Flow-triggered systems respond to changes in flow rather than pressure.
c. Pressure-triggering on new ventilators may be as sensitive as flow-triggering.
d. Flow-triggering will decrease the work of breathing in patients with small
endotracheal tubes and auto-PEEP.

ANS: D
Flow triggering may not be effective in reducing work of breathing because of the presence of a
small endotracheal tube or auto-PEEP.

DIF: Recall REF: p. 1098 OBJ: 6

Copyright © 2013, 2009, 2003, 1999, 1995, 1990, 1982, 1977, 1973, 1969 by Mosby, an imprint of Elsevier Inc.
Test bank 41-81

44. Which of the following is false about flow triggering of spontaneous breaths during mechanical
ventilation?
a. Flow triggering lowers the patient’s work of breathing.
b. Flow triggering is preferred for initiating spontaneous breaths.
c. Flow triggering reduces the work of breathing due to small endotracheal tubes.
d. Flow triggering results in better patientventilator synchrony.

ANS: C
Flow triggering may not be effective in reducing work of breathing because of the presence of a
small endotracheal tube or auto-PEEP.

DIF: Recall REF: p. 1098 OBJ: 6

45. Which of the following trigger levels is appropriate when setting a ventilator for flow-triggering?
a. 9 to 11 L/min below baseline flow
b. 7 to 9 L/min below baseline flow
c. 4 to 6 L/min below baseline flow
d. 1 to 3 L/min below baseline flow

ANS: D
In general, for flow triggering the trigger flow should be set 1 to 3L/min below baseline or bias
flow.

DIF: Recall REF: p. 1098 OBJ: 6

46. For adults with otherwise normal lungs who are receiving ventilatory support in the continuous
mandatory ventilation control or assist-control mode, inspiratory flow should be set to provide
what 1:E?
a. 2:1
b. 3:1
c. 1:1
d. 1:2

ANS: D
For most adults, an initial inspiratory time of approximately 1 second (0.8 to 1.2 seconds) with a
resultant 1:E ratio of 1:2 or lower is a good starting point.

DIF: Recall REF: p. 1099 OBJ: 6

47. When starting flow-limited ventilatory support on an adult patient, which of the following
inspiratory flow settings would you initially select?
a. 60 L/min
b. 50 L/min
c. 40 L/min
d. 30 L/min

Copyright © 2013, 2009, 2003, 1999, 1995, 1990, 1982, 1977, 1973, 1969 by Mosby, an imprint of Elsevier Inc.
Test bank 41-82

ANS: A
This value corresponds to an initial peak flow setting of approximately 60 L/min with a range of
40 to 80 L/min and a down ramp or square flow waveform.

DIF: Recall REF: p. 1099 OBJ: 6

48. A chronic obstructive pulmonary disease (COPD) patient receiving ventilatory support in the
CMV assist-control mode at a rate of 14 and a VT of 750 ml exhibits clinical signs of air-
trapping. Which of the following would you recommend to correct this problem?
1. Decrease “E” time.
2. Increase the inspiratory flow rate.
3. Decrease the assist-control rate.
a. 1 and 2
b. 1 and 3
c. 2 and 3
d. 1, 2, and 3

ANS: C
Higher flow (up to 100 L/min) may improve gas exchange in COPD patients, probably because
of the resulting increase in expiratory time.

DIF: Analysis REF: p. 1099 OBJ: 6

49. Beneficial effects of using high inspiratory flows in patients with chronic airflow obstruction
receiving flow-limited mechanical ventilation include which of the following?
1. decreased work of breathing
2. improved gas exchange
3. decreased auto-PEEP
a. 1 and 2
b. 1 and 3
c. 2 and 3
d. 1, 2, and 3

ANS: D
Higher flow (up to 100 L/min) may improve gas exchange in chronic obstructive pulmonary
disease patients, probably because of the resulting increase in expiratory time.

DIF: Recall REF: p. 1099 OBJ: 6

50. Which of the following ventilator adjustments would NOT decrease inspiratory time?
a. increase the peak flow
b. increase the tidal volume
c. change the flow pattern from a sine wave to a square wave
d. none of the above

ANS: B

Copyright © 2013, 2009, 2003, 1999, 1995, 1990, 1982, 1977, 1973, 1969 by Mosby, an imprint of Elsevier Inc.
Test bank 41-83

To decrease inspiratory time, one may increase the peak flow, decrease the tidal volume, or
change from a down ramp or sine wave to a square wave flow pattern.

DIF: Recall REF: p. 1100 OBJ: 6

51. Which of the following inspiratory flow patterns would result in the lowest peak inspiratory
pressure?
a. accelerating flow pattern
b. square flow pattern
c. decelerating flow pattern
d. constant flow pattern

ANS: C
However, it seems to be clear that as one moves from an increasing (“accelerating”) flow
waveform to a square wave to a decreasing flow waveform while holding inspiratory time
constant, there tends to be a predictable decrease in peak airway pressure and a corresponding
increase in mean airway pressure.

DIF: Recall REF: p. 1102 OBJ: 6

52. What flow pattern is least optimal for ventilating a patient with cardiovascular instability?
a. accelerating flow pattern
b. square flow pattern
c. decelerating flow pattern
d. constant flow pattern

ANS: C
A square or even accelerating waveform may be useful in reducing mean airway pressure in
patients with severe hypotension or cardiovascular instability.

DIF: Recall REF: p. 1102 OBJ: 6

53. Which of the following statements is NOT true regarding the use of an inspiratory pause during
mechanical ventilation?
a. It may be useful in ARDS patients.
b. It may be useful when obtaining a chest radiograph.
c. It has been shown to increase effectiveness of bronchodilator therapy.
d. It will increase mean airway pressure.

ANS: C

Copyright © 2013, 2009, 2003, 1999, 1995, 1990, 1982, 1977, 1973, 1969 by Mosby, an imprint of Elsevier Inc.
Test bank 41-84

The use of a single long inspiratory pause (up to 40 seconds) has been advocated in the
management of acute lung injury/ARDS as a lung recruitment maneuver, and the results have
been encouraging. Use of an inspiratory pause has been suggested for administration of
bronchodilators to improve medication delivery. In the treatment of chronic obstructive
pulmonary disease patients, however, a 5-second inspiratory pause did not result in significant
improvement in measures related to the effectiveness of the bronchodilator. If a brief inspiratory
pause is used, 1:E ratio and mean airway pressure increase. An inspiratory pause of 0.5 to 1.0
second applied for a single breath is used for measurement of plateau pressure (Pplat) and in
estimation of airway resistance (Raw)

where PIP is peak inspiratory pressure and is inspiratory peak flow (square wave).
An inspiratory pause can also be used to ensure a full inspiration before a chest radiograph is
obtained, and this step may improve the quality of the resulting radiograph.

DIF: Recall REF: p. 1116 OBJ: 6

54. Immediately after cardiac arrest and resuscitation, a patient is placed on a ventilator in the
continuous mandatory ventilation assist-control mode. What initial FIO2 would you recommend?
a. 1.0
b. 0.8
c. 0.6
d. 0.4

ANS: A
Examples of disease states or conditions that typically warrant an initial FIO 2 of 1.0 include
acute pulmonary edema, ARDS, near drowning, cardiac arrest, severe trauma, suspected
aspiration, severe pneumonia, carbon monoxide poisoning, and any disease state or condition
resulting in a large right-to-left shunt.

DIF: Recall REF: p. 1116 OBJ: 6

55. When adjusting the FIO2 setting for a patient receiving mechanical ventilatory support, what
should your goal be?
a. Decrease the FIO2 to below 0.70 as soon as possible.
b. Maintain the highest possible FIO2 as long as needed.
c. Decrease the FIO2 to below 0.30 as soon as possible.
d. Decrease the FIO2 to below 0.50 as soon as possible.

ANS: D
After initiation of mechanical ventilation with an FIO2 of 1.0, the FIO2 should be reduced to 0.40
to 0.50 or less as soon as is practical to avoid oxygen toxicity and absorption atelectasis.

DIF: Recall REF: p. 1116 OBJ: 6

Copyright © 2013, 2009, 2003, 1999, 1995, 1990, 1982, 1977, 1973, 1969 by Mosby, an imprint of Elsevier Inc.
Test bank 41-85

56. An adult patient in respiratory failure has the following blood gases on a nasal cannula at 5
L/min: pH = 7.20; PaCO2 = 67 mm Hg; HCO3= 27 mEq/L; PaO2 = 89 mm Hg. The attending
physician orders intubation and ventilatory support. What FIO2 would you recommend to start
with?
a. 0.21
b. 0.30
c. 0.50
d. 0.90

ANS: C
Patients who have undergone previous blood gas measurement or oximetry who are doing well
clinically and patients with disease states or conditions that normally respond to low to moderate
concentrations of oxygen may begin ventilation with 50 to 70% oxygen.

DIF: Application REF: p. 1104 OBJ: 6

57. In which of the following conditions is PEEP NOT likely to be useful?


a. ARDS
b. pulmonary edema
c. acute lung injury
d. neuromuscular disease

ANS: D
PEEP and continuous positive airway pressure are effective techniques for improving and
maintaining lung volume and improving oxygenation for patients with acute restrictive disease
such as acute lung injury, pneumonia, pulmonary edema, and ARDS.
A PaO2 less than 50 to 60 mm Hg with an FIO2 greater than 0.40 to 0.50 is a good general
starting place for considering use of PEEP or continuous positive airway pressure.

DIF: Recall REF: p. 1104 OBJ: 6

58. Which of the following criteria represents the recommended starting point for considering the
use of PEEP?
a. PaO2 < 40-50 on FIO2 > 0.80 – 1.0
b. PaO2 < 50-60 on FIO2 > 0.40-0.50
c. PaO2 < 100 on FIO2 of 1.0
d. PaO2 < 50 on FIO2 > 0.75

ANS: B DIF: Recall REF: p. 1104 OBJ: 6

59. To prevent atelectasis and improve gas exchange, most thoracic surgery patients placed on
ventilatory support receive which of the following?
a. 0 cm H2O PEEP
b. 5 cm H2O PEEP
c. 8 cm H2O PEEP

Copyright © 2013, 2009, 2003, 1999, 1995, 1990, 1982, 1977, 1973, 1969 by Mosby, an imprint of Elsevier Inc.
Test bank 41-86

d. 10 cm H2O PEEP

ANS: B
In terms of ventilator initiation, initial PEEP/CPAP levels usually are 5 cm H 2O.

DIF: Recall REF: p. 1104 OBJ: 6

60. In which of the following clinical situations is the incidence of auto-PEEP the greatest?
1. patients with high respiratory rates
2. intubated patients with obstructive lung disease
3. patients with low minute volumes
a. 1 and 2
b. 1 and 3
c. 2 and 3
d. 1, 2, and 3

ANS: A
PEEP has been advocated in small increments (2-3 cm H2O) for overcoming auto-PEEP,
particularly in the care of patients with obstructive lung disease.

DIF: Recall REF: p. 1118 OBJ: 6

61. A patient receiving continuous mandatory ventilation in the assist-control mode develops auto-
PEEP. Which of the following general approaches would you consider to minimize the effects of
auto-PEEP in this patient?
1. increasing expiratory time
2. applying PEEP
3. switching ventilating mode to synchronized intermittent mandatory ventilation
a. 1 and 2
b. 1 and 3
c. 2 and 3
d. 1, 2, and 3

ANS: C
Box 44-11summarizes methods for minimizing the effects of auto-PEEP.

DIF: Recall REF: p. 1105 OBJ: 6

62. A patient receiving continuous mandatory ventilation in the assist-control mode develops auto-
PEEP. Which of the following changes in ventilatory patterns would you consider to minimize
the effects of auto-PEEP in this patient?
1. decreasing the rate or increasing VT
2. using low-rate synchronized intermittent mandatory ventilation
3. decreasing the peak inspiratory flow
4. lowering the VT and letting the PaCO2 rise
a. 2 and 4

Copyright © 2013, 2009, 2003, 1999, 1995, 1990, 1982, 1977, 1973, 1969 by Mosby, an imprint of Elsevier Inc.
Test bank 41-87

b. 3 and 4
c. 1, 2, and 3
d. 1, 2, and 4

ANS: D
Box 44-11 summarizes methods for minimizing the effects of auto-PEEP.

DIF: Recall REF: p. 1105 OBJ: 6

63. A chronic obstructive pulmonary disease (COPD) patient in respiratory failure is receiving
ventilatory support in the volume-targeted intermittent mandatory ventilation mode at a rate of
6/min. You measure an auto-PEEP level of 9 cm H2O. Which of the following would you
recommend to decrease the effects of auto-PEEP in this patient?
a. Decreasing the rate and increasing VT.
b. Lowering the VT and letting the PaCO2 rise.
c. Applying 4 to 6 cm H2O PEEP.
d. Decreasing the peak inspiratory flow.

ANS: C
Box 44-11summarizes methods for minimizing the effects of auto-PEEP.

DIF: Analysis REF: p. 1105 OBJ: 6

64. When the therapist is initially setting the high-pressure alarm on the ventilator and the patient’s
plateau pressure is less than 30 cm H2O, what should the high pressure alarm be set at?
a. 5 to 10 cm H2O above the peak pressure
b. 10 to 20 cm H2O above the peak pressure
c. 10 to 12 cm H2O above the plateau pressure
d. 10 to 15 cm H2O above the mean airway pressure

ANS: B
If the plateau pressure is less than 30 cm H2O, the high pressure limit can be adjusted to 10 to 20
cm H2O above the peak inspiratory pressure.

DIF: Application REF: p. 1106 OBJ: 6

65. After placing a patient on a volume-cycled ventilator in the continuous mandatory ventilation
assist-control mode, you note that 55 cm H2O pressure is required to deliver the preset VT of 950
ml. What high-pressure limit would you now set for this patient?
a. 60 cm H2O
b. 70 cm H2O
c. 80 cm H2O
d. 90 cm H2O

ANS: B

Copyright © 2013, 2009, 2003, 1999, 1995, 1990, 1982, 1977, 1973, 1969 by Mosby, an imprint of Elsevier Inc.
Test bank 41-88

If the plateau pressure is less than 30 cm H2O, the high pressure limit can be adjusted to 10 to 20
cm H2O above the peak inspiratory pressure.

DIF: Analysis REF: p. 1106 OBJ: 6

66. If available, the FIO2 alarm should be set to what percentage?


a. ±3%
b. ±5%
c. ±8%
d. ±10%

ANS: B
Suggested initial settings for these alarms and backup ventilator settings are described in Table
44-8.

DIF: Recall REF: p. 1106 OBJ: 6

67. What limits should be initially set for high and low VT values and/or minute volume alarms on a
ventilatory support device?
a. ±5% to 10%
b. ±10% to 15%
c. ±15% to 20%
d. ±20% to 25%

ANS: B
Suggested initial settings for these alarms and backup ventilator settings are described in Table
44-8.

DIF: Recall REF: p. 1106 OBJ: 6

68. A heat-moisture exchanger (HME) should be avoided in which of the following circumstances?
1. patients with excessive secretions
2. patients with a high FIO2
3. patients with low body temperature
a. 1 only
b. 1 and 2 only
c. 1 and 3 only
d. 1, 2, and 3

ANS: C
Use of HMEs should be avoided in the care of patients with secretion problems and those with
low body temperature (less than 32° C), high spontaneous minute ventilation (greater than 10
L/min), or air leaks in which exhaled tidal volume is less than 70% of delivered tidal volume.

DIF: Recall REF: p. 1108 OBJ: 6

Copyright © 2013, 2009, 2003, 1999, 1995, 1990, 1982, 1977, 1973, 1969 by Mosby, an imprint of Elsevier Inc.
Test bank 41-89

69. Which of the following criteria should be met before considering use of a heat-moisture
exchanger (HME) for a patient being placed on ventilatory support?
1. There should be no problem with retained secretions.
2. The patient should not have a fever (normothermic).
3. The patient should be adequately hydrated.
4. The support should be short term (24 to 48 hours).
a. 1, 2, and 3
b. 2 and 4
c. 1, 2, 3, and 4
d. 3 and 4

ANS: C
Use of HMEs should be avoided in the care of patients with secretion problems and those with
low body temperature (less than 32° C), high spontaneous minute ventilation (greater than 10
L/min), or air leaks in which exhaled tidal volume is less than 70% of delivered tidal volume.

DIF: Recall REF: p. 1108 OBJ: 6

70. For which of the following patients requiring ventilatory support would you recommend against
using a heat-moisture exchanger (HME) for airway humidification?
1. patient whose expired VT is less than 70% of the delivered VT
2. patient with a spontaneous minute ventilation of 14 L/min
3. patient with body temperature less than 32° C
a. 1 and 2
b. 1 and 3
c. 2 and 3
d. 1, 2, and 3

ANS: D
Use of HMEs should be avoided in the care of patients with secretion problems and those with
low body temperature (less than 32° C), high spontaneous minute ventilation (greater than 10
L/min), or air leaks in which exhaled tidal volume is less than 70% of delivered tidal volume.

DIF: Recall REF: p. 1108 OBJ: 6

71. A dehydrated, feverish patient suffering from acute bacterial pneumonia is being intubated in
order to provide mechanical ventilatory support. Which of the following devices would you
select to control humidification and airway temperature for this patient?
a. unheated large-volume wick humidifier
b. heated wick humidifier with servo-control
c. large-reservoir, high-output heated jet nebulizer
d. heat-moisture exchanger

ANS: B

Copyright © 2013, 2009, 2003, 1999, 1995, 1990, 1982, 1977, 1973, 1969 by Mosby, an imprint of Elsevier Inc.
Test bank 41-90

We prefer an optimal humidity approach and use of a heated humidifier to deliver gas in the
range of 35° to 37° C at the airway.

DIF: Analysis REF: p. 1092 OBJ: 6

72. A patient suffering from postoperative complications has been receiving mechanical ventilation
for 6 days with a volume ventilator. A heat-moisture exchanger (HME) is providing control over
humidification and airway temperature. Over the past 24 hours, the patient’s secretions have
decreased in quantity but are thicker and more purulent. Which of the following actions would
you suggest at this time?
a. Replace the HME.
b. Switch over to a heated wick humidifier.
c. Administer acetylcysteine every 2 hours via the nebulizer.
d. Increase the frequency of suctioning.

ANS: B
We prefer an optimal humidity approach and use of a heated humidifier to deliver gas in the
range of 35° to 37° C at the airway.

DIF: Analysis REF: p. 1092 OBJ: 6

73. When using a heated humidifier during mechanical ventilation, the inspired gas temperature at
the airway should be set to what level?
a. 29° to 31° C
b. 31° to 35° C
c. 35° to 37° C
d. 38° to 40° C

ANS: C
We prefer an optimal humidity approach and use of a heated humidifier to deliver gas in the
range of 35° to 37° C at the airway.

DIF: Recall REF: p. 1092 OBJ: 6

74. Indications for delivering sigh breaths during mechanical ventilation include all of the following
except:
a. before and after suctioning
b. during chest physical therapy
c. in patients with stiff lungs
d. when small VT values are used

ANS: C
Constant, monotonous tidal ventilation at a small volume (<7 ml/kg) may result in progressive
atelectasis. Sighs may be used to prevent atelectasis. Atelectasis may be caused before and after
suctioning and when using small tidal volumes. CPT is also used when attempting to correct
atelectasis.

Copyright © 2013, 2009, 2003, 1999, 1995, 1990, 1982, 1977, 1973, 1969 by Mosby, an imprint of Elsevier Inc.
Test bank 41-91

DIF: Recall REF: p. 1107 OBJ: 6

75. Which of the following would you assess immediately after a patient is placed on a ventilatory
support device?
1. ABGs
2. patient’s airway
3. patient’s vital signs
a. 1 and 2
b. 1 and 3
c. 2 and 3
d. 1, 2, and 3

ANS: D
Initial patient evaluation should include physical assessment, assessment of ventilator settings,
cardiovascular assessment, oximetry, and measurement of arterial blood gases (Box 44-16).

DIF: Recall REF: p. 1107 OBJ: 6

76. All of the following machine factors can have a major impact on adult patient–ventilator
interaction except:
a. humidification system
b. PEEP valve function
c. trigger sensitivity
d. tubing compliance

ANS: D
Factors that may affect patientventilator interaction are listed in Box 44-17.

DIF: Recall REF: p. 1108 OBJ: 6

77. Which of the following factors influence patient–ventilator interaction?


1. artificial airway
2. trigger sensitivity
3. presence of auto-PEEP
a. 1 and 2
b. 1 and 3
c. 2 and 3
d. 1, 2, and 3

ANS: D
Factors that may affect patientventilator interaction are listed in Box 44-17.

DIF: Recall REF: p. 1108 OBJ: 6

Copyright © 2013, 2009, 2003, 1999, 1995, 1990, 1982, 1977, 1973, 1969 by Mosby, an imprint of Elsevier Inc.
Test bank 41-92

78. Which of the following would you initially verify in assessing the airway of a patient placed on
ventilatory support?
1. cuff pressure
2. tube position
3. tube patency
a. 1 and 2
b. 1, 2, and 3
c. 1 and 3
d. 2 and 3

ANS: B
The artificial airway should be assessed for proper placement, patency, and cuff inflation. Size,
position, and depth of the endotracheal tube and cuff pressure, including volume used to inflate
the cuff, should be recorded.

DIF: Recall REF: p. 1108 OBJ: 6

79. After setting up a patient on a ventilatory support device, which of the following supplementary
equipment would you require to be available at the bedside?
1. suction source and catheters
2. backup artificial airway
3. manual resuscitator with oxygen
a. 2 and 3
b. 1 and 2
c. 1, 2, and 3
d. 1 and 3

ANS: C
An extra endotracheal tube or tracheostomy tube of the correct size should be placed at the
patient’s bedside, and the equipment needed to replace the airway must be available and easily
accessible. A clean, functioning manual resuscitator with oxygen supply and suction equipment,
including an appropriate supply of suction catheters, sterile water or saline solution, and sterile
gloves also must be placed near the bedside.

DIF: Recall REF: p. 1108 OBJ: 6

80. You have just placed a chronic obstructive pulmonary disease (COPD) patient on intermittent
mandatory ventilation at a rate of 8/min, a VT of 750 ml, and an FIO2 of 0.40. To ensure proper
equilibration between the alveolar and arterial gas tensions, how long should you wait before
drawing a sample for measurement of the ABG?
a. 5 minutes
b. 10 minutes
c. 15 minutes
d. 30 minutes

ANS: D

Copyright © 2013, 2009, 2003, 1999, 1995, 1990, 1982, 1977, 1973, 1969 by Mosby, an imprint of Elsevier Inc.
Test bank 41-93

ABGs should be measured 20 to 30 minutes after initiation of mechanical ventilation.

DIF: Analysis REF: p. 1108 OBJ: 9

81. When adjusting a patient’s oxygenation during mechanical ventilatory support, what should your
goal be?
a. SaO2 of 80% to 90%
b. PaO2 of 100 to 150 mm Hg
c. SaO2 of 95% to 100%
d. PaO2 of 60 to 100 mm Hg

ANS: D
The FIO2 is then titrated to achieve a PaO2 in the range of 60 to 80 mm Hg with an SaO2 of 90%
or greater or an SpO2 of 92% or greater.

DIF: Recall REF: p. 1113 OBJ: 9

82. When titrating the FIO2 level downward from 100% to 40%, what is the maximum increment
that should be applied between estimates of oxygenation?
a. 5%
b. 10%
c. 20%
d. 25%

ANS: C
In either case, it is suggested that oxygen levels be titrated down from 100% to 50% oxygen in
decrements not to exceed 20%.

DIF: Recall REF: p. 1113 OBJ: 9

83. When titrating the FIO2 down from 50% to 21%, in what increments should it be reduced?
a. all at once is acceptable
b. no more than 5%
c. 5% to 10%
d. 10% to 20%

ANS: C
For reducing oxygen concentration from 50% to 21%, oxygen changes should be in steps of 5%
to 10% followed by oximetry or measurement of blood gases.

DIF: Recall REF: p. 1113 OBJ: 9

84. A patient with ARDS receiving ventilatory support with PEEP through a volume-cycled
ventilator has a plateau pressure of 38 cm H2O. ABGs on 55% O2 are as follows: pH = 7.44;
PCO2 = 37 mm Hg; HCO3= 25 mEq; PO2 = 55 mm Hg; SaO2 = 88%. Which of the following
would you recommend?

Copyright © 2013, 2009, 2003, 1999, 1995, 1990, 1982, 1977, 1973, 1969 by Mosby, an imprint of Elsevier Inc.
Test bank 41-94

a. Increase the PEEP level.


b. Make no changes.
c. Reduce the VT.
d. Increase the FIO2.

ANS: B
An SaO2 of 88% to 90% may be acceptable for patients who need an FIO2 of 0.80 or more for an
extended time.

DIF: Analysis REF: p. 1113 OBJ: 7

85. When is the PEEP/CPAP level optimum?


a. Oxygen delivery to the tissues is maximized.
b. Pressures are maintained below 15 cm H2O.
c. CaO2 – CvO2 is maximized.
d. The PaO2 is 60 to 100 mm Hg.

ANS: A
Optimal or best PEEP may be defined as the PEEP that maximizes oxygen delivery (DO 2).

DIF: Recall REF: p. 1114 OBJ: 7

86. The following data are gathered during a PEEP study (FIO2 = 0.60). Based on these data, what is
the optimum PEEP level?
PEEP cm H2O 0 5 10 15 20 25
PaO2 mm Hg 46 54 67 73 75 74
Compliance ml/cm H2O 18 23 26 30 24 19
Systolic pressure 125 123 114 115 104 94
Diastolic pressure 90 88 83 84 76 68

a. 10 cm H2O
b. 15 cm H2O
c. 20 cm H2O
d. 25 cm H2O

ANS: B
Best PEEP has been exceeded at the point when an increase in PEEP is followed by a decrease in
compliance.

DIF: Application REF: p. 1115 OBJ: 7

87. When using a pressure-volume curve to identify optimal PEEP levels, what does the upper
inflection point represent?
a. point of lung recruitment
b. point of lung over distension
c. optimal PEEP level

Copyright © 2013, 2009, 2003, 1999, 1995, 1990, 1982, 1977, 1973, 1969 by Mosby, an imprint of Elsevier Inc.
Test bank 41-95

d. optimal compliance level

ANS: B
The upper inflection point may indicate lung overdistention.

DIF: Recall REF: p. 1115 OBJ: 7

88. When performing a lung recruitment strategy, which of the following would cause the therapist
to stop?
1. Mean blood pressure drops of 80 to 65 mm Hg.
2. Heart rate increases from 88 to 110/min.
3. Patient has a run of premature ventricular complexes.
a. 1 only
b. 2 only
c. 3 only
d. 1, 2, and 3

ANS: C
The recruitment maneuver is stopped if there is a decrease in SpO2 to less than 88%, a significant
change in heart rate (greater than 140 beats/min or less than 60 beats/min), a significant change
in mean arterial blood pressure (less than 60 mm Hg or decrease greater than 20 mm Hg from
baseline) or the development of cardiac arrhythmia.

DIF: Recall REF: p. 1118 OBJ: 7

89. When the patient stabilizes on mechanical ventilation with a PEEP of 12 cm H 2O and the FIO2
has been reduced to 0.40, how should the PEEP level reduced?
a. in increments of 2 cm H2O every 6 hours
b. in increments of 3 to 5 cm H2O every 2 hours
c. in increments of 3 to 5 cm H2O every one hour
d. in increments of 5 cm H2O every 2 hours

ANS: A
After reduction of the FIO2 to 0.40, PEEP can be reduced gradually as the patient improves at a
rate of 2 cm H2O every 6 to 8 hours.

DIF: Recall REF: p. 1127 OBJ: 8

90. What is the recommended response to a drop in PaO2 when the PEEP level is reduced in a
mechanically ventilated patient?
a. Increase the FIO2.
b. Return the PEEP to the previous level.
c. Increase the rate of mechanical breaths.
d. Do nothing.

ANS: B

Copyright © 2013, 2009, 2003, 1999, 1995, 1990, 1982, 1977, 1973, 1969 by Mosby, an imprint of Elsevier Inc.
Test bank 41-96

If the PaO2 decreases after PEEP is decreased the PEEP level should be returned to its prior
setting.

DIF: Recall REF: p. 1104 OBJ: 8

91. Which of the following techniques can be used to improve oxygenation beyond increasing the
FIO2 or PEEP level?
1. proning the patient
2. use of an expiratory pause
3. use of inverse 1:E ratio ventilation
a. 1 only
b. 1 and 2 only
c. 2 and 3 only
d. 1 and 3 only

ANS: D
Other techniques that may be helpful in improving arterial oxygen levels include the use of PCV
with a prolonged inspiratory time, use of an inspiratory pause, inverse 1:E ratio ventilation, and
prone positioning.

DIF: Recall REF: p. 1119-1120 OBJ: 8

92. In what clinical condition has pressure-controlled ventilation with a prolonged inspiratory time
been shown to be helpful?
a. ARDS/acute lung injury
b. pulmonary embolism
c. bilateral pneumonia
d. severe pulmonary fibrosis

ANS: A
Pressure-control ventilation with prolonged inspiratory time has been associated with
improvement in PaO2 in patients with acute lung injury/ARDS.

DIF: Recall REF: p. 1116 OBJ: 8

93. How frequently should a clinician make changes when weaning from PEEP?
a. 10 to 20 minutes
b. 30 to 45 minutes
c. 1 hour
d. 6- 8 hours

ANS: D
Generally, PEEP is sustained at the set level until FiO2 is less than 0.5, and when PEEP is
decreased, it should be decreased in increments of 2 cm H2O no more frequently than about
every 6 to 8 hours.

Copyright © 2013, 2009, 2003, 1999, 1995, 1990, 1982, 1977, 1973, 1969 by Mosby, an imprint of Elsevier Inc.
Test bank 41-97

DIF: Recall REF: p. 1127 OBJ: 8

94. What is the primary concern when using proning to improve oxygenation in the patient with
ARDS?
a. sudden increase in PaCO2
b. displacement of tubes and lines
c. pneumothorax
d. hemodynamic compromise

ANS: B
Care must be taken to ensure that endotracheal tubes, intravenous lines, and catheters are not
blocked or dislodged.

DIF: Recall REF: p. 1121 OBJ: 7

95. What is considered to be the single best indicator of effective ventilation?


a. PaO2
b. SaO2
c. PaCO2
d. pH

ANS: C
Arterial PaCO2 is considered the single best index of effective ventilation.

DIF: Recall REF: p. 1121 OBJ: 9

96. Your patient develops a fever while being mechanically ventilated in the control mode. As a
result of the fever, the patient’s CO2 production increases while alveolar ventilation is
unchanged. What is the probable change in ABGs?
a. increase in PaCO2
b. decrease in PaO2
c. decrease in PaCO2
d. all the above

ANS: A
Increases in A or decreases in CO2 result in a decrease in PaCO2, whereas increases in CO2
or decreases in A result in an increase in PaCO2.

DIF: Application REF: p. 1096 OBJ: 9

97. A patient receiving control-mode continuous mandatory ventilation has the following ABGs on
an FIO2 of 0.4: pH = 7.51; PCO2 = 30 mm Hg; HCO3= 25 mm Hg. Her current minute
ventilation (VE) is 7.9 L/min. What new VE would you recommend?
a. 9.0 L/min
b. 6.7 L/min

Copyright © 2013, 2009, 2003, 1999, 1995, 1990, 1982, 1977, 1973, 1969 by Mosby, an imprint of Elsevier Inc.
Test bank 41-98

c. 7.5 L/min
d. 5.9 L/min

ANS: D
Box 44-20 gives an example of the effect of a change in A on PaCO2.

DIF: Analysis REF: p. 1122 OBJ: 9

98. A patient receiving control-mode continuous mandatory ventilation has the following ABGs on
an FIO2 of 0.5: pH = 7.23; PCO2 = 61 mm Hg; HCO3= 26 mm Hg. The current minute
ventilation (VE) is 9.2 L/min. What new VE would you recommend?
a. 10.6 L/min
b. 14.0 L/min
c. 12.4 L/min
d. 5.8 L/min

ANS: B
Box 44-20 gives an example of the effect of a change in A on PaCO2.

DIF: Analysis REF: p. 1122 OBJ: 9

99. During initial mechanical ventilation of the chronic obstructive pulmonary disease (COPD)
patient with chronic hypercapnia, what PaCO2 is most likely used as a target value?
a. 30 mm Hg
b. 40 mm Hg
c. 55 mm Hg
d. 75 mm Hg

ANS: C
In the care of patients with acute exacerbation of COPD and accompanying chronic ventilatory
failure, the clinician may target ventilatory support to achieve the patient’s “normal” PaCO 2 and
pH. For COPD patients with chronic hypercapnia, this may mean a target PaCO2 of 50 to 60 mm
Hg with a pH of 7.30 to 7.35.

DIF: Recall REF: p. 1121 OBJ: 9

100. When setting the tidal volume on a patient being mechanically ventilated, what criteria should be
kept in mind?
a. It should never cause the plateau pressure to exceed 28 mm Hg.
b. It should never cause the peak pressure to exceed 35 mm Hg.
c. It should result in the static pressure of less than 10 mm Hg.
d. It should result in a peak pressure of no more than 25 mm Hg.

ANS: A
Tidal volume usually is based on specific patient considerations but should ideally never result in
a plateau pressure of 28 cm H2O or greater.

Copyright © 2013, 2009, 2003, 1999, 1995, 1990, 1982, 1977, 1973, 1969 by Mosby, an imprint of Elsevier Inc.
Test bank 41-99

DIF: Recall REF: p. 1106 OBJ: 9

101. What is the predicted change in tidal volume by adding 6 inches of tubing to a ventilator circuit?
a. decrease of 50 to 70 ml
b. decrease of 30 to 50 ml
c. decrease of 20 to 30 ml
d. no change

ANS: A
In general, Vt reduces between 50 – 70 ml for each 6 inches (15 cm) of dead space added by
tubing in a circuit.

DIF: Recall REF: p. 1122 OBJ: 9

102. Which of the following would decrease PaCO2 when ventilating a patient using synchronized
intermittent mandatory ventilation with pressure support?
a. increase the level of pressure support.
b. Decrease the tidal volume.
c. Decrease the mechanical rate.
d. Increase the FIO2.

ANS: A
The PaCO2 can be decreased by increasing tidal volume, increasing PSV for spontaneous
breaths, or increasing the machine rate.

DIF: Recall REF: p. 1124 OBJ: 9

103. What phrase is used to describe the situation where the patient with acute lung injury is
ventilated with a smaller tidal volume and the PaCO2 is allowed to increase above normal range
to avoid additional lung injury?
a. physiologic ventilation
b. permissive hypercapnia
c. adjusted ventilation
d. dialed acidosis

ANS: B
This technique is known as permissive hypercapnia.

DIF: Recall REF: p. 1125 OBJ: 9

104. Patient’s RR is 12 breaths/min, PaCO2 is 60 mm Hg. If a PaCO2 of 40 were desired, the RR be


set at what value?
a. 18
b. 23
c. 35

Copyright © 2013, 2009, 2003, 1999, 1995, 1990, 1982, 1977, 1973, 1969 by Mosby, an imprint of Elsevier Inc.
Test bank 41-100

d. 40

ANS: B
This formula predicts what RR adjustment affects will be on PaCO2, with no change in VCO2 or
VDphys:

Initial Desired
PaCO2(1)  f(1) = PaCO2(2)  f(2)

60  12 = 40  f(2) = Adjust rate to 18 breaths/min

DIF: Application REF: p. 1122 OBJ: 6

105. All of the following primary goals of mechanical ventilation, except:


a. Adequate alveolar ventilation (VA)
b. Maintain adequate hemoglobin levels
c. Restore acid-base balance
d. Maintain adequate alveolar oxygenation

ANS: B
Primary goals of mechanical ventilation are: adequate alveolar ventilation (V A), maintain tissue
oxygenation, FIO2, PEEP, and MAP, Restore acid-base balance, reduce WOB and myocardial
work, provide PEEP/CPAP to recruit lung, lung protective strategy: Small VT and appropriate
PEEP levels and maintain Pplat < 30 cm H2O.

DIF: Recall REF: p. 1089 OBJ: 6

Copyright © 2013, 2009, 2003, 1999, 1995, 1990, 1982, 1977, 1973, 1969 by Mosby, an imprint of Elsevier Inc.
Test bank 41-101

Chapter 45: Noninvasive Ventilation

1. Which of the following devices is considered to be the first electrically powered negative
pressure ventilator?
a. the Bird
b. the iron lung
c. the chest cuirass
d. the rocking bed

ANS: B
The first electrically powered, negative-pressure ventilator was known as the “iron lung.”

DIF: Recall REF: p. 1132 OBJ: 5

2. Why did the use of intermittent positive-pressure breathing declined in the 1980s?
a. due to a lack of scientific evidence to support its use for delivering aerosolized
medication
b. due to its cost of implementation
c. due to its complexity
d. replaced by newer technology

ANS: A
The use of intermittent positive-pressure breathing declined significantly in the mid 1980s after a
randomized, controlled trial revealed no benefit, compared to a simple small-volume nebulizer,
in the treatment of patients with chronic obstructive pulmonary disease.

DIF: Recall REF: p. 1133 OBJ: 5

3. All of the following are goals of noninvasive ventilation (NIV) in the acute care setting, except:
a. Avoid intubation.
b. Decrease incidence of ventilation-associated pneumonia.
c. Decrease length of stay.
d. Improve mobility.

ANS: D
Avoiding intubation and invasive positive-pressure ventilation, improving survival, decreasing
the length of ventilatory support, decreasing the length of hospitalization, and decreasing the
incidence of ventilator associated pneumonia are major goals of NIV in the acute care setting
(emergency department, intensive care unit, or hospital ward).

DIF: Recall REF: p. 1134 OBJ: 1

4. Which of the following therapies should be considered as first line of therapy in patients with
exacerbation of chronic obstructive pulmonary disease (COPD)?

Copyright © 2013, 2009, 2003, 1999, 1995, 1990, 1982, 1977, 1973, 1969 by Mosby, an imprint of Elsevier Inc.
Test bank 41-102

a. noninvasive ventilation (NIV)


b. mechanical ventilation
c. high-flow nasal cannula
d. systemic steroids

ANS: A
The differences in reported outcome between these studies suggests that NIV should be
considered standard of care for COPD patients in an acute exacerbation and offered as first-line
therapy to patients in all institutions treating the COPD patient.

5. Your patient has acute pulmonary edema from left heart failure. Which therapy should be tried
first?
a. noninvasive ventilation (NIV)
b. continuous positive airway pressure (CPAP)
c. mechanical ventilation
d. positive end-expiratory pressure

ANS: B
CPAP is considered by most first-line therapy for acute cardiogenic pulmonary edema.

DIF: Application REF: p. 1134 OBJ: 5

6. Which of the following noninvasive ventilation (NIV) settings are adequate for a patient with
cardiogenic pulmonary edema?
a. continuous positive airway pressure (CPAP) at 10 to 15 cm H2O with 100%
oxygen
b. CPAP at 10 to 15 cm H2O with 50% oxygen
c. CPAP at 8 to 12 cm H2O with 100% oxygen
d. CPAP at 8 to 12 cm H2O with 50% oxygen

ANS: C
CPAP should be administered at 8 to 12 cm H2O with 100% oxygen.

DIF: Application REF: p. 1135 OBJ: 5

7. Which of the following indications for noninvasive ventilation (NIV) is where the greatest
controversy exists?
a. chronic obstructive pulmonary disease
b. pneumonia
c. hypoxemic respiratory failure
d. nocturnal hypoventilation

ANS: C
This is the indication for NIV where the greatest controversy exists.

Copyright © 2013, 2009, 2003, 1999, 1995, 1990, 1982, 1977, 1973, 1969 by Mosby, an imprint of Elsevier Inc.
Test bank 41-103

DIF: Recall REF: p. 1136 OBJ: 1

8. All of the following are benefits of continuous positive airway pressure (CPAP) in postoperative
major abdominal surgery, except:
a. lower mortality
b. lower intubation rate
c. lower incidence of pneumonia
d. lower rate of sepsis

ANS: A
The CPAP group had a lower rate of intubation, pneumonia, overall infection, and sepsis.

DIF: Recall REF: p. 1137 OBJ: 1

9. All of the following are potential benefits of using noninvasive ventilation (NIV) during weaning
except reduced:
a. length of intensive care unit (ICU) stay
b. incidence of nosocomial pneumonia
c. mortality rate
d. incidence of pulmonary embolism

ANS: D
In one study, it was found that NIV reduced weaning time, length of ICU stay, incidence of
nosocomial pneumonia, and 60-day mortality compared with conventional weaning with
invasive pressure support ventilation.

DIF: Recall REF: p. 1135 OBJ: 2

10. All of the following groups of patients are considered at risk for reintubation, except:
a. patients with APACHE 2 score greater than 12 on the day of extubation
b. patients older than 45 years
c. patients with congestive heart failure (CHF)
d. patients with one or more weaning failure

ANS: B
Overall, patients at risk were defined as those with chronic obstructive pulmonary disease
(COPD) or CHF, ineffective cough and excessive secretions, one or more weaning failure, one or
more comorbid condition, upper airway obstruction, age older than 65 years, and APACHE 2
score of greater than 12 on the day of extubation.

DIF: Recall REF: p. 1134 OBJ: 1

11. Which of the following restrictive thoracic diseases are successfully managed with noninvasive
ventilation (NIV)?
1. postpolio syndrome
2. neuromuscular disease

Copyright © 2013, 2009, 2003, 1999, 1995, 1990, 1982, 1977, 1973, 1969 by Mosby, an imprint of Elsevier Inc.
Test bank 41-104

3. spinal cord injuries


4. severe kyphoscoliosis
a. 1, 2, and 4
b. 1, 2, 3, and 4
c. 2 and 4
d. 2, 3, and 4

ANS: B
Restrictive thoracic diseases successfully managed with NIV include postpolio syndrome,
neuromuscular diseases, chest wall deformities, spinal cord injuries, and severe kyphoscoliosis.

DIF: Recall REF: p. 1138 OBJ: 1

12. How does noninvasive ventilation (NIV) benefit the patient with restrictive thoracic disease?
1. It rests the respiratory muscles.
2. It lowers the PaCO2.
3. It improves lung compliance.
4. It improves pulmonary function testing.
a. 1, 2, and 3
b. 2 and 4
c. 1, 2, and 4
d. 1, 3, and 4

ANS: A
The first mechanism is the ability of NIV to rest the respiratory muscles. Second, NIV lowers the
PaCO2, and the decrease is believed to reset the central ventilatory controller and establish a new
baseline PaCO2. The third mechanism is the improvement in lung compliance, lung volume, and
dead space that result from NIV.

DIF: Recall REF: p. 1138 OBJ: 1

13. The use of noninvasive ventilation (NIV) in the long-term care of patients with chronic
obstructive pulmonary disease (COPD) will benefit the patient in all the following ways except:
a. better gas exchange
b. improved sleep quality
c. resetting the respiratory center to better respond to hypoxemia
d. unloading the respiratory muscles

ANS: C
First, positive inspiratory pressure may improve gas exchange and unload the respiratory
muscles, allowing these muscles to recover, gain strength, and reduce fatigue. These benefits
should reduce symptoms associated with hypoventilation and improve quality of life. Second,
patients with severe COPD have poor sleep quality, shorter sleep time, and nocturnal
hypoventilation.

DIF: Recall REF: p. 1139 OBJ: 1

Copyright © 2013, 2009, 2003, 1999, 1995, 1990, 1982, 1977, 1973, 1969 by Mosby, an imprint of Elsevier Inc.
Test bank 41-105

14. Which of the following groups of patients with nocturnal hypoventilation respond to noninvasive
ventilation (NIV)?
a. hypoxic
b. hypercarbic
c. acidotic
d. hypocarbic

ANS: B
Patients with hypercarbia and nocturnal desaturation may be most likely to benefit from
nocturnal NIV.

DIF: Recall REF: p. 1139 OBJ: 3

15. Which of the following are indications to use noninvasive ventilation (NIV) in patients who have
chronic obstructive pulmonary disease (COPD) with nocturnal hypoventilation?
1. PaCO2 greater than 55 mm Hg
2. more than two hospitalizations related to hypercapnic respiratory failure
3. PaCO2 greater than 65 mm Hg
4. PaCO2 between 50 and 54 mm Hg with nocturnal desaturation
a. 1, 2, and 3
b. 2 and 4
c. 1, 2, and 4
d. 1, 3, and 4

ANS: C
The current recommendation from a consensus conference is to use NIV in the care of patients
with severe COPD who have symptoms of nocturnal hypoventilation and one of the following:
PaCO2 of 55 mm Hg or greater, a PaCO2 between 50 and 54 mm Hg with nocturnal desaturation,
or more than two hospital admissions related to hypercapnic respiratory failure.

DIF: Recall REF: p. 1139 OBJ: 2

16. Which of the following is least likely to indicate the need for noninvasive ventilation (NIV) in
the acute care setting?
a. paradoxical breathing
b. jugular venous distention
c. respiratory rate more than 25/min
d. use of accessory muscles

ANS: B
Signs and symptoms of respiratory distress include use of accessory muscles, paradoxical
breathing, a respiratory rate of 25 breaths/min or greater, and the presence of moderate to severe
dyspnea.

DIF: Recall REF: p. 1140 OBJ: 2

Copyright © 2013, 2009, 2003, 1999, 1995, 1990, 1982, 1977, 1973, 1969 by Mosby, an imprint of Elsevier Inc.
Test bank 41-106

17. All of the following findings are exclusion criteria for using noninvasive ventilation (NIV) in the
patient with acute respiratory failure, except:
a. apnea
b. hemodynamic or cardiac instability
c. low risk of aspiration
d. lack of cooperation by the patient

ANS: C
Exclusion criteria include apnea, hemodynamic or cardiac instability, lack of cooperation by the
patient, facial burns, facial trauma, copious amounts of secretions, high risk of aspiration, and
anatomic abnormalities that interfere with gas delivery.

DIF: Recall REF: p. 1140 OBJ: 2

18. All of the following predict successful use of noninvasive ventilation (NIV) in the respiratory
failure patient, except:
a. minimal air leak
b. low severity of illness
c. improvement in gas exchange within 30 minutes
d. pH = 7.20

ANS: D
See Box 45-5.

DIF: Recall REF: p. 1141 OBJ: 3

19. Your patient is being ventilated with a nasal mask to relieve dyspnea. He has a long history of
chronic obstructive pulmonary disease and hypercarbia. What is the goal of noninvasive
ventilation in this setting with regard to the ABGs?
a. Return the PaCO2 to 40 to 45 mm Hg.
b. Return the pH to near normal.
c. Return the PaCO2 to less than 60 mm Hg.
d. Return the bicarbonate level to near normal.

ANS: C
See Box 45-5.

DIF: Application REF: p. 1141 OBJ: 2

20. Which of the following are contraindications for the use of noninvasive ventilation (NIV)?
1. nonsupportive family
2. lack of financial resources
3. copious amounts of secretions
4. uncooperative behavior on the part of the patient
a. 1, 2, and 3

Copyright © 2013, 2009, 2003, 1999, 1995, 1990, 1982, 1977, 1973, 1969 by Mosby, an imprint of Elsevier Inc.
Test bank 41-107

b. 2 and 4
c. 1, 2, and 4
d. 1, 2, 3, and 4

ANS: D
Relative contraindications for the use of NIV for restrictive thoracic disease, nocturnal
hypoventilation, and chronic obstructive pulmonary disease include an nonsupportive family,
lack of financial resources, required ventilator assistance for most of the day, copious amounts of
secretions, uncooperative behavior on the part of the patient, high risk of aspiration, and any
anatomic abnormality that interferes with gas delivery.

DIF: Recall REF: p. 1141 OBJ: 2

21. Which of the following interfaces is most commonly used to apply noninvasive ventilation
(NIV) in the acute setting?
1. nasal or full face mask
2. mouthpiece
3. endotracheal tube
4. nasal pillows
a. 1 and 2
b. 1 and 3
c. 1, 2, and 4
d. 1 only

ANS: A
Full-face and nasal masks are the most commonly used interfaces in the acute care setting.

DIF: Recall REF: p. 1145 OBJ: 5

22. Which of the following is a potential risk of overtightening the straps of the mask?
a. absence of an air leak
b. tissue necrosis
c. eye irritation
d. claustrophobia

ANS: B
Caution must be taken not to overtighten the straps on the mask, because excessive pressure on
the bridge of the nose can cause tissue necrosis.

DIF: Recall REF: p. 1154 OBJ: 4

23. Compared with nasal masks, full-face masks are associated with all of the following, except:
a. increase in dead space
b. risk of aspiration
c. claustrophobia
d. hypocapnia

Copyright © 2013, 2009, 2003, 1999, 1995, 1990, 1982, 1977, 1973, 1969 by Mosby, an imprint of Elsevier Inc.
Test bank 41-108

ANS: D
Compared with nasal masks, full-face masks are associated with an increase in dead space, risk
of aspiration, and claustrophobia.

DIF: Recall REF: p. 1143 OBJ: 4

24. Which of the following interfaces should be used in greater than 90% of the patients with
hypoventilation?
a. full-face mask
b. nasal mask
c. nasal pillows
d. oral mask

ANS: A
If the problem is ventilation, the full-face mask is the interface of choice and should be used
initially in greater than 90% of patients requiring noninvasive positive-pressure ventilation for
acute respiratory failure.

DIF: Recall REF: p. 1145 OBJ: 4

25. Which of the following interfaces appear to be more efficient to improve ventilation?
1. nasal pillows
2. full-face mask
3. nasal mask
4. oral mask
a. 1 only
b. 1 and 2
c. 1, 2, 3, and 4
d. 1 and 4

ANS: B
The investigators reported that the full-face mask and nasal pillows improved ventilation more
than the nasal mask but that the nasal mask was better tolerated.

DIF: Recall REF: p. 1145 OBJ: 4

26. Which of the following interfaces that improve ventilation appears to be more tolerated?
a. nasal pillows
b. full-face mask
c. nasal mask
d. oral mask

ANS: A
The investigators reported that the full-face mask and nasal pillows improved ventilation more
than the nasal mask but that the nasal mask was better tolerated.

Copyright © 2013, 2009, 2003, 1999, 1995, 1990, 1982, 1977, 1973, 1969 by Mosby, an imprint of Elsevier Inc.
Test bank 41-109

DIF: Recall REF: p. 1145 OBJ: 4

27. All of the following ventilators are used for noninvasive ventilation (NIV), except:
a. critical care
b. portable
c. noninvasive
d. negative pressure

ANS: D
They include noninvasive ventilators, critical care ventilators, and portable volume ventilators.

DIF: Recall REF: p. 1145 OBJ: 5

28. Which of the following ventilators is generally only used for chronic noninvasive ventilation
(NIV)?
a. critical care
b. portable
c. noninvasive
d. negative pressure

ANS: B
However, portable volume ventilators are generally used only for chronic NIV.

DIF: Recall REF: p. 1150 OBJ: 5

29. Which of the following are characteristics of most noninvasive ventilators?


1. electrically powered
2. blower driven
3. microprocessor controlled
4. double-circuit design
a. 1 only
b. 1, 2, and 3
c. 1, 2, 3, and 4
d. 1 and 4

ANS: B
Most noninvasive ventilators are electrically powered, blower driven, and microprocessor
controlled (Figure 45-13).

DIF: Recall REF: p. 1149 OBJ: 5

30. Which of the following is required for noninvasive ventilators to work properly?
a. exhalation valve
b. pressure alarm
c. leak

Copyright © 2013, 2009, 2003, 1999, 1995, 1990, 1982, 1977, 1973, 1969 by Mosby, an imprint of Elsevier Inc.
Test bank 41-110

d. blender

ANS: C
A small leak is required in the circuit or patient interface for these ventilators to work properly.

DIF: Recall REF: p. 1143 OBJ: 5

31. Which of the following is the most important advantage of noninvasive ventilators over other
types of ventilators?
a. cost
b. ability to trigger and cycle appropriately when small-to-moderate air leaks are
present
c. ability to remove CO2
d. ability to oxygenate

ANS: B
The most important advantage of noninvasive ventilators over other types of ventilators is the
ability to trigger and cycle appropriately when small to moderate-size air leaks are present.

DIF: Recall REF: p. 1145 OBJ: 5

32. Which of the following are the minimum performing characteristics of most noninvasive
ventilators?
1. mandatory rate of 50 breaths/min or less
2. inspiratory pressure of 30 cm H2O or less
3. PEEP of 15 cm H2O or less
4. inspiratory flow of 180 L/min or less at 20 cm H2O
a. 1 only
b. 1, 2, and 3
c. 1, 2, 3, and 4
d. 2, 3, and 4

ANS: D
Noninvasive positive pressure ventilators should provide a mandatory rate of 30 breaths/min or
less, inspiratory pressure of 30 cm H2O or less, positive end-expiratory pressure (PEEP or end
positive airway pressure [EPAP]) of 15 cm H2O or less, inspiratory flow of 180 L/min or less at
20 cm H2O, ideally an FIO2 from 0.21 to about 1.0, minimal rebreathing potential, and
antiasphyxia capabilities.

DIF: Recall REF: p. 1146 OBJ: 6

33. All of the following are a required ventilator alarm for noninvasive ventilation (NIV), except:
a. loss of power
b. circuit disconnect
c. battery failure
d. blender alarm

Copyright © 2013, 2009, 2003, 1999, 1995, 1990, 1982, 1977, 1973, 1969 by Mosby, an imprint of Elsevier Inc.
Test bank 41-111

ANS: D
The ventilator must have alarms for circuit disconnect, loss of power, and battery failure if a
battery is present.

DIF: Recall REF: p. 1146 OBJ: 6

34. Which level of positive end-expiratory pressure (PEEP) is necessary to prevent rebreathing of
carbon dioxide?
a. 1 to 3 cm H2O
b. 3 to 5 cm H2O
c. 5 to 7 cm H2O
d. 7 to 9 cm H2O

ANS: B
These reports suggest the use of 3 to 5 cm H2O PEEP or the use of a nonrebreathing valve to
prevent rebreathing of carbon dioxide.

DIF: Recall REF: p. 1146 OBJ: 6

35. All of the following modes are commonly seen on noninvasive ventilators, except:
a. continuous positive airway pressure (CPAP)
b. spontaneous (pressure assist)
c. pressure assist
d. volume control

ANS: D
Modes on noninvasive ventilators usually include CPAP, pressure support (spontaneous), and
pressure assist/control (timed).

DIF: Recall REF: p. 1146 OBJ: 6

36. What mode of ventilation is most often used for noninvasive ventilation (NIV) when a critical
care ventilator is in use?
a. pressure support ventilation (PSV)
b. continuous positive airway pressure
c. synchronized intermittent mandatory ventilation
d. control

ANS: A
The mode of ventilation most used for NIV on intensive care unit ventilators is PSV.

DIF: Recall REF: p. 1146 OBJ: 7

Copyright © 2013, 2009, 2003, 1999, 1995, 1990, 1982, 1977, 1973, 1969 by Mosby, an imprint of Elsevier Inc.
Test bank 41-112

37. Your patient is being ventilated with a common critical care ventilator using pressure support
ventilation (flow cycled) with a nasal mask. A leak is present that is preventing the appropriate
termination of the inspiratory cycle. What is the best response?
a. Switch to volume control mode.
b. Switch to time-cycled mode.
c. Switch to nasal pillows.
d. Switch to full face mask.

ANS: B
Time-cycled (instead of flow-cycled), pressure-limited ventilation in the presence of air leaks
markedly improves patientventilator synchrony and patient comfort.

DIF: Analysis REF: p. 1148 OBJ: 7

38. All of the following statements are true about the use of home care ventilators for delivering
noninvasive ventilation (NIV), except:
a. They operate only with pressure triggering.
b. They can operate on AC or DC power sources.
c. They can have a single-limb ventilator circuit.
d. Flow delivery pattern can be adjusted.

ANS: D
In some, flow delivery is limited to a sine wave flow pattern.

DIF: Recall REF: p. 1150 OBJ: 5

39. What strategy should be used when the patient complains of nasal congestion during the use of a
nasal mask for noninvasive ventilation (NIV)?
a. Switch to an oral mask.
b. Switch to a face mask.
c. Add a heated humidifier.
d. Reduce the inspiratory flow.

ANS: C
The application of heated humidity relieves nasal resistance and congestion.

DIF: Application REF: p. 1150 OBJ: 8

40. Which of the following is the current recommendation for adding humidity while using
noninvasive ventilation (NIV)?
a. It is never recommended.
b. It is recommended for short-term application (less than 1 day).
c. It is recommended for long-term application (longer than 1 day).
d. It is always recommended.

ANS: C

Copyright © 2013, 2009, 2003, 1999, 1995, 1990, 1982, 1977, 1973, 1969 by Mosby, an imprint of Elsevier Inc.
Test bank 41-113

Because the upper airway is not bypassed during NIV, the current recommendation is no
humidity for short-term applications (less than 1 day).

DIF: Recall REF: p. 1150 OBJ: 8

41. What is the recommended initial setting for positive end-expiratory pressure (PEEP) when
delivering noninvasive ventilation in the pressure triggered timed mode?
a. 2 to 5 cm H2O
b. 5 to 8 cm H2O
c. 8 to 12 cm H2O
d. 12 to 20 cm H2O

ANS: B
The vast majority of patients only require PEEP levels of 5 to 8 cm H2O.

DIF: Recall REF: p. 1151 OBJ: 6

42. What is the recommended initial setting for ventilating pressure when delivering noninvasive
ventilation in the pressure triggered timed mode?
a. 2 to 5 cm H2O
b. 5 to 8 cm H2O
c. 8 to 12 cm H2O
d. 12 to 20 cm H2O

ANS: C
The majority of patients only require ventilating pressure of 8 to 12 cm H2O.

DIF: Recall REF: p. 1151 OBJ: 6

43. Which of the following define successful application of noninvasive ventilation (NIV)?
1. overall improvement of patient’s blood gas
2. normal blood gas
3. PaO2 increased
4. PaCO2 decreased
a. 1 only
b. 1, 2, and 3
c. 1, 2, 3, and 4
d. 1, 3, and 4

ANS: D
Successful application of NIV is easy to define: the patient’s blood gases improve, PaCO 2
decreases, and pH normalizes, while PaO2 increases.

DIF: Recall REF: p. 1151 OBJ: 7

Copyright © 2013, 2009, 2003, 1999, 1995, 1990, 1982, 1977, 1973, 1969 by Mosby, an imprint of Elsevier Inc.
Test bank 41-114

44. What physiologic effect will raising the expiratory positive airway pressure have in the patient
receiving noninvasive ventilation?
a. Increase the functional residual capacity.
b. Increase the tidal volume.
c. Decrease the PCO2.
d. Lower the mean airway pressure.

ANS: A
See Table 45-1.

DIF: Recall REF: p. 1152 OBJ: 8

45. Which of the following is likely to occur when decreasing the expiratory positive airway
pressure in the patient being ventilated using noninvasive ventilation?
a. improved PaO2
b. increased functional residual capacity
c. increased tidal volume
d. increased inspiratory positive airway pressure

ANS: C
See Table 45-1.

DIF: Recall REF: p. 1152 OBJ: 8

46. What is the best option for the patient in respiratory failure who continues to deteriorate 30
minutes after the initiation of noninvasive ventilation?
a. Wait another 30 minutes and monitor the patient.
b. Begin continuous positive airway pressure.
c. Intubate and begin mechanical ventilation.
d. Ventilate the patient using a bag-valve-mask.

ANS: C
If the patient’s vital signs and blood gas values are worsening after 30 minutes on optimal
settings, intubation should be considered.

DIF: Recall REF: p. 1151 OBJ: 8

47. Initiating noninvasive ventilation (NIV) can be done in the following settings, except:
a. emergency department
b. intensive care unit
c. hospital ward
d. nursing home

ANS: D
In the acute care setting, NIV can be initiated in the emergency department, critical care unit,
intermediate care unit, or hospital ward.

Copyright © 2013, 2009, 2003, 1999, 1995, 1990, 1982, 1977, 1973, 1969 by Mosby, an imprint of Elsevier Inc.
Test bank 41-115

DIF: Recall REF: p. 1153 OBJ: 7

48. What patient population is the only group currently accepted for the use of noninvasive
ventilation (NIV) in the hospital ward?
a. chronic obstructive pulmonary disease (COPD) patients with near normal pH
b. postoperative patients without pneumonia
c. no-code patients
d. asthma patients with normal SpO2

ANS: A
Currently, the only patient population for whom initiation and management of NIV on a hospital
ward are recommended are hypercapnic COPD patients with a pH of 7.30 or greater.

DIF: Recall REF: p. 1141 OBJ: 8

49. What complication associated with noninvasive ventilation (NIV) is most common?
a. aspiration
b. hypotension
c. nasal congestion
d. air leaks

ANS: D
See Table 45-2.

DIF: Recall REF: p. 1154 OBJ: 8

50. All of the following techniques are useful to avoid claustrophobia in the patient being ventilated
by face mask, except:
a. Allow patient to hold the mask.
b. Increase the inspiratory flow.
c. Use sedation.
d. Use a larger mask.

ANS: D
See Table 45-2.

DIF: Recall REF: p. 1154 OBJ: 8

51. Which of the following are possible solutions to correct a large air leak during noninvasive
ventilation?
1. Selecting an appropriately sized mask
2. Applying chin straps
3. Using a full-face mask
4. Decreasing the inspiratory time
a. 1 and 2 only

Copyright © 2013, 2009, 2003, 1999, 1995, 1990, 1982, 1977, 1973, 1969 by Mosby, an imprint of Elsevier Inc.
Test bank 41-116

b. 2 and 2 only
c. 1, 2, and 3 only
d. 1, 2, 3, and 4

ANS: C
Small air leaks should be expected during noninvasive ventilation. Large air leaks should be
addressed immediately. Air leaks can be avoided by selecting an appropriately sized mask,
changing to a full-face mask, repositioning the mask, readjusting the straps and adding a
forehead spacer.

DIF: Application REF: p. 1153 OBJ: 8

52. Which of the following are factors associated with pressure and flow-related complications
during noninvasive ventilation?
1. sinus and ear pain
2. nasal congestion
3. upper airway dryness
4. gastric insufflations
a. 1 and 2 only
b. 2 and 2 only
c. 1, 2, and 3 only
d. 1, 2, 3, and 4

ANS: D
Air pressure and flow-related complications include nasal congestion, upper airway dryness,
sinus and ear pain, eye irritation, and gastric insufflations.

DIF: Application REF: p. 1154 OBJ: 8

53. The respiratory therapist has placed a patient on noninvasive ventilation. After two hours the
patient showing significant improvement in her work of breathing but is experiencing nasal and
oral dryness. Which of the following corrective action should be performed?
a. Add heated humidifier.
b. Add ultrasonic nebulizer.
c. Immediately discontinue NIV.
d. Intubate the patient.

ANS: A
Heated humidity (about 30 degrees Celsius) should always be provided with NIV to avoid nasal
symptoms, the accumulation of secretions in the back of the oral pharynx and enhance patient
tolerance.

DIF: Recall REF: p. 1154 OBJ: 8

Copyright © 2013, 2009, 2003, 1999, 1995, 1990, 1982, 1977, 1973, 1969 by Mosby, an imprint of Elsevier Inc.
Test bank 41-117

Chapter 46: Monitoring the Patient in the Intensive Care Unit

1. While monitoring patients, signals, or values are susceptible to variability due to all of the
following, except:
a. Artifacts
b. factitious events
c. instrument drift
d. seasonal variation

ANS: D
Signals or values are susceptible to variability due to artifacts, factitious events, physiologic
variation, and instrument drift.

DIF: Recall REF: p. 1161 OBJ: 1

2. Temporary variation in pulmonary artery pressure readings due to movement of the


hemodynamic monitoring line is an example of what type of variability?
a. Artifact
b. factitious event
c. physiologic variation
d. instrument drift

ANS: A
Artifacts are frequently seen, for example, when the patient or monitoring lines are moved.

DIF: Application REF: p. 1161 OBJ: 1

3. You are monitoring blood pressure during mechanical ventilation of a patient with pneumonia. A
temporary increase in blood pressure occurs when the patient coughs. This temporary spike in
blood pressure represents what type of variability?
a. Artifact
b. factitious event
c. physiologic variation
d. instrument drift

ANS: C
The signal itself can exhibit a random variability related to the inherent imprecision of the signal
or due to normal physiologic variability in the patient. Blood pressure, for example, changes
within a certain range for many reasons.

DIF: Application REF: p. 1161 OBJ: 1

4. Which of the following are the reasons monitors are needed?


1. continuous assessment
2. analysis of vital signs

Copyright © 2013, 2009, 2003, 1999, 1995, 1990, 1982, 1977, 1973, 1969 by Mosby, an imprint of Elsevier Inc.
Test bank 41-118

3. measurement of values that caregivers cannot detect


a. 1 only
b. 1 and 2
c. 1 and 3
d. 1, 2, and 3

ANS: C
Monitors are needed for two main reasons: (1) continuous assessment (humans need breaks) and
(2) measurement of values that caregivers cannot detect such as ECG findings and airway
pressure.

DIF: Recall REF: p. 1162 OBJ: 1

5. All of the following parameters are major factors in determining tissue oxygenation, except:
a. arterial oxygenation
b. tissue perfusion
c. oxygen uptake
d. R/Q ratio

ANS: D
Tissue oxygenation depends on inspired oxygen levels (FIO2), inspired partial pressure of
oxygen (PIO2), alveolar oxygen tension (PAO2), arterial oxygenation (PaO2, SaO2, oxygen
content of arterial blood [CaO2]), oxygen delivery (DO2), tissue perfusion, and oxygen uptake.

DIF: Recall REF: p. 1162 OBJ: 4

6. In low-perfusion patients, what site would be best for monitoring SpO2?


a. Finger
b. Earlobe
c. Nose
d. Forehead

ANS: A
Finger probes appear more accurate than forehead, nose, or earlobe probes during low perfusion
states.

DIF: Recall REF: p. 1163 OBJ: 4

7. All of the following are likely to cause errors in SpO2 readings, except:
a. Anemia
b. deeply pigmented skin
c. motion due to shivering
d. significant tachycardia

ANS: D

Copyright © 2013, 2009, 2003, 1999, 1995, 1990, 1982, 1977, 1973, 1969 by Mosby, an imprint of Elsevier Inc.
Test bank 41-119

Intense daylight and fluorescent, incandescent, xenon, and infrared light sources have caused
errors in pulse oximetric readings. Anemia and deeply pigmented skin can affect the accuracy of
pulse oximetry; however, the effect of anemia is not clinically significant until the hemoglobin
level is markedly reduced.

DIF: Recall REF: p. 1163-1164 OBJ: 4

8. What is the normal approximate value for oxygen consumption?


a. 150 ml/min
b. 200 ml/min
c. 250 ml/min
d. 300 ml/min

ANS: C
Normal resting oxygen consumption is approximately 250 ml/min, and oxygen consumption
increases with activity, stress, and temperature.

DIF: Recall REF: p. 1164 OBJ: 4

9. What method(s) is/are used to measure oxygen consumption?


1. Fick method
2. analysis of inspired and expired gases
3. scans
a. 1 only
b. 1 and 2
c. 1 and 3
d. 1, 2, and 3

ANS: B
Two primary methods are in general use: the Fick method and analysis of inspired and expired
gases.

DIF: Recall REF: p. 1164 OBJ: 4

10. What is the normal range for P(Aa)O2 in a healthy 30-year-old person breathing room air?
a. 0 to 10 mm Hg
b. 5 to 15 mm Hg
c. 10 to 20 mm Hg
d. cannot predict

ANS: B
For example, a healthy person has a P(A–a)O2 of 5 to 15 mm Hg while breathing room air.

DIF: Recall REF: p. 1164 OBJ: 4

Copyright © 2013, 2009, 2003, 1999, 1995, 1990, 1982, 1977, 1973, 1969 by Mosby, an imprint of Elsevier Inc.
Test bank 41-120

11. Your patient has a P(A–a)O2 of 200 mm Hg while breathing 100% oxygen. What is the estimated
percentage shunt?
a. 5%
b. 10%
c. 20%
d. 30%

ANS: B
P(A - a)O2 increases to 100 to 150 mm Hg when the person is breathing 100%
oxygen.

DIF: Application REF: p. 1164 OBJ: 4

12. What is considered normal for the PaO2/FIO2 ratio?


a. greater than 50
b. greater than 150
c. greater than 250
d. greater than 400

ANS: D
A normal PaO2/FIO2 ratio while breathing room air is about 400 to 500 mm Hg.

DIF: Recall REF: p. 1165 OBJ: 4

13. What parameter is considered to be the most accurate and reliable measure of oxygenation
efficiency?
a. PaO2/FIO2 ratio
b. P(A – a)O2/PaO2 ratio
c.
d. PaO2/SaO2 ratio

ANS: C
The most accurate and reliable measure of oxygenation efficiency is direct computation of the
physiologic shunt .

DIF: Recall REF: p. 1165 OBJ: 4

14. All of the following parameters are used in calculating the Murray lung injury score of a patient
with acute lung injury, except:
a. results of chest radiograph
b. PEEP setting
c. lung compliance
d. cardiac output

ANS: D

Copyright © 2013, 2009, 2003, 1999, 1995, 1990, 1982, 1977, 1973, 1969 by Mosby, an imprint of Elsevier Inc.
Test bank 41-121

The Murray lung injury score quantifies the injury level using four factors: chest radiographic
findings, PaO2/FIO2 ratio, positive end-expiratory pressure (PEEP) setting, and compliance.

15. What is the best single measure of effective ventilation in the intensive care unit patient?
a. dead spacetidal volume ratio
b. PaCO2
c. end-tidal PCO2
d.

ANS: B
Because of the relationship between alveolar ventilation and PaCO2, the single best index of
effective ventilation is measurement of the PaCO2.

DIF: Recall REF: p. 1165 OBJ: 5

16. What is the best measure of the efficiency of gas exchange in the lung?
a. PaCO2
b. VDS/VT
c. end-tidal PCO2
d. PaCO2/PETCO2

ANS: B
The dead space-tidal volume ratio (VD/VT) is a measure of the efficiency of gas exchange.

DIF: Recall REF: p. 1166 OBJ: 6

17. What is the normal range for VDS/VT?


a. 0.10 to 0.20
b. 0.20 to 0.40
c. 0.30 to 0.50
d. 0.40 to 0.55

ANS: B
In healthy persons who are sitting, the VD/VT ratio is 0.20 to 0.40.

DIF: Recall REF: p. 1166 OBJ: 6

18. In which of the following disorders would an increased VDS/VT ratio not be likely?
a. congestive heart failure
b. pulmonary embolism
c. acute lung injury
d. hypothalamus tumor

Copyright © 2013, 2009, 2003, 1999, 1995, 1990, 1982, 1977, 1973, 1969 by Mosby, an imprint of Elsevier Inc.
Test bank 41-122

ANS: D
Frequently the VD/VT ratio is increased in patients with congestive heart failure, pulmonary
embolism, acute lung injury, or pulmonary hypertension and in patients undergoing mechanical
ventilation.

DIF: Application REF: p. 1166 OBJ: 6

19. All of the following values of VDS/VT, are suitable for weaning, except
a. 0.10
b. 0.20
c. 0.30
d. 0.60

ANS: D
A VD/VT ratio greater than 0.60 is predictive of lack of success at discontinuance of ventilation.

20. What is the normal disparity between end-tidal PCO2 and PaCO2?
a. End-tidal PCO2 is 1 to 5 mm Hg less than PaCO2
b. End-tidal PCO2 is 5 to 10 mm Hg less than PaCO2
c. End-tidal PCO2 is 1 to 5 mm Hg higher than PaCO2
d. End-tidal PCO2 is 5 to 10 mm Hg higher than PaCO2

ANS: C
The PETCO2 normally is 1 to 5 mm Hg less than the PaCO2.

DIF: Recall REF: p. 1167 OBJ: 6

21. All of the following lead to an increased end-tidal PCO2,except:


a. decreased effective ventilation
b. increased metabolic rate
c. decreased minute ventilation
d. Exercise

ANS: D
Positive pressure ventilation (especially with PEEP), pulmonary embolism, cardiac arrest, and
pulmonary hypoperfusion also may cause an increase in PaCO2 to PETCO2 gradient [P(a –
ET)CO2]. Exercise and a large tidal volume can reverse the P(a – ET)CO2 gradient, the PETCO2
can actually exceed the PaCO2.

DIF: Application REF: p. 1167 OBJ: 6

Copyright © 2013, 2009, 2003, 1999, 1995, 1990, 1982, 1977, 1973, 1969 by Mosby, an imprint of Elsevier Inc.
Test bank 41-123

22. All of the following are associated with a decreased end-tidal PCO2, except:
a. decreased metabolic rate
b. increase in lung perfusion
c. rapid and very shallow breathing
d. decreased CO2 production

ANS: B
Positive pressure ventilation (especially with PEEP), pulmonary embolism, cardiac arrest, and
pulmonary hypoperfusion also may cause an increase in PaCO2 to PETCO2 gradient
[P(a – ET)CO2]. Exercise and a large tidal volume can reverse the P(a – ET)CO2 gradient, the
PETCO2 can actually exceed the PaCO2.

DIF: Application REF: p. 1169 OBJ: 6

23. What is the value of determining the lower inflection point during measurement of the pressure-
volume curve?
a. best tidal volume
b. best PEEP level
c. to determine best airway size
d. to determine type of ventilator inspiratory flow pattern

ANS: B
A recommended strategy for setting PEEP is to set a level slightly above the lower inflection
point with the goal of recruitment and stabilization of dependent alveoli that would otherwise
sustain injury from repetitive opening, closing, and reopening during tidal ventilation.

DIF: Recall REF: p. 1169 OBJ: 7

24. What is the normal range for lung compliance?


a. 40 to 60 ml/cm H2O
b. 60 to 100 ml/cm H2O
c. 80 to 120 ml/cm H2O
d. 100 to 120 ml/cm H2O

ANS: B
Normal compliance ranges between 60 and 100 ml/cm H2O.

25. Which of the following is the Raw of intubated patients receiving ventilatory support?
a. 1 to 2 cm H2O/L/sec
b. 3 to 5 cm H2O/L/sec
c. 5 to 10 cm H2O/L/sec
d. 10 to 20 cm H2O/L/sec

ANS: C

Copyright © 2013, 2009, 2003, 1999, 1995, 1990, 1982, 1977, 1973, 1969 by Mosby, an imprint of Elsevier Inc.
Test bank 41-124

Intubated patients receiving mechanical ventilatory support typically have a Raw of 5 to 10 cm


H2O/L/sec.

DIF: Recall REF: p. 1170 OBJ: 7

26. Which of the following conditions is associated with an increased lung compliance
measurement?
a. Atelectasis
b. Pneumonia
c. Emphysema
d. bronchial intubation

ANS: C
See Box 46-7.

DIF: Application REF: p. 1169 OBJ: 7

27. All of the following are causes of increased airway resistance, except:
a. small endotracheal tube
b. high gas flow
c. increased secretions
d. Suctioning

ANS: D
See Box 46-7.

DIF: Application REF: p. 1169 OBJ: 7

28. What is the upper limit for plateau airway pressure that is recommended during mechanical
ventilation?
a. less than 30 cm H2O
b. 40 to 50 cm H2O
c. 60 to 80 cm H2O
d. depends on the patient

ANS: A
The Pplat ideally should not exceed 28 cm H2O because elevated Pplat increases the likelihood of
developing ventilator-induced lung injury.

DIF: Recall REF: p. 1170 OBJ: 8

29. All of the following factors are associated with an increased risk for auto-PEEP, except:
a. mechanical ventilation of a patient with obstructive lung disease
b. high minute volume during mechanical ventilation
c. acute respiratory distress syndrome (ARDS) patients
d. pulmonary fibrosis

Copyright © 2013, 2009, 2003, 1999, 1995, 1990, 1982, 1977, 1973, 1969 by Mosby, an imprint of Elsevier Inc.
Test bank 41-125

ANS: D
Patients receiving mechanical ventilation for obstructive airways disease have a large degree of
inhomogeneity in the emptying of lung units, and auto-PEEP can develop even at relatively low
minute ventilation. Auto-PEEP is common in mechanically ventilated patients with high minute
ventilation and thus occurs in some patients with ARDS.

30. All of the following are associated with auto-PEEP, except:


a. erroneous calculation of static lung compliance
b. hemodynamic compromise
c. Barotraumas
d. effective trigger sensitivity

ANS: D
The presence of auto-PEEP results in the underestimation of mean alveolar pressure when mean
airway pressure is being monitored to reflect mean alveolar pressure. An increase in mean
alveolar pressure due to auto-PEEP may exacerbate the hemodynamic effects of positive
pressure ventilation and increase the likelihood of barotrauma in a manner similar to that seen
with the application of PEEP.

DIF: Application REF: p. 1170 OBJ: 9

31. What is the average total work of breathing for healthy persons?
a. 0.31 to 0.2 J/L
b. 0.3 to 0.5 J/L
c. 0.5 to 0.8 J/L
d. 0.8 to 1.5 J/L

ANS: B
For healthy persons, the average total work of breathing ranges between 0.030 and 0.050 kg/m/L
(0.3 to 0.5 J/L).

DIF: Recall REF: p. 1171 OBJ: 9

32. What is the normal range for the percent of oxygen consumption consumed by the respiratory
muscles?
a. 2% to 5%
b. 5% to 10%
c. 10% to 15%
d. 20% to 25%

Copyright © 2013, 2009, 2003, 1999, 1995, 1990, 1982, 1977, 1973, 1969 by Mosby, an imprint of Elsevier Inc.
Test bank 41-126

ANS: A
Normal O2R is approximately 2% to 5% of total oxygen consumption.

DIF: Recall REF: p. 1172 OBJ: 9

33. Which of the following breathing patterns suggests respiratory muscle decompensation?
a. rapid and deep breaths
b. rapid and shallow breaths
c. slow and shallow breaths
d. slow and deep breaths

ANS: B
When muscular strength is limited, patients tend to meet minute ventilation ( E) requirements by
increasing frequency (f) while decreasing tidal volume (VT).

DIF: Recall REF: p. 1172 OBJ: 9

34. Which of the following two parameters are most commonly used for bedside assessment of
respiratory muscle strength?
a. vital capacity and maximum inspiratory pressure
b. vital capacity and peak flow
c. maximum inspiratory pressure and MVV
d. MVV and vital capacity

ANS: A
The two values most commonly used for bedside assessment of respiratory muscle strength are
vital capacity (VC) and maximal inspiratory pressure (MIP).

35. A vital capacity (VC) value below what value indicates significant muscle weakness?
a. 10 to 15 ml/kg
b. 20 to 25 ml/kg
c. 30 to 40 ml/kg
d. 50 to 60 ml/kg

ANS: A
A VC less than 10 to 15 ml/kg indicates considerable muscle weakness, which may inhibit the
ability to breathe spontaneously.

DIF: Recall REF: p. 1173 OBJ: 9

36. What is the best parameter to measure when trying to assess respiratory muscle endurance?
a. FVC
b. MIP
c. maximum voluntary ventilation (MVV)
d. NIF

Copyright © 2013, 2009, 2003, 1999, 1995, 1990, 1982, 1977, 1973, 1969 by Mosby, an imprint of Elsevier Inc.
Test bank 41-127

ANS: C
A measure used to assess respiratory muscle reserve, endurance, or fatigue is MVV.

DIF: Recall REF: p. 1173 OBJ: 9

37. All of the following are common purposes of using ventilator graphics, except:
a. to detect auto-PEEP
b. to assess effects of bronchodilators
c. to determine patient ventilator synchrony
d. to determine best FIO2

ANS: D
Ventilator graphics clearly show many important patientventilator interactions, such as
presence of auto-PEEP, elevated airway pressure, presence of secretions, and the general pattern
and dependability of supported ventilation (Figure 46-10).

DIF: Application REF: p. 1181 OBJ: 10

38. What is the normal range for mean arterial pressure?


a. 60 to 80 mm Hg
b. 50 to 100 mm Hg
c. 80 to 100 mm Hg
d. 90 to 120 mm Hg

ANS: C
See Table 46-3.

39. What is the normal central venous pressure reading?


a. 2 to 6 mm Hg
b. 5 to 12 mm Hg
c. 8 to 16 mm Hg
d. 12 to 22 mm Hg

ANS: A
See Table 46-3.

40. What is the normal mean pulmonary artery pressure?


a. 5 mm Hg
b. 10 mm Hg
c. 15 mm Hg
d. 20 mm Hg

ANS: C
See Table 46-3.

Copyright © 2013, 2009, 2003, 1999, 1995, 1990, 1982, 1977, 1973, 1969 by Mosby, an imprint of Elsevier Inc.
Test bank 41-128

41. What is the normal range for cardiac output?


a. 2 to 4 L/min
b. 4 to 8 L/min
c. 5 to 10 L/min
d. depends on patient age

ANS: B
See Table 46-3.

DIF: Recall REF: p. 1185 OBJ: 11

42. What is the normal range for pulmonary capillary wedge pressure?
a. 5 to 10 mm Hg
b. 15 to 20 mm Hg
c. 20 to 25 mm Hg
d. 30 to 35 mm Hg

ANS: A
See Table 46-3.

43. All of the following are associated with an increase in central venous pressure, except:
a. right heart failure
b. pulmonary valvular stenosis
c. pulmonary embolism
d. dehydration

ANS: D
See Box 46-9.

44. What hemodynamic parameter is best useful for estimated left ventricular end-diastolic pressure?
a. CVP
b. pulmonary capillary wedge pressure (PCWP)
c. SVR
d. PVR

ANS: B
PCWP is an estimate of left atrial pressure, which reflects left ventricular end-diastolic pressure.

DIF: Recall REF: p. 1187 OBJ: 11

45. What parameter is best used to assess left ventricular afterload?


a. CVP
b. PCWP
c. SVR
d. PVR

Copyright © 2013, 2009, 2003, 1999, 1995, 1990, 1982, 1977, 1973, 1969 by Mosby, an imprint of Elsevier Inc.
Test bank 41-129

ANS: C
An increase in systemic vascular resistance increases left ventricular afterload.

DIF: Recall REF: p. 1187 OBJ: 11

46. What medication is associated with dilated and fixed pupils in the intensive care unit patient?
a. atropine
b. lidocaine
c. vanceril
d. aminophylline

ANS: A
Dilated and fixed (unresponsive to light) pupils are seen in patients who have been given
atropine.

47. At what level of intracranial pressure will venous drainage be impeded and cerebral edema
develop in uninjured tissue?
a. 10 to 15 mm Hg
b. 20 to 30 mm Hg
c. 30 to 35 mm Hg
d. 40 to 45 mm Hg

ANS: C
At intracranial pressure levels of 30 to 35 mm Hg, venous drainage is impeded and edema
develops in uninjured tissue.

DIF: Recall REF: p. 1191 OBJ: 12

48. What is the Glasgow Coma Scale (GCS) score that requires intracranial pressure monitoring?
a. less than 8
b. less than 9
c. less than 10
d. less than 11

ANS: A
Head-injured patients with GCS scores of 8 and less need monitoring of intracranial pressure.

DIF: Recall REF: p. 1191 OBJ: 12

49. What is the most common method of estimating GFR?


a. blood urea nitrogen
b. blood urea nitrogen and creatinine
c. plasma creatinine and creatinine clearance
d. urine output

Copyright © 2013, 2009, 2003, 1999, 1995, 1990, 1982, 1977, 1973, 1969 by Mosby, an imprint of Elsevier Inc.
Test bank 41-130

ANS: C
The most common method of estimating GFR (renal function) is measurement of plasma
creatinine and creatinine clearance rate.

DIF: Recall REF: p. 1191 OBJ: 13

50. What level of urine output is considered anuria?


a. less than 500 ml/day
b. less than 250 ml/day
c. less than 150 ml/day
d. less than 50 ml/day

ANS: D
Anuria is present when urine output is less than 50 ml/d.

DIF: Recall REF: p. 1192 OBJ: 13

51. Elevation of which of the following substances is consistent with hepatic inflammation?
a. aspartate aminotransferase
b. Bilirubin
c. alkaline phosphatase
d. blood urea nitrogen

ANS: A
Elevated levels of aspartate aminotransferase and alanine aminotransferase suggest hepatic
inflammation.

52. Which of the following physical findings is NOT consistent with starvation?
a. temporal muscle wasting
b. sunken supraclavicular fossae
c. decreased adipose stores
d. less 50 ml/day

ANS: D
Temporal muscle wasting, sunken supraclavicular fossae and decreased adipose stores are easily
recognized signs of starvation.

Copyright © 2013, 2009, 2003, 1999, 1995, 1990, 1982, 1977, 1973, 1969 by Mosby, an imprint of Elsevier Inc.
Test bank 41-131

53. Which of the following levels of albumin are consistent with severe malnutrition?
a. less than 3.0 g/dl
b. less than 2.5 g/dl
c. less than 2.2 g/dl
d. less than 4.0 g/dl

ANS: C
Serum albumin concentration is the most frequently used laboratory measure of nutritional
status, a value less than 2.2 g/dl generally reflecting severe malnutrition.

DIF: Recall REF: p. 1192 OBJ: 13

54. Which of the following is NOT a global monitoring index?


a. APACHE
b. APS
c. TISS
d. ATS

ANS: D
These indices (Acute Physiology and Chronic Health Evaluation [APACHE 1, 2, 3 and 4], Acute
Physiology Score [APS], Therapeutic Intervention Scoring System [TISS], and Burns Weaning
Assessment Program [BWAP]) are determinations of scores from a number of monitored values
obtained from snapshots of the patient’s condition, usually during the first 24 hours after hospital
admission.

55. Which of the following should be considered first if medical and mechanical problems have been
excluded and the patient continues to fight the ventilator or exhibit high levels of agitation or
distress?
a. paralytics
b. sedatives
c. narcotics
d. anesthetics

ANS: B
If medical and mechanical problems have been excluded and the patient continues to fight the
ventilator or exhibit high levels of agitation or distress, sedation should be considered.

56. All of the following are used to determine a patient’s neurologic status, except:
a. measuring VD/VT ratio
b. pupillary response and eye movement
c. corneal and gag reflex
d. respiratory rate and pattern

ANS: A

Copyright © 2013, 2009, 2003, 1999, 1995, 1990, 1982, 1977, 1973, 1969 by Mosby, an imprint of Elsevier Inc.
Test bank 41-132

Neurologic examinations include evaluation of the following: mental status, pupillary response,
eye movements, corneal responses, gag reflex, respiratory rate and pattern, motor evaluation and
sensory evaluation.

57. Which of the following bedside assessment of respiratory muscle strength is commonly used on
ventilated patients?
a. tidal volume
b. maximal inspiratory pressure
c. minute ventilation
d. respiratory rate

ANS: B
Two values commonly used for bedside assessment of respiratory muscle strength are vital
capacity (VC) and maximal inspiratory pressure (MIP).

58. Which of the following are primary reasons to measure intracranial pressure (ICP)?
1. to monitor patients at risk of life-threatening intracranial hypertension
2. to monitor for evidence of infection
3. to assess the effects of therapy aimed at reducing ICP
4. to maintain the mean ICP > 20 mm Hg
a. 1 only
b. 1, 2 and 4 only
c. 1, 2, 3 only
d. 1, 2, 3 and 4

ANS: C
There are three primary reasons to measure intracranial pressure (ICP): (1) to monitor patients at
risk of life-threatening intracranial hypertension, (2) to monitor for
evidence of inflection, and (3) to assess the effects of therapy aimed at reducing ICP. Also,
normal mean ICP for a patient in the supine is normally 10 to 15 mm Hg, ICP. Elevations in ICP
to 15 to 20 mm Hg compress the capillary bed and compromise microcirculation.

DIF: Recall REF: p. 1191 OBJ: 12

59. Which of the following factors affect cardiac performance?


1. preload
2. afterload
3. contractility
4. cardiac output
a. 1 and 2 only
b. 1 and 3 only
c. 1, 2, and 3 only
d. 1, 2, 3, and 4

ANS: C

Copyright © 2013, 2009, 2003, 1999, 1995, 1990, 1982, 1977, 1973, 1969 by Mosby, an imprint of Elsevier Inc.
Test bank 41-133

Cardiac Performance is affected by preload, contractility and afterload, and is evaluated by the
measurement of cardiac output

DIF: Recall REF: p. 1188 OBJ: 11

Copyright © 2013, 2009, 2003, 1999, 1995, 1990, 1982, 1977, 1973, 1969 by Mosby, an imprint of Elsevier Inc.
Test bank 41-134

Chapter 47: Discontinuing Ventilatory Support

1. All of the following factors will increase ventilatory demand (workload) except:
a. severe hypoxemia
b. pulmonary infection
c. increased compliance
d. Bronchospasm

ANS: C
Factors that may increase ventilatory workload are summarized in Box 47-1.

2. Ventilatory capacity is determined by all of the following except:


a. central nervous system (CNS) drive
b. trigger level
c. muscle strength
d. muscle endurance

ANS: B
Ventilatory capacity is determined by (1) CNS drive, (2) ventilatory muscle strength, and (3)
ventilatory muscle endurance.

DIF: Recall REF: p. 1221 OBJ: 1

3. All of the following factors can reduce a patient’s ventilatory drive except:
a. respiratory alkalosis
b. metabolic acidosis
c. depressant drugs
d. decreased metabolism

ANS: B
Box 47-2 summarizes factors that may reduce ventilatory drive.

DIF: Recall REF: p. 1202 OBJ: 2

4. When is ventilator dependence likely to occur?


1. when ventilatory capacity exceeds demand
2. when arterial hypoxemia is present
3. when the patient is malnourished
4. when the cardiovascular system is unstable
a. 2 and 4
b. 1, 2, and 3
c. 3 and 4
d. 2, 3, and 4

ANS: D

Copyright © 2013, 2009, 2003, 1999, 1995, 1990, 1982, 1977, 1973, 1969 by Mosby, an imprint of Elsevier Inc.
Test bank 41-135

Other factors that may contribute to ventilator dependence include inadequate arterial
oxygenation, poor tissue oxygen delivery, myocardial ischemia, arrhythmias, low cardiac output,
and cardiovascular instability.

DIF: Recall REF: p. 1202 OBJ: 2

5. What is the most important prerequisite for weaning a patient from ventilatory support?
a. improvement in the original problem requiring mechanical ventilation
b. assurance that the patient’s ventilatory demand exceeds the patient’s capacity
c. objective evidence indicating good respiratory muscle endurance
d. ability to maintain adequate oxygenation with an FIO2 less than 0.4

ANS: A
The single most important criterion to consider when evaluating a patient for ventilator
discontinuation or weaning is whether there has been significant alleviation or reversal of the
disease state or condition that necessitated use of the ventilator in the first place.

DIF: Recall REF: p. 1203 OBJ: 3

6. What is the least reliable weaning index?


a. vital capacity (VC)
b. maximum inspiratory capacity (MIP)
c. minute ventilation (VE)
d. rapid-shallow breathing index (f/VT)

ANS: A
With respect to the more traditional weaning indices, vital capacity can be highly variable,
whereas MIP, minute ventilation, respiratory rate (f), and f/VT tend to be more reliable.

DIF: Recall REF: p. 1204 OBJ: 3

7. All of the following oxygenation measures support a patient’s readiness to wean except:
a. PaO2/PAO2 (a/A) = 0.45
b. PaO2/FIO2 (P/F) = 110
c. PAO2 – PaO2 = 240 mm Hg
d. physiologic shunt ( ) = 12%

ANS: B
See Table 47-1.

DIF: Analysis REF: p. 1204 OBJ: 3

8. All of the following indicate that an adult patient is ready to be weaned from ventilatory support
except:
a. VC = 1.9 L
b. spontaneous rate of 32/min

Copyright © 2013, 2009, 2003, 1999, 1995, 1990, 1982, 1977, 1973, 1969 by Mosby, an imprint of Elsevier Inc.
Test bank 41-136

c. = 8%
d. MIP = –45 cm H2O

ANS: B
See Table 47-1.

DIF: Analysis REF: p. 1204 OBJ: 3

9. All of the following indicate that an adult patient is ready to be weaned from ventilatory support
except:
a. PAO2 – PaO2 = 430 on 100% O2
b. VD/VT = 0.55
c. MIP = –33 cm H2O
d. PO2 = 76 mm Hg on 40% O2

ANS: A
See Table 47-1.

10. Which of the following five adult patients receiving ventilatory support is the best candidate for
weaning?
Maximum voluntary
Patient VC VE ventilation (MVV) MIP VD/VT
a.
b.
c.
d.

ANS: A
See Table 47-1.

DIF: Analysis REF: p. 1204 OBJ: 3

Copyright © 2013, 2009, 2003, 1999, 1995, 1990, 1982, 1977, 1973, 1969 by Mosby, an imprint of Elsevier Inc.
Test bank 41-137

11. Which of the following patients exhibits an acceptable ventilatory demand?


VE PaCO2
a.
b.
c.
d.

ANS: C
See Table 47-1.

DIF: Analysis REF: p. 1204 OBJ: 3

12. A patient has an adequate ventilatory reserve if which of the following is TRUE?
a. ability to double the resting minute ventilation
b. normal PaCO2 and minute ventilation less than 10 L/min
c. MIP = –55 cm H2O
d. VD/VT less than 0.4

ANS: A
See Table 47-1.

DIF: Analysis REF: p. 1204 OBJ: 3

13. You measure the spontaneous rate of breathing and VT on four patients receiving ventilator
support. For which one is successful weaning most likely?
Breathing frequency VT
a.
b.
c.
d.

ANS: B
See Table 47-1.

DIF: Analysis REF: p. 1204 OBJ: 3

14. A patient receiving ventilator support has a spontaneous rate of breathing of 26/min and an
average VT of 300 ml. What is this patient’s rapid-shallow breathing index?
a. 12
b. 87
c. 105
d. 66

ANS: B
15. Which of the following signs observed on a mechanically ventilated patient indicate that
successful weaning is unlikely?

Copyright © 2013, 2009, 2003, 1999, 1995, 1990, 1982, 1977, 1973, 1969 by Mosby, an imprint of Elsevier Inc.
Test bank 41-138

1. palpable scalene muscle use during inspiration


2. palpable abdominal tensing during expiration
3. presence of an irregular breathing pattern
4. patient unable to alter breathing pattern on command
a. 2 and 4
b. 1, 2, and 3
c. 3 and 4
d. 1, 2, 3, and 4

ANS: D
Evaluation of patients for the presence of palpable scalene muscle use during inspiration, an
irregular ventilatory pattern, palpable abdominal muscle tensing during expiration, and inability
to alter ventilatory pattern on command can be helpful in assessment of the potential for
prolonged spontaneous ventilation.

16. Which of the following is false about the P0.1 measure?


a. P0.1 correlates well with central respiratory drive.
b. P0.1 is the airway pressure measured 100 ms after occlusion.
c. P0.1 is an effort-dependent measure of respiratory drive.
d. Chronic obstructive pulmonary disease (COPD) patients with a P0.1 greater than 6
cm H2O are difficult to wean.

ANS: C
Airway occlusion pressure (P0.1) is the inspiratory pressure measured 100 milliseconds after
airway occlusion. The P0.1 is effort independent and correlates well with central respiratory drive.
Ventilator-dependent patients with COPD who have a P0.1 greater than 6 cm H2O tend to be
difficult to wean.

17. Successful weaning is less likely when a patient’s work of breathing exceeds what level?
a. 4 J/min
b. 8 J/min
c. 12 J/min
d. 16 J/min

ANS: D
Successful weaning has been found to be less likely among patients with spontaneous work
levels greater than 1.6 kg/m/min (16 J/min) or 0.14 kg/m/L (1.4 J/L).

18. Above what pressure-time index (PTI) will most patients be unable to sustain spontaneous
breathing?
a. 0.03
b. 0.05
c. 0.10
d. 0.15

ANS: D

Copyright © 2013, 2009, 2003, 1999, 1995, 1990, 1982, 1977, 1973, 1969 by Mosby, an imprint of Elsevier Inc.
Test bank 41-139

A PTI greater than 0.15 to 0.18 has been associated with diaphragmatic fatigue, and a PTI
greater than 0.15 cannot be sustained indefinitely.

19. Which of the following metabolic factors can hinder weaning?


1. excessive carbohydrate feeding
2. amino acidbased parenteral nutrition
3. calorie intake = 1.5  resting energy expenditure (REE)
a. 2 and 3
b. 1 and 2
c. 1, 2, and 3
d. 1 and 3

ANS: B
Excessive carbohydrate feeding can increase carbon dioxide production and may precipitate
acute hypercapnic respiratory failure. Parenteral nutrition solutions containing amino acid
formulations (arginine/lysine) can cause metabolic acidosis and thus increase ventilatory
demand.

DIF: Recall REF: p. 1206 OBJ: 5

20. All of the following indicate that a patient’s renal function is adequate for weaning except:
a. output = 20 ml/hr
b. no major weight gain
c. no edema present
d. normal electrolytes

ANS: A
The patient ideally should have an adequate urine output (greater than 1000 ml/day), and there
should be no inappropriate weight gain or edema.

DIF: Application REF: p. 1206 OBJ: 5

21. Which of the following electrolyte imbalances can hinder weaning from ventilatory support?
1. hypophosphatemia
2. hypomagnesemia
3. hypokalemia
a. 2 and 3
b. 1 and 2
c. 1, 2, and 3
d. 1 and 3

ANS: A
Key electrolytes should be normal (magnesium, 1.8 to 3.0 mEq/L; phosphate, 2.5 to 4.8 mEq/L;
potassium, 3.5 to 5.0 mEq/L).

DIF: Recall REF: p. 1206 OBJ: 5

Copyright © 2013, 2009, 2003, 1999, 1995, 1990, 1982, 1977, 1973, 1969 by Mosby, an imprint of Elsevier Inc.
Test bank 41-140

22. Which of the following cardiovascular signs would indicate that a patient’s cardiovascular status
is unstable and that weaning should NOT begin at this time?
a. cardiac index of 2.5 L/min/m2
b. hemoglobin content of 10 g/dl
c. heart rate of 108/min
d. systolic blood pressure of 80 mm Hg

ANS: D
Table 47-2 provides criteria for confirming cardiovascular stability.

DIF: Application REF: p. 1207 OBJ: 4

23. Of the following adult patients receiving ventilatory support, which has a stable enough
cardiovascular profile to consider weaning?
Systolic blood
Heart rate pressure (mm Hg) Hemoglobin (g/dl) Clinical status
a.
b.
c.
d.

ANS: A
Table 47-2 provides criteria for confirming cardiovascular stability.

24. Prerequisites for successful weaning include:


1. psychological readiness
2. adequate gag and swallow reflexes
3. ability to follow instructions
4. adequate cough
a. 1, 2, and 3
b. 2 and 4
c. 1, 2, 3, and 4
d. 3 and 4

ANS: C
Adequate central nervous system function is needed to ensure stable ventilatory drive, adequate
secretion clearance (cough and deep breathing), and protection of the airway (gag reflex and
swallow). In addition, level of consciousness, dyspnea, anxiety, depression, and motivation can
affect weaning success. The patient ideally is awake and alert, free of seizures, and able to follow
instructions.

DIF: Recall REF: p. 1206-1207 OBJ: 5

Copyright © 2013, 2009, 2003, 1999, 1995, 1990, 1982, 1977, 1973, 1969 by Mosby, an imprint of Elsevier Inc.
Test bank 41-141

25. Which of the following must you verify when considering weaning an obtunded patient?
1. adequate gag reflex
2. no depressant drugs
3. adequate cough
a. 2 and 3
b. 1 and 2
c. 1, 2, and 3
d. 1 and 3

ANS: D
Obtunded patients should, at a minimum, have an adequate gag reflex and cough.

DIF: Recall REF: p. 1207 OBJ: 5

26. All of the following drug categories can depress ventilatory drive and hinder weaning except:
a. analgesics
b. narcotics
c. hypnotics
d. antibiotics

ANS: D
Level of consciousness is affected by the use of narcotic, sedative, and analgesic drugs.

DIF: Recall REF: p. 1207 OBJ: 5

27. Which of the following techniques can help to decrease a patient’s imposed work of breathing
during weaning from ventilatory support?
1. use of pressure-supported ventilation (PSV)
2. trigger breath by flow, not pressure
3. application of small amounts of continuous positive airway pressure (CPAP) or positive end-
expiratory pressure (PEEP)
4. use of automatic tube compensation (ATC)
a. 2 and 4
b. 1, 2, and 3
c. 3 and 4
d. 1, 2, 3, and 4

ANS: D
The patient’s ventilatory workload should be minimized with PSV. Flow trigger, flow-by, or
ATC also may be helpful in minimizing imposed ventilatory work. Intrinsic PEEP during
mechanical ventilation may increase trigger work, and small amounts of PEEP or CPAP can help
overcome this problem.

28. Common approaches used to wean patients from ventilatory support include which of the
following?

Copyright © 2013, 2009, 2003, 1999, 1995, 1990, 1982, 1977, 1973, 1969 by Mosby, an imprint of Elsevier Inc.
Test bank 41-142

1. T-tube alternating with mechanical ventilation


2. pressure-supported ventilation (PSV)
3. intermittent mandatory ventilation
a. 2 and 3
b. 1 and 2
c. 1, 2, and 3
d. 1 and 3

ANS: C
There are three basic methods of discontinuing ventilatory support: (1) spontaneous breathing
trials (usually with a T tube) alternating with mechanical ventilatory support, (2) synchronized
intermittent mandatory ventilation, and (3) PSV.

DIF: Recall REF: p. 1209 OBJ: 6

29. Which of the following ventilator strategies would you consider as a good alternative to T-tube
trials when using a rapid weaning protocol?
1. continuous positive airway pressure (CPAP) with flow-by (flow triggering)
2. low-level pressure-supported ventilation (PSV)
3. intermittent mandatory ventilation
a. 2 and 3
b. 1 and 2
c. 1, 2, and 3
d. 1 and 3

ANS: B
Rather than using a T-tube trial, some clinicians prefer to maintain the patient attached to the
ventilator with zero PSV and zero CPAP.

DIF: Recall REF: p. 1210 OBJ: 6

30. Which method of weaning may be useful to minimize auto-PEEP?


a. intermittent mandatory ventilation
b. synchronized intermittent mandatory ventilation
c. continuous positive airway pressure (CPAP)
d. T-piece

ANS: C
Low levels of CPAP may be useful in maintaining lung volumes and overcoming intrinsic PEEP,
if present.

DIF: Application REF: p. 1210 OBJ: 6

Copyright © 2013, 2009, 2003, 1999, 1995, 1990, 1982, 1977, 1973, 1969 by Mosby, an imprint of Elsevier Inc.
Test bank 41-143

31. A physician orders a T-tube trial for a patient receiving ventilatory support in the assist-control
mode with an FIO2 of 0.4. What FIO2 would you recommend for this patient during the
spontaneous breathing period?
a. 0.3
b. 0.4
c. 0.5
d. 0.6

ANS: C
See Box 47-8.

DIF: Application REF: p. 1211 OBJ: 6

32. Advantages of adding continuous positive airway pressure (CPAP) to T-tube weaning include all
of the following except:
a. improved blood oxygenation
b. decreased work of breathing
c. compensation for auto-PEEP
d. faster weaning or extubation

ANS: D
CPAP has the advantage of maintaining lung volume during the weaning phase and thus of
improving the patient’s oxygenation status. Minimal levels of CPAP may be useful in reducing
work of breathing and compensating for auto-PEEP, particularly in patients with obstructive lung
disease.

DIF: Recall REF: p. 1214 OBJ: 6

33. An alert patient receiving intermittent mandatory ventilation at a rate of 8/min and V T of 600 ml
has stable vital signs and satisfactory blood gases on an FIO2 of 0.45. What would you do to
initiate weaning for this patient?
a. Lengthen the automatic sigh interval.
b. Decrease the mandatory rate to 5 to 6/min.
c. Increase FIO2 to 60%.
d. Decrease the VT to 500 ml.

ANS: B
At this point, the rate is reduced in a stepwise manner until complete spontaneous breathing can
be achieved.

DIF: Application REF: p. 1215 OBJ: 6

34. A physician has selected a pressure support protocol to wean a patient off ventilatory support.
Which of the following pressure levels would you recommend to begin the weaning process?
a. pressure sufficient to obtain a VT of 3 to 5 ml/kg of ideal body weight (IBW)
b. pressure sufficient to overcome the imposed workload

Copyright © 2013, 2009, 2003, 1999, 1995, 1990, 1982, 1977, 1973, 1969 by Mosby, an imprint of Elsevier Inc.
Test bank 41-144

c. pressure sufficient to obtain a VT of 6 to 10 ml/kg IBW


d. pressure equal to 30% of the volume-cycled peak inspiratory pressure

ANS: C
For initial ventilator setup in the pressure support mode, the beginning pressure level can be
adjusted to deliver an appropriate tidal volume, usually approximately 6-10 ml/kg of IBW.

35. A physician is using a pressure support protocol to wean a patient off ventilatory support. The
patient is now at a 5 cm H2O pressure level and has a spontaneous respiratory rate of 21/min.
Other cardiovascular and respiratory signs indicate that the patient remains stable. Which of the
following actions would you recommend at this point?
a. Switch the patient to 5 cm H2O continuous positive airway pressure (CPAP)
through the endotracheal tube.
b. Extubate the patient and provide supplemental O2.
c. Switch the patient to intermittent mandatory ventilation at a rate of 2/min.
d. Decrease the pressure support level to 3 cm H2O.

ANS: B
In general, patients who can spontaneously breathe comfortably at this level of pressure support
can be extubated without problems.

36. An alert patient receiving ventilatory support through a demand flow intermittent mandatory
ventilation system exhibits clinical signs of an increased work of breathing whenever you try to
decrease the mandatory rate below 6/min. In order to aid in weaning this patient, which of the
following would you recommend?
a. Apply a low level of pressure support.
b. Apply a high level of inspiratory pressure.
c. Increase the mandatory VT.
d. Decrease the mandatory VT.

ANS: A
With synchronized intermittent mandatory ventilation, the addition of pressure support can
overcome the work of breathing imposed during “spontaneous” breaths because of the presence
of endotracheal and tracheostomy tubes, demand flow systems, and ventilator circuits.

DIF: Application REF: p. 1216 OBJ: 6

37. What are some advantages of mandatory minute ventilation (MMV) as a weaning tool?
1. It provides greater control over PaCO2 than intermittent mandatory ventilation does.
2. It prevents acidemia with acute hypoventilation.
3. It eliminates concerns over depressant drugs.
4. It ensures an efficient pattern of ventilation.
a. 2 and 4
b. 1, 2, and 3
c. 3 and 4
d. 1, 2, 3, and 4

Copyright © 2013, 2009, 2003, 1999, 1995, 1990, 1982, 1977, 1973, 1969 by Mosby, an imprint of Elsevier Inc.
Test bank 41-145

ANS: B
See Box 47-12.

DIF: Recall REF: p. 1216 OBJ: 7

38. Which of the following ventilator modes can ensure delivery of a preset VT during spontaneous
breathing?
1. volume support or VERSUS (Siemens Servo Ventilator 300)
2. volume-assured pressure support or VAPS (Bird 8400ST)
3. augmented minute ventilation or MMV (Bear 1000)
a. 2 and 3
b. 1 and 2
c. 1 and 3
d. 1, 2, and 3

ANS: B
Volume-assured pressure support is similar to volume support in that a minimum preset tidal
volume is maintained by means of automatic adjustment of the ventilator.

DIF: Recall REF: p. 1216 OBJ: 7

39. Which of the following is false about noninvasive positive-pressure ventilation (NPPV)?
a. NPPV can support ventilation without a tracheal airway.
b. NPPV should not be used with patients at risk for aspiration.
c. Patients likely to fail weaning are good candidates for NPPV.
d. NPPV can be used to prevent reintubation when weaning fails.

ANS: C
Patients who are likely to be unsuccessful at weaning are not good candidates for NPPV.

DIF: Recall REF: p. 1210-1211 OBJ: 7

40. In most weaning protocols, what minimum blood gas parameters are needed to start the process?
a. PaO2 greater than 70 mm Hg and PaCO2 less than 50 mm Hg on FIO2 less than 0.6
and PEEP less than 5 cm H2O
b. PaO2 greater than 70 mm Hg and PaCO2 less than 50 mm Hg on FIO2 less than 0.4
and PEEP 5 cm H2O or greater
c. PaO2 greater than 50 mm Hg and PaCO2 less than 50 mm Hg on FIO2 less than 0.6
and PEEP less than 10 cm H2O
d. PaO2 greater than 70 mm Hg and PaCO2 less than 70 mm Hg on FIO2 less than 0.6
and PEEP 5 cm H2O or greater

ANS: B
See Box 47-14.

Copyright © 2013, 2009, 2003, 1999, 1995, 1990, 1982, 1977, 1973, 1969 by Mosby, an imprint of Elsevier Inc.
Test bank 41-146

DIF: Recall REF: p. 1227 OBJ: 6

41. Which of the following weaning methods provides the best respiratory muscle strength
conditioning?
a. pressure-supported ventilation
b. T-tube
c. intermittent mandatory ventilation
d. volume-assured pressure support (VAPS)

ANS: B
See Table 47-4.

DIF: Recall REF: p. 1219 OBJ: 7

42. All of the following are disadvantages of using the T-tube method for weaning except:
a. more staff time required
b. abrupt transition sometimes difficult
c. high imposed work of breathing
d. lack of alarm systems

ANS: C
See Table 47-4.

DIF: Recall REF: p. 1219 OBJ: 7

43. All of the following are disadvantages of using intermittent mandatory ventilation for weaning
except:
a. potentially high work of breathing
b. weaning time possibly prolonged
c. patientventilator dyssynchrony
d. higher mean airway pressures

ANS: D
See Table 47-4.

DIF: Recall REF: p. 1219 OBJ: 7

44. All of the following are advantages of using pressure-supported ventilation for weaning except:
a. guaranteed VT
b. reduced work of breathing
c. respiratory muscle fatigue prevented
d. better patient comfort and synchrony

ANS: A
See Table 47-4.

Copyright © 2013, 2009, 2003, 1999, 1995, 1990, 1982, 1977, 1973, 1969 by Mosby, an imprint of Elsevier Inc.
Test bank 41-147

DIF: Recall REF: p. 1219 OBJ: 7

45. Which of the following cardiovascular changes would you consider a bad sign during weaning a
patient from ventilatory support?
a. development of chest pain
b. increase in both stroke volume and cardiac index
c. increase in heart rate from 95 to 110/min
d. fall in blood pressure from 143/95 to 126/88 mm Hg

ANS: A
See Table 47-6.

46. While monitoring a patient being weaned through a T-tube protocol, signs indicating that
mechanical ventilation should be restored include all of the following except:
a. development of cardiac arrhythmias
b. asynchronous or paradoxical breathing
c. development of severe hypotension
d. moderate rise in respiratory rate

ANS: D
47. While monitoring a patient during a T-tube weaning trial, you notice the following: an increase
in heart rate from 86 to 100/min; an increase in respiratory rate from 12 to 23/min; an increase in
PaCO2 from 39 to 45 mm Hg; and a decrease in PaO2 from 82 to 73 mm Hg. Which of the
following actions would be appropriate at this time?
a. Reconnect the patient to the ventilator with prior settings.
b. Request that the patient be administered a mild sedative.
c. Suction the patient after manual hyperinflation or oxygenation.
d. Encourage the patient to relax, and continue careful monitoring.

ANS: D
See Table 47-6.

DIF: Analysis REF: p. 1220 OBJ: 6

48. Which of the following changes can be expected when weaning a patient through a T-tube trial?
1. increase in respiratory rate of 10/min
2. increase in heart rate of 15 to 20/min
3. 5 to 10 mm Hg rise in the arterial PCO2
4. doubling of the minute ventilation
a. 2 and 4
b. 1, 2, and 3
c. 3 and 4
d. 1, 2, 3, and 4

ANS: B

Copyright © 2013, 2009, 2003, 1999, 1995, 1990, 1982, 1977, 1973, 1969 by Mosby, an imprint of Elsevier Inc.
Test bank 41-148

See Table 47-6.

DIF: Application REF: p. 1220 OBJ: 6

49. While monitoring a patient during a T-tube weaning trial, you notice the following: increased
patient agitation; increased heart rate (from 90 to 118/min); increased respiratory rate (from 17 to
33/min with some paradoxical motion); and premature ventricular contractions (PVCs)
increasing to an average of 5/min. Which of the following actions would be appropriate at this
time?
a. Reconnect the patient to the ventilator with prior settings.
b. Encourage the patient to relax, and continue careful monitoring.
c. Request that the patient be given a stat (immediate) bolus of lidocaine.
d. Request that the patient be given a strong sedative or hypnotic.

ANS: A
See Table 47-6.

DIF: Analysis REF: p. 1220 OBJ: 6

50. Which of the following is FALSE about artificial tracheal airways and weaning?
a. There are decreases in tube inner diameter (ID) and increases in VE increase the
work of breathing.
b. The added work due to artificial airways can increase ventilator dependence.
c. Artificial airways can increase the work of breathing nearly threefold.
d. Tracheostomy tubes increase the work of breathing more than can endotracheal
tubes.

ANS: D
The presence of an artificial airway may increase airway resistance nearly threefold, although
some evidence calls into question the assumption that breathing through an endotracheal tube
offers a greater work of breathing than does breathing through a natural airway postextubation.
In a study with 14 successfully extubated patients, at the end of a 2-hour spontaneous breathing
trial there was no difference in work of breathing before and after extubation.

51. What is the best way to decrease the work of breathing imposed by an artificial airway on a
patient receiving ventilatory support?
a. Provide pressure support.
b. Decrease inspiratory flow.
c. Lower the minute ventilation.
d. Use low rates of breathing.

ANS: A
Pressure support ventilation can be very effective in overcoming this imposed work.

DIF: Application REF: p. 1216 OBJ: 6

Copyright © 2013, 2009, 2003, 1999, 1995, 1990, 1982, 1977, 1973, 1969 by Mosby, an imprint of Elsevier Inc.
Test bank 41-149

52. What are some factors that indicate a patient’s readiness for extubation?
1. adequate oxygenation or ventilation with spontaneous breathing
2. minimal risk for upper airway obstruction
3. adequate airway protection or minimal aspiration risk
4. adequate clearance of pulmonary secretions
a. 2 and 4
b. 1, 2, and 3
c. 3 and 4
d. 1, 2, 3, and 4

ANS: D
See Box 47-15.

DIF: Recall REF: p. 1224 OBJ: 8

53. What patients are at high risk for postextubation upper airway obstruction?
1. those with neuromuscular disorders
2. those who have had major neck surgery
3. those with infectious masses or abscesses
a. 2 and 3
b. 1 and 2
c. 1 and 3
d. 1, 2, and 3

ANS: D
Compression of the airway due to traumatic or postoperative hematoma of the neck, infectious
masses or abscesses, and malignant tumors or compression after major head or neck surgery can
lead to upper airway obstruction after extubation.

DIF: Recall REF: p. 1221 OBJ: 6

54. In considering a patient for endotracheal tube extubation, which of the following procedures
would you recommend to determine the risk of postextubation upper airway obstruction?
a. methylene blue test
b. pre- and post-bronchodilator
c. cuff leak test
d. forced vital capacity

ANS: C
The cuff leak test is recommended to detect airway obstruction before extubation.

DIF: Recall REF: p. 1221 OBJ: 5

Copyright © 2013, 2009, 2003, 1999, 1995, 1990, 1982, 1977, 1973, 1969 by Mosby, an imprint of Elsevier Inc.
Test bank 41-150

55. Which of the following patients are at high risk for severe laryngeal edema after an endotracheal
tube extubation?
1. pediatric burn victim
2. patient with epiglottitis
3. smoke inhalation patient
4. pulmonary fibrosis patient
a. 2 and 4
b. 1, 2, and 3
c. 3 and 4
d. 1, 2, 3, and 4

ANS: B
Children, patients with epiglottitis or angioedema (dermal, subcutaneous, or submucosal edema
of the face or larynx), and patients who have sustained smoke inhalation are at greater risk.

DIF: Recall REF: p. 1222 OBJ: 5

56. In considering a patient for extubation, which of the following would you recommend to
minimize the risk of postextubation aspiration?
a. Perform and confirm a positive cuff leak test.
b. Discontinue (DC) tube feeding 4 to 6 hours before extubation.
c. Perform deep endotracheal suctioning before extubation.
d. Keep the cuff inflated when removing the tube.

ANS: B
Withholding feeding 4 to 6 hours before extubation and clamping feeding tubes may be prudent.

DIF: Recall REF: p. 1221 OBJ: 8

57. Common causes for weaning failure include all of the following except:
a. myocardial ischemia
b. critical illness polyneuropathy
c. psychological dependence
d. secondary polycythemia

ANS: D

Copyright © 2013, 2009, 2003, 1999, 1995, 1990, 1982, 1977, 1973, 1969 by Mosby, an imprint of Elsevier Inc.
Test bank 41-151

Myocardial ischemia may occur frequently among ventilator-dependent patients and has been
associated with weaning failure. Critical illness polyneuropathy has been cited as a frequent
cause of neuromuscular weaning failure among critically ill patients. Unsuspected
neuromuscular disease may be an important factor in ventilator dependency. Inability to wean
can sometimes be attributed to psychological dependence, poor oxygenation status, or
cardiovascular instability (congestive heart failure or ischemia).

DIF: Recall REF: p. 1222 OBJ: 9

58. A patient whom you are trying to wean below 5 cm H2O pressure support develops respiratory
muscle fatigue. Which of the following would you recommend to overcome this problem?
1. Make sure there is adequate O2 transport or cardiac output.
2. Make sure that the patient is adequately nourished.
3. Check and replace any depleted electrolytes.
4. Clear secretions and provide bronchodilation.
a. 2 and 4
b. 1, 2, and 3
c. 3 and 4
d. 1, 2, 3, and 4

ANS: D
See Table 47-8.

59. All of the following are useful strategies in managing the psychological problems encountered in
weaning some patients from ventilator support except to:
a. secure a psychiatric consult
b. decrease environmental stress
c. avoid mental stimulation
d. teach relaxation methods

ANS: C
See Table 47-8.

DIF: Recall REF: p. 1223 OBJ: 10

60. Who should make the decisions related to terminal weaning?


a. patient
b. patient’s family and patient’s physician
c. nurse
d. respiratory therapist

ANS: B

Copyright © 2013, 2009, 2003, 1999, 1995, 1990, 1982, 1977, 1973, 1969 by Mosby, an imprint of Elsevier Inc.
Test bank 41-152

The decision should be made by the family in consultation with the patient’s physician and in
accordance with established ethical and legal guidelines.

DIF: Recall REF: p. 1224 OBJ: 10

61. In an effort to determine a patient’s need for ventilatory support, which of the following factors
increase ventilatory workload?
1. decreased lung compliance
2. decreased thoracic compliance
3. increased airway resistance
4. artificial airways
a. 1 only
b. 1, and 2 only
c. 1, 2, and 3 only
d. 1, 2, 3, and 4

ANS: D
See Box 47-3.

DIF: Recall REF: p. 1203 OBJ: 2

62. The physician has requested that the respiratory therapist determine a patient’s rapid shallow
breathing index (RSBI) before placing the patient on PSV. The patient has a respiratory rate of
32/min and VT of 300 mL. What is the patient’s RSBI?
a. 107
b. 110
c. 112
d. 114

ANS: A
The formula for calculating RSBI is f/VT. The therapist would calculates 32/.30 = 107

63. Which of the following health care disciplines should be involved in the care of a patient who is
considered difficult to wean?
1. physical therapy
2. speech therapy
3. social services
4. occupational therapy
a. 1 and 2 only
b. 2 and 3 only
c. 1, 2, and 4 only
d. 1, 2, 3, and 4

ANS: D

Copyright © 2013, 2009, 2003, 1999, 1995, 1990, 1982, 1977, 1973, 1969 by Mosby, an imprint of Elsevier Inc.
Test bank 41-153

Copyright © 2013, 2009, 2003, 1999, 1995, 1990, 1982, 1977, 1973, 1969 by Mosby, an imprint of Elsevier Inc.

You might also like